You are on page 1of 201

hi Comprehensive Basic Science Self-Assessment - Google Chrome x

i starttest.com/ITDVersions/19.3.0.1/ITDStart.aspx?SVC=3f50c3c0-1874-400e-af3a-5036e79dec9f

Exam Section 1: Item 1 of 50 National Board of Medical Examiners Time Remaining:


• Mark Comprehensive Basic Science Self-Assessment 4 hr 57 min 11 sec

1. A 40-year-old woman with a 25-year history of type 1 diabetes mellitus has a serum creatinine concentration of 2.5 mg/dl. A creatinine clearance test is ordered to determine the
glomerular filtration rate . This test is limited, compared to an inulin clearance test, because of which of the following properties of creatinine?

CJ A) Actively pumped into the distal convoluted tubule


B) Crosses freely into the loop of Henle
C) Reabsorbed into the collecting duct
U D) Secreted by the proximal tubule

D. Page 570 (636) of FA2019.

GFR = Clearance of inulin.


Normal GFR = 100 mL/min

Creatinine clearance overestimates GFR because creatinine is excreted by tubules.

Remember the definitions. The afferent arteriole brings many nutrients and substances to the glomerulus, where filtration happens based on pressures.
Then along the tubule, some things are reabsorbed (all sugar, proteins, etc in PCT), while some things are secreted (PAH).

Everything that was filtered and secreted gets excreted (clearance). Inulin is neither reabsorbed nor secreted so it is freely filtered and best indicates GFR.
Creatinine is secreted by the kidney, so it overestimates GFR.

~ ~ p , ,.,,.
Next Lab Values Calculator Review Help Pause
hi Comprehensive Basic Science Self-Assessment - Google Chrome x
i starttest.com/ITDVersions/19.3.0.1/ITDStart.aspx?SVC=3f50c3c0-1874-400e-af3a-5036e79dec9f

Exam Section 1: Item 2 of 50 National Board of Medical Examiners Time Remaining:


• Mark Comprehensive Basic Science Self-Assessment 4 hr 57 min 19 sec

2. A 10-year-old boy who was adopted from the Democratic Republic of Congo 2 weeks ago is brought to the physician for an initial examination. He appears slim, has thin extremities,
and is in no distress. His temperature is 37 .3°C (99.1 °F), pulse is 100/min, respirations are 20/min, and blood pressure is 118/68 mm Hg. The lungs are clear, and heart sounds are
normal. The abdomen is soft and nontender, and there is no hepatosplenomegaly. There are four 0.5-mm, firm , nontender nodules: two over the right iliac crest, one on the left thigh,
and one on the left knee. Examination of a skin snip from one of the nodules shows microfilariae. Which of the following is the most likely vector of this patient's infection?

A) Black fly
B) Bodylouse A. Page 159 (164) FA2019
u C) Cu/ex species mosquito Onchocerca volvulus by the female black fly.
U D) lxodes species tick The other answer chocies are generally related to bacteria or unrelated to any nematodes.
E) Reduviid bug
This child is presenting with practically no symptoms besides the four black skin nodules indicative of O volvulus.

, ~ ~ p , ,.,,.
Previous Next Lab Values Calculator Review Help Pause
hi Comprehensive Basic Science Self-Assessment - Google Chrome x
i starttest.com/ITDVersions/19.3.0.1/ITDStart.aspx?SVC=3f50c3c0-1874-400e-af3a-5036e79dec9f

Exam Section 1: Item 3 of 50 National Board of Medical Examiners Time Remaining:


• Mark Comprehensive Basic Science Self-Assessment 4 hr 57 min 28 sec

3. A 3-year-old girl is brought to the physician for a well-child examination. She has had a flat purplish lesion that measures 2.1 cm in diameter on the right side of her face since birth.
The lesion shows large cavernous vascular channels. Which of the following is the most likely outcome of the lesion?

CJ A) Concurrent congenital heart disease


B) Local invasion
C) Malignant degeneration
U D) Necrosis and scarring
E) Spontaneous regression
E. Page 467 (525) FA2019.
Nevus simplex. Not in First Aid.

This is not a Strawberry Hemangioma, which would be raised or nodular. But if you did think it is a strawberry hemangioma, then that also regresses.
This patient has a nevus simplex. Another similar lesion is a stork bite (port-wine stain).

These lesions disappear/regress by age 5-6 years.

, ~ ~ p , ,.,,.
Previous Next Lab Values Calculator Review Help Pause
hi Comprehensive Basic Science Self-Assessment - Google Chrome x
i starttest.com/ITDVersions/19.3.0.1/ITDStart.aspx?SVC=3f50c3c0-1874-400e-af3a-5036e79dec9f

Exam Section 1: Item 4 of 50 National Board of Medical Examiners Time Remaining:


• Mark Comprehensive Basic Science Self-Assessment 4 hr 57 min 49 sec

4. At 2:00 AM , after sleeping for 5 hours, a 32-year-old man with type 2 diabetes mellitus has a decrease in serum glucose concentration to 37 mg/dl. In response to the hypoglycemia,
cortisol secreted by the adrenal cortex induces the synthesis of which of the following enzymes in the adrenal medulla?

CJ A) Acetyl-CoA carboxylase
B) Homocysteine methyltransferase
C) Methionine adenosyltransferase
U D) Methylmalonyl-CoA racemase
E) Phenylethanolamine N-methyltransferase

E. Page 83 FA2019.

This question is a simple biochemistry question regarding mechanisms of managing hypoglycemia.


Cortisol is one of the defenses against hypoglycemia.
The question asks for an enzyme in the adrenal medulla, which synthesizes NE and Epi.
Choice E is the only relevant enzyme. It works with SAM to convert NE to Epi and is upregulated by cortisol.

You should know exactly what the other enzymes are for.
Acetyl-CoA carboxylase is the first enzyme for fatty acid synthesis and takes Acetyl CoA and yields Malonyl CoA.
Choice B is related to homocysteine becoming methionine and is related to homocystinuria.
Choice C is a relevant enzyme because it makes the SAM that PMNT uses it is not upregulated by cortisol.
Choice D, a "racemase" is probably an enzyme youve never heard of. It is an epimerase involved in fatty acid and amino acid breakdown.

, ~ ~ p , ,.,,.
Previous Next Lab Values Calculator Review Help Pause
hi Comprehensive Basic Science Self-Assessment - Google Chrome x
i starttest.com/ITDVersions/19.3.0.1/ITDStart.aspx?SVC=3f50c3c0-1874-400e-af3a-5036e79dec9f

Exam Section 1: Item 5 of 50 National Board of Medical Examiners Time Remaining:


• Mark Comprehensive Basic Science Self-Assessment 4 hr 57 min 59 sec

5. A 25-year-old woman takes an overdose of barbiturates and is found unresponsive several hours later by her husband. She is comatose and undergoes endotracheal intubation and
placement on a mechanical ventilator. Three weeks later, she opens her eyes spontaneously and has roving conjugate eye movements. She has no purposeful responses to stimuli.
Brain stem reflexes remain intact. An EEG is diffusely slow without evidence of cortical reactivity. The patient has signed a living will and, consistent with its directives, her husband
requests that mechanical ventilation be discontinued. Which of the following is the most appropriate next step in patient care?

A) Extubate the patient and discontinue mechanical ventilation; do cardiopulmonary resuscitation if her heart stops beating
B) Extubate the patient and discontinue mechanical ventilation; make no attempt to do cardiopulmonary resuscitation in case of cardiac or respiratory failure
u C) Extubate the patient and discontinue mechanical ventilation; reintubate the patient if spontaneous respirations cease
U D) Obtain a court order to maintain mechanical ventilation
E) Refer the case to the hospital ethics committee

Very simple and straightforward.


Patient is unresponsive, has no EEG activity, and no purposeful movement.
The question states "consistent with its directives" so her husband is simply requesting what the patient wanted.

The patient does not have a DNI or DNR since she was intubated. She must just be allowed to die rather than sustaining her body alive.

, ~ ~ p , ,.,,.
Previous Next Lab Values Calculator Review Help Pause
hi Comprehensive Basic Science Self-Assessment - Google Chrome x
i starttest.com/ITDVersions/19.3.0.1/ITDStart.aspx?SVC=3f50c3c0-1874-400e-af3a-5036e79dec9f

Exam Section 1: Item 6 of 50 National Board of Medical Examiners Time Remaining:


• Mark Comprehensive Basic Science Self-Assessment 4 hr 58 min 9 sec

6. A 63-year-old woman undergoes operative repair of a leaking berry aneurysm in the circle of Willis. Two days later, a CT scan of the head shows a widening of the subarachnoid
space. Which of the following changes in cerebrospinal fluid is the most likely cause of this finding?

CJ A) Decreased absorption by the choroid plexus


B) Decreased movement through the arachnoid villi
C) Decreased movement through the cerebral aqueduct
U D) Increased absorption by the choroid plexus
E) Increased movement through the arachnoid villi
F) Increased production by the choroid plexus

B. Page 510 (570) and page 491 (571).

This patient has a communicating hydrocephalus, as is evident by the fact that there is no obstruction that is impeding the CSF flow.
The patient has a widening of the subarachnoid space because the arachnoid granulations are not absorbing the CSF, most likely due to inflammatory injury.
Blood is inflammatory. Her leaking aneurysm has caused inflammation and damaged the arachnoid granulations.
The arachnoid granulations absorb CSF and send it to the dural venous sinuses, which empty into the internal jugular vein.

The other choices also do not make much sense.

The choroid plexus does not absorb CSF, it produces CSF.


Choice C is indicative of a noncommunicating (obstructive) hydrocephalus.

, ~ ~ p , ,.,,.
Previous Next Lab Values Calculator Review Help Pause
hi Comprehensive Basic Science Self-Assessment - Google Chrome x
i starttest.com/ITDVersions/19.3.0.1/ITDStart.aspx?SVC=3f50c3c0-1874-400e-af3a-5036e79dec9f

Exam Section 1: Item 7 of 50 National Board of Medical Examiners Time Remaining:


• Mark Comprehensive Basic Science Self-Assessment 4 hr 58 min 34 sec

7. A 23-year-old woman is brought to the emergency department because of shortness of breath for 2 weeks. Her respirations are 28/min. Physical examination shows no other
abnormalities. Laboratory studies show:
Serum
Na+ 135 mEq/L
K+ 4.0 mEq/L
c1- 110 mEq/L
Hco3- 15 mEq/L
Arterial pH on room air 7.25
Urine
Sodium 20 mEq/L
Chlorine 30 mEq/L
Potassium 15 mEq/L

Which of the following is the most likely diagnosis?

A) Alcoholic ketoacidosis
E. Page 580 (649) FA2019.
B) Crohn disease
This patient has a non-anion gap metabolic acidosis that she is immediately compensating for with hyperventilation.
CJ C) Diabetic ketoacidosis
You can remove A, C, D, and F since those are part of the MUDPILES.
D) Lactic acidosis Choice B would present with malabsorption symptoms.
E) Renal tubular acidosis IBD patients tend to have reduced Na and Cl absorption and increased K secretion.
F) Salicylate poisoning

, ~ ~ p , ,.,,.
Previous Next Lab Values Calculator Review Help Pause
hi Comprehensive Basic Science Self-Assessment - Google Chrome x
i starttest.com/ITDVersions/19.3.0.1/ITDStart.aspx?SVC=3f50c3c0-1874-400e-af3a-5036e79dec9f

Exam Section 1: Item 8 of 50 National Board of Medical Examiners Time Remaining:


• Mark Comprehensive Basic Science Self-Assessment 4 hr 56 min 59 sec

8. A 30-year-old woman comes to the physician because of a 2-week history of right flank pain. Her blood pressure is 168/98 mm Hg. Physical examination shows a 7-cm mass that is
palpated in the right side of the abdomen. A CT scan shows a mass in the retroperitoneal space on the right side that is compressing the renal artery. If the perfusion pressure to the
affected kidney is decreased, but glomerular filtration rate and renal plasma flow remain unchanged, which of the following mechanisms mediates the autoregulation in the renal
arterioles in this patient?

A) Decreased afferent and efferent arteriolar resistance


B) Decreased afferent arteriolar resistance
u C) Decreased efferent arteriolar resistance
U D) Increased afferent and efferent arteriolar resistance
E) Increased afferent arteriolar resistance

B. Page 571 (638) and page 295 (317) FA2019


Glomerular dynamics and autoregulation are high-yield.
For the kidney there are myogenic and tubuloglomerular feedback mechanisms of autoregulation.
The myogenic mechanism is where the afferent arteriole controls blood flow purely based off of blood pressure entering the kidney.
The tubuloglomerular feedback mechanism involves the JGA and a paracrine signaling mechanism utilizing ATP, adenosine, and NO.

For our patient, there is decreased blood entering the renal artery.
They also mention that the autoregulatory mechanism will preserve GFR and RPF.
If you constrict either arteriole, you will alter the GFR and the RPF, so delete choices D and E.

Afferent arteriole dilation (usually caused by prostaglandins, PDA) usually increases GFR and RPF but since perfusion into the kidney
is reduced, nothing will be severely altered.

, ~ ~ p , ,.,,.
Previous Next Lab Values Calculator Review Help Pause
hi Comprehensive Basic Science Self-Assessment - Google Chrome x
i starttest.com/ITDVersions/19.3.0.1/ITDStart.aspx?SVC=3f50c3c0-1874-400e-af3a-5036e79dec9f

Exam Section 1: Item 9 of 50 National Board of Medical Examiners Time Remaining:


• Mark Comprehensive Basic Science Self-Assessment 4 hr 56 min 52 sec

9. A male newborn is delivered in the hospital at 40 weeks' gestation to a 31-year-old primigravid woman following an uncomplicated spontaneous vaginal delivery. The mother
immigrated to the USA from Brazil at 20 weeks' gestation. At 8 weeks' gestation, she was evaluated because of a 1-month history of low-grade fever, marked fatigue, and diffuse
lymphadenopathy. Her symptoms resolved spontaneously, and she received no treatment. At that time, test results for Epstein-Barr virus and cytomegalovirus were negative. The
mother's only medication was a prenatal vitamin. The newborn is 33 cm (13 in; 25th percentile) long and weighs 2438 g (5 lb 6 oz; 35th percentile); head circumference is 31 cm (12
in; 3rd percentile). Temperature is 37.0°C (98.6°F), pulse is 120/min, respirations are 18/min, and blood pressure is 80/50 mm Hg. Eye examination shows chorioretinitis. CT scan of
the head shows hydrocephalus and cranial calcifications. This newborn's congenital infection was most likely acquired by the mother via which of the following modes of
transmission?

U A) Contact with fomites


B) Ingestion of undercooked meat
C) Inhalation of air droplets
u D) Kissing
E) Tick bite

B. Page 156 (161).


Toxoplasma gondii.
The mother is fine. The baby has the triad of chorioretinitis, hydrocephalus (head circumference is in 3rd percentile), and intracranial calcifications.
In the USA, ingestion of undercooked pork is the most likely answer choice.

Oocysts in cat feces are dangerous so pregnant women should avoid cat litter.
But even if contact with cat litter is an answer choice, cysts in meat is the most common route of infection and more likely.

, ~ ~ p , ,.,,.
Previous Next Lab Values Calculator Review Help Pause
hi Comprehensive Basic Science Self-Assessment - Google Chrome x
i starttest.com/ITDVersions/19.3.0.1/ITDStart.aspx?SVC=3f50c3c0-1874-400e-af3a-5036e79dec9f

Exam Section 1: Item 10 of 50 National Board of Medical Examiners Time Remaining:


• Mark Comprehensive Basic Science Self-Assessment 4 hr 56 min 44 sec

10. A 53-year-old woman comes to the physician because of a 2-week history of fainting precipitated by exercise and intermittent substernal chest pain that is relieved by rest. Her blood
pressure is 120/80 mm Hg . Cardiac examination shows a prominent left ventricular impulse. A late-peaking, harsh midsystolic murmur is heard best at the second right intercostal
space with radiation to the carotid arteries bilaterally. An ECG shows evidence of left ventricular hypertrophy. Angiography shows no coronary artery lesions. Which of the following is
the best explanation for the syncope?

A) Coronary artery compression by hypertrophic left ventricular myocardium


B) Coronary artery spasm
u C) Fixed cardiac output in spite of increased demand
U D) Mitral valve prolapse
E) Vasopressor-induced hypotension

C. Page 289 (308) FA2019).


The midsystolic murmur that radiates to the carotid arteries bilaterally is indicative of aortic stenosis.
The SAD symptoms of syncope, angina, and dyspnea are a result of not being able to increase CO when there is demand.

, ~ ~ p , ,.,,.
Previous Next Lab Values Calculator Review Help Pause
hi Comprehensive Basic Science Self-Assessment - Google Chrome x
i starttest.com/ITDVersions/19.3.0.1/ITDStart.aspx?SVC=3f50c3c0-1874-400e-af3a-5036e79dec9f

Exam Section 1: Item 11 of 50 National Board of Medical Examiners Time Remaining:


• Mark Comprehensive Basic Science Self-Assessment 4 hr 56 min 36 sec

11. A 4 7-year-old man comes to the physician because of abdominal enlargement and rectal bleeding for 3 days. He drinks approximately twelve 12-ounce cans of beer daily. Physical
examination shows scleral icterus and bleeding internal hemorrhoids. Increased pressure in which of the following veins is the most likely cause of the hemorrhoids in this patient?

CJ A) Inferior rectal vein


B) Inferior vena cava E. Page 360 (401) FA2019.
C) Internal pudenda! vein This patient has internal hemorrhoids, which are above the pectinate line. That area has the superior rectal vein (suprior above).
U D) Middle rectal vein Below the pectinate line are external painful hemorrhoids, which have the inferior and middle rectal veins (inferior below). Easy.
E) Superior rectal vein

, ~ ~ p , ,.,,.
Previous Next Lab Values Calculator Review Help Pause
hi Comprehensive Basic Science Self-Assessment - Google Chrome x
i starttest.com/ITDVersions/19.3.0.1/ITDStart.aspx?SVC=3f50c3c0-1874-400e-af3a-5036e79dec9f

Exam Section 1: Item 12 of 50 National Board of Medical Examiners Time Remaining:


• Mark Comprehensive Basic Science Self-Assessment 4 hr 56 min 27 sec

12. A new screening test for colon cancer is done in 86 patients with colon cancer and 124 persons who do not have colon cancer. The results are shown:
Colon Cancer
Present Absent
Positive 67 6 73
Test Results
Negative 19 118 137
86 124 210

The specificity of the test is which of the following? C. Page 257 (268) FA2019.
Very straightforward question with an ideal table.
A) 67/73 = 92%
Plug and chug.
U B) 67/86 = 78%
C) 118/124 = 95% Specifcity is the TNrate, which is 1- FPrate, which is TN/(TN + FP).
D) 118/137 = 86% 118/ (118+6) = 118/124 = 95%
E) 137/210 = 65%

, ~ ~ p , ,.,,.
Previous Next Lab Values Calculator Review Help Pause
hi Comprehensive Basic Science Self-Assessment - Google Chrome x
i starttest.com/ITDVersions/19.3.0.1/ITDStart.aspx?SVC=3f50c3c0-1874-400e-af3a-5036e79dec9f

Exam Section 1: Item 13 of 50 National Board of Medical Examiners Time Remaining:


• Mark Comprehensive Basic Science Self-Assessment 4 hr 56 min 13 sec

13. A clinical study is designed to evaluate the association of caffeine consumption and pancreatic cancer. In this study, alpha is set at 0.05 and beta at 0.10. Wh ich of the following best
describes the likelihood of missing an association between caffeine consumption and pancreatic cancer in this study?

CJ A) 5%
B) 10%
B. Page 262 (274) FA2019.
The "likelihood of missing an association" is a false-negative. Stating there is no effect or difference when there reall yis one is beta.
C) 85%
Type 2 error is the beta value itself, 0.10 which is 10%. Very straightforward question.
U D) 90%
E) 95% Do not confuse this with power, which is 1-beta, and that would be 90% here.

, ~ ~ p , ,.,,.
Previous Next Lab Values Calculator Review Help Pause
hi Comprehensive Basic Science Self-Assessment - Google Chrome x
i starttest.com/ITDVersions/19.3.0.1/ITDStart.aspx?SVC=3f50c3c0-1874-400e-af3a-5036e79dec9f

Exam Section 1: Item 14 of 50 National Board of Medical Examiners Time Remaining:


• Mark Comprehensive Basic Science Self-Assessment 4 hr 56 min 5 sec

14. A 68-year-old woman comes to the emergency department 2 hours after she vomited blood. Three weeks ago, she began pharmacotherapy after experiencing a vertebral
compression fracture secondary to osteoporosis. Her temperature is 36.8°C (98.2°F), pulse is 104/min, respirations are 16/min, and blood pressure is 110/60 mm Hg. Physical
examination shows mild epigastric tenderness. Endoscopy shows multiple lower esophageal erosions. Which of the following medications is the most likely cause of the
hematemesis?

A) Alendronate A. Page 475 (533) FA2019.


B) Calcitonin Bisphosphonates have significant side effects such as esophagitis, osteonecrosis of the jaw, and atypical femoral stress fractures.
u C) Calcium carbonate It is important to take this medication with abundant water and to remain upright after taking it.
U D) Conjugated estrogens
E) Raloxifene
0 F) Sodium fluoride

, ~ ~ p , ,.,,.
Previous Next Lab Values Calculator Review Help Pause
hi Comprehensive Basic Science Self-Assessment - Google Chrome x
i starttest.com/ITDVersions/19.3.0.1/ITDStart.aspx?SVC=3f50c3c0-1874-400e-af3a-5036e79dec9f

Exam Section 1: Item 15 of 50 National Board of Medical Examiners Time Remaining:


• Mark Comprehensive Basic Science Self-Assessment 4 hr 55 min 58 sec

15. A 33-year-old woman at 34 weeks' gestation has a grade 2/6 systolic ejection murmur heard at the second intercostal space. The remainder of the physical examination is
unremarkable. The best explanation for this finding is an increase in which of the following?

CJ A) Blood pressure E. Page 305 ( 331) FA2019.


B) Peripheral vascular resistance Pregnancy is one of the risk factors for dilated cardiomyopathy.
C) Pulmonary vascular resistance There is no problem with her valves or blood pressure (A and B are the same answer).
U D) Pulse An increase in pulse or HR would not cause a murmur. And she does not have pulmonary HTN (choice C).
E) Stroke volume
Dilated cardiomyopathy increases stroke volume.

, ~ ~ p , ,.,,.
Previous Next Lab Values Calculator Review Help Pause
hi Comprehensive Basic Science Self-Assessment - Google Chrome x
i starttest.com/ITDVersions/19.3.0.1/ITDStart.aspx?SVC=3f50c3c0-1874-400e-af3a-5036e79dec9f

Exam Section 1: Item 16 of 50 National Board of Medical Examiners Time Remaining:


• Mark Comprehensive Basic Science Self-Assessment 4 hr 55 min 46 sec

16. A 27-year-old man sustains a spinal cord transection at C-8 in a skiing accident. In which of the following situations is he most likely to be able to achieve and sustain an erection?

A) Erotic dreams
U B) Penile stimulation
U C) Reading erotic literature
D) Testosterone injections
E) Viewing erotic material

B. Page 612 (689) FA2019. Also -- https://www.ncbi.nlm.nih.gov/pmc/articles/PMC4896089/


The physiology of an erection involves cerebral control, autonomic control, and molecular
mechanisms. Cerebrally controlled erections are induced by erotic visual stimuli or thoughts,
which is related to choices A, C, and E. The medial preoptic area (memorized as medial penis area
and the penis is medial) and paraventricular nucleus of the hypothalamus are the main structures
involved. Dopamine is the most important brain neurotransmitter for an erection, most likely
through its stimulation of oxytocin release.

Autonomic control is through parasympathetic stimulation and central suppression of the


sympathetic nervous system. The sacral S2-S4 provide parasympathetic supply through the
sacral reflex arc that mediates an erection through tactile penile stimulation, which would work
in our patient.

Patients with a lesion above T9 cannot maintain a psychogenic erection because central
suppression to the sympathetic stimulation is necessary for that. Patients with sacral spinal cord
injury may maintain a psychogenic erection but it will be weaker than normal.

The molecular control is through NO such as through the use of viagra.

, ~ ~ p , ,.,,.
Previous Next Lab Values Calculator Review Help Pause
hi Comprehensive Basic Science Self-Assessment - Google Chrome x
i starttest.com/ITDVersions/19.3.0.1/ITDStart.aspx?SVC=3f50c3c0-1874-400e-af3a-5036e79dec9f

Exam Section 1: Item 17 of 50 National Board of Medical Examiners Time Remaining:


• Mark Comprehensive Basic Science Self-Assessment 4 hr 55 min 37 sec

17. A 40-year-old man comes to the physician because of a 6-month history of constipation and weakness. He has a family history of
colorectal carcinoma in multiple members of both sexes. Physical examination shows no abnormalities. Laboratory studies show a
hemoglobin concentration of 10 g/dl, hematocrit of 32%, and mean corpuscular volume of 73 µm 3. A colonoscopy is done, and a lesion
is found. A photograph of a section of the colon removed during subsequent colectomy is shown. Analysis shows mutations in the
MHS2 gene. Which of the following is the most likely diagnosis?

U A) Familial adenomatous polyposis


B) Fanconi anemia
U C) HNPCC syndrome
D) Li-Fraumeni syndrome
E) Neurofibromatosis, type 1
F) Xeroderma pigmentosum

C. Page 382 (425) of FA2019.


First of all, they meant MSH2 (typo).
This question is about Lynch syndrome, which usually has extensive family history.
The 3-2-1 rule of 3 relatives in 2 generations with 1 being under 50 (such as the patient himself) will all have
history of CRC. The most important thing here is that this is NON-POLYPOSIS so there will be no polyps.

The gross image is a beautiful example of what would appear as an "apple core" lesion on barium enema x-ray.
HNPCC is related to endometrial, ovarian, and skin cancers and there are many genes involved such as
MSH2, MLH1, MSH6, and PMS2.

, ~ ~ p , ,.,,.
Previous Next Lab Values Calculator Review Help Pause
hi Comprehensive Basic Science Self-Assessment - Google Chrome x
i starttest.com/ITDVersions/19.3.0.1/ITDStart.aspx?SVC=3f50c3c0-1874-400e-af3a-5036e79dec9f

Exam Section 1: Item 18 of 50 National Board of Medical Examiners Time Remaining:


• Mark Comprehensive Basic Science Self-Assessment 4 hr 55 min 26 sec

18. A 22-year-old woman comes to the physician because of nausea and vomiting for 1 week. She has a 6-month history of headaches. She works as a data entry technician and has
recently found it difficult to focus on the text. She also has had trouble looking up from her desk at the clock on the wall without moving her head. Neurologic examination shows
marked vertical gaze palsy and impaired accommodation. Horizontal gaze is normal. An MRI of the brain is most likely to show a tumor at which of the following locations?

A) Arcuate nucleus
B) Frontal cortex
C) Pineal gland
D) Pituitary gland
E) Pontine paramedian reticular formation

C. Page 516 (576) FA2019.


This patient is presenting with Parinaud syndrome, which has a vertical gaze palsy due to compression of the tectum.
She most likely has a pinealoma. This is a very straightforward and simple question.

In males, this can lead to precocious puberty and they would have elevated hCG because this is similar to germ cell tumors like a seminoma.

, ~ ~ p , ,.,,.
Previous Next Lab Values Calculator Review Help Pause
hi Comprehensive Basic Science Self-Assessment - Google Chrome x
i starttest.com/ITDVersions/19.3.0.1/ITDStart.aspx?SVC=3f50c3c0-1874-400e-af3a-5036e79dec9f

Exam Section 1: Item 19 of 50 National Board of Medical Examiners Time Remaining:


• Mark Comprehensive Basic Science Self-Assessment 4 hr 55 min 7 sec

19. In a study of antibiotic resistance, a strain of Escherichia coli resistant to ampicillin but sensitive to streptomycin is cultured with a strain of Salmonella enteritidis sensitive to
ampicillin but resistant to streptomycin. After 4 hours of cocultivation, the broth is plated onto solid medium containing both ampicillin and streptomycin. S. enteritidis bacteria that are
resistant to both antibiotics grow at a frequency of one per 100 cells. Which of the following is the most likely mechanism of the acquisition of ampicillin resistance by the
S. enteritidis?
A. Page 130 FA2019.
A) Conjugation
Conjugation is when two bacteria mate through a conjugal bridge and transfer a plasmid from one to the other.
B) Gene duplication Here the E coli is resistant to ampicillin and the Salmonella is resistant to streptomycin. They are cultured together.
U C) Point mutation Now the Salmonella is resistant to both antibiotics. This is because the E coli gave its ampicillin resistance plasmid to the Salmonella.
U D) Transduction
E) Transposition Gene duplication and point mutation are irrelevant.

Transduction involves a virus.

Transposition is an event that occurs within the same one bacteria, not between 2 different bacteria.

, ~ ~ p , ,.,,.
Previous Next Lab Values Calculator Review Help Pause
hi Comprehensive Basic Science Self-Assessment - Google Chrome x
i starttest.com/ITDVersions/19.3.0.1/ITDStart.aspx?SVC=3f50c3c0-1874-400e-af3a-5036e79dec9f

Exam Section 1: Item 20 of 50 National Board of Medical Examiners Time Remaining:


• Mark Comprehensive Basic Science Self-Assessment 4 hr 54 min 59 sec

20. A 15-month-old boy is brought to the physician by his mother because of a 9-month history of recurrent bacterial infections. The patient has not had recurrent viral infections. He is at
the 10th percentile for length and weight. Physical examination shows multiple areas of honey yellow, crusted lesions over the lower extremities. A Gram stain of one of the lesions
shows many gram-positive cocci in clusters but no leukocytes. Laboratory studies show no abnormalities except for a leukocyte count of 30,000/mm3. This patient most likely has a
rare autosomal recessive disease leading to a lack of CD18 expression. The leukocytes would be deficient in which of the following characteristics?

A) Cytokine production
E. Page 117 FA2019.
B) Helper T-cell function
This patient has LAD1, a deficiency in LFA1 integrin CD18. Autosomal recessive.
u C) lmmunoglobulin gene rearrangement
Patients present with recurrent skin and mucosal bacterial infections due to phagocyte dysfunction.
U D) Killing of intracellular bacteria There is impaired migration and chemotaxis. Delayed separation of the umbilical cord is usually a notable problem.
E) Migration

, ~ ~ p , ,.,,.
Previous Next Lab Values Calculator Review Help Pause
hi Comprehensive Basic Science Self-Assessment - Google Chrome x
i starttest.com/ITDVersions/19.3.0.1/ITDStart.aspx?SVC=3f50c3c0-1874-400e-af3a-5036e79dec9f

Exam Section 1: Item 21 of 50 National Board of Medical Examiners Time Remaining:


• Mark Comprehensive Basic Science Self-Assessment 4 hr 54 min 52 sec

21. A 10-year-old girl is brought to the physician by her mother because of increased hair growth on her face and increased muscle mass during the past 3 months. Breast development
is Tanner stage 1, and pubic hair development is Tanner stage 5. Physical examination shows clitorimegaly. Pelvic examination shows a normal-appearing vagina. Serum studies
show:
Luteinizing hormone 0.8 mlU/mL (N=1.0-5.9)
Dehydroepiandrosterone 6.2 nmol/L (N=5.2-19.8)
Dehydroepiandrosterone sulfate 0.9 µmol/L (N=0.9-3.4)
Testosterone 14 nmol/L (N=0.2-0.7)

An unregulated increase in hormone production by which of the following types of cells is the most likely cause of this patient's hirsutism?

A) Adrenal zona fasciculata cells


B) Adrenal zona glomerulosa cells D. Page 632 (711) of FA2019.
C) Ovarian follicle cells She has a SL tumor that resembles testicular histology and produces androgens, leading to virilization.
D) Ovarian Sertoli-Leydig cells The patient's presentation is classic for this tumor.
E) Pituitary acidophils
Choices A (cortisol) and B (aldosterone) are irrelevant and this patient does not have a CAH.
F) Pituitary basophils
Choices E (prolactin and GH) and F (FLAT) are irrelevant and this patient does not have a pituitary adenoma.

, ~ ~ p , ,.,,.
Previous Next Lab Values Calculator Review Help Pause
hi Comprehensive Basic Science Self-Assessment - Google Chrome x
i starttest.com/ITDVersions/19.3.0.1/ITDStart.aspx?SVC=3f50c3c0-1874-400e-af3a-5036e79dec9f

Exam Section 1: Item 22 of 50 National Board of Medical Examiners Time Remaining:


• Mark Comprehensive Basic Science Self-Assessment 4 hr 54 min 45 sec

22. A 45-year-old man is brought to the emergency department 2 hours after the sudden onset of severe pain in the left flank that awakened him from sleep. He reports that the pain
moved over the next 30 minutes to the left lower abdomen and now is principally localized in the scrotum. Physical examination shows no palpable masses in the abdomen or the
scrotum. There is no rebound tenderness of the abdominal wall. Which of the following is the most likely cause of the pain?

A) Appendicitis
B) Carcinoma of the testis
C) Diverticulitis of the descending colon
D) Renal calculi in the left ureter
E) Testicular torsion

D.
Pretty obvious. The pain was of sudden-onset. It started in the left flank and moved to the scrotal area because the stone travelled to the penis.
No palpable masses in the abdomen or scrotum rules out any tumors. No rebound tenderness rules out anything related to the appendix.

Classic "loin to groin" pain of nephrolithiasis.

, ~ ~ p , ,.,,.
Previous Next Lab Values Calculator Review Help Pause
hi Comprehensive Basic Science Self-Assessment - Google Chrome x
i starttest.com/ITDVersions/19.3.0.1/ITDStart.aspx?SVC=3f50c3c0-1874-400e-af3a-5036e79dec9f

Exam Section 1: Item 23 of 50 National Board of Medical Examiners Time Remaining:


• Mark Comprehensive Basic Science Self-Assessment 4 hr 54 min 38 sec

23. A 38-year-old woman , gravida 4, para 4, comes to the physician because of a 6-month history of urinary incontinence when sneezing and coughing. Physical examination shows
normal-appearing female external genitalia. When the patient is asked to perform a Valsalva maneuver, leakage of urine from the urethra is detected. She is given instructions to
perform Kegel exercises to strengthen the muscles of the perineum. Which of the following muscles in this patient is most likely to remain unaffected by the exercises?

A) Bulbospongiosus
B) Deep transverse perinea! muscle
C) External urethral sphincter
D) Internal anal sphincter
E) lschiocavernosus

D. Page 588 (660) FA2019.


This patient has stress incontinence, as is evident from her urine leakage after extensive intra-abdominal pressure from a cough or sneeze.
The problem is due to urethral hypermobility or the intrinsic urethral sphincter.
Kegel exercises strengthen the pelvic floor. The main targets for treatment are the levator ani muscles; iliococcygeus, pucococcygeus, & puborectalis.
Pessaries are another treatment option.

Regardless, any exercise will generally strengthen skeletal muscle. The internal anal sphincter is smooth muscle under autonomic control.
It will not strengthen from exercise.
The external anal sphincter, on the other hand, is under pudendal nerve control (feces leakage during delivery) and would strengthen from exercise.

, ~ ~ p , ,.,,.
Previous Next Lab Values Calculator Review Help Pause
hi Comprehensive Basic Science Self-Assessment - Google Chrome x
i starttest.com/ITDVersions/19.3.0.1/ITDStart.aspx?SVC=3f50c3c0-1874-400e-af3a-5036e79dec9f

Exam Section 1: Item 24 of 50 National Board of Medical Examiners Time Remaining:


• Mark Comprehensive Basic Science Self-Assessment 4 hr 54 min 29 sec

24. A 1-week-old newborn is brought to the physician because of poor feeding , vomiting, and progressive lethargy during the past 4 days. She was born at term ; pregnancy, labor, and
delivery were uncomplicated, and she had no congenital anomalies. She is being breast-fed. She has a healthy 2-year-old brother; a sister died at the age of 10 days after a full-term
birth. Physical examination shows decreased muscle tone and poor responsiveness; reflexes are normal. Her serum bicarbonate concentration is 8 mEq/L, pH is 7.15, and plasma
ammonia concentration is 10 times the upper limit of normal. Which of the following is the most likely cause?

A) Mitochondrial disorder
B) Mucopolysaccharidosis disorder
u C) Organic acid metabolism disorder
U D) Renal tubular acidosis
E) X-linked leukodystrophy

C. Page 82 and 85 FA2019. Page 85 of FA2020.


This patient has hyperammonemia. Everything else is alright though.
This is most likely some organic acid metabolism issue. There are plenty, like propionic acidemia, methylmalonic acidemia, etc.

In methylmalonic acidemia patients have hypoglycemia (accumulation of methylmalonate inhibits gluconeogenesis), ketosis (to make up for
hypoglycemia), and hyperammonemia (accumulation of methylmalonate inhibits the urea cycle).

, ~ ~ p , ,.,,.
Previous Next Lab Values Calculator Review Help Pause
hi Comprehensive Basic Science Self-Assessment - Google Chrome x
i starttest.com/ITDVersions/19.3.0.1/ITDStart.aspx?SVC=3f50c3c0-1874-400e-af3a-5036e79dec9f

Exam Section 1: Item 25 of 50 National Board of Medical Examiners Time Remaining:


• Mark Comprehensive Basic Science Self-Assessment 4 hr 54 min 23 sec

25. A 45-year-old man comes to the physician because of a 3-day history of fever and muscle pain. He recently went on a hunting expedition to northern Canada, where he consumed
polar bear meat. His temperature is 39.5°C (103.1 °F). Physical examination shows periorbital edema and muscle tenderness. Which of the following tests is most likely to establish
the diagnosis?

A) Eosinophil count E. Page 159 (164) FA2019.


B) Examination of the stool for ova This patient has the classic symptoms of Trichinella spiralis.
C) Gram stain of the stool The nematode travels the bloodstream and enters striated muscle.
D) X-ray survey of large muscular masses
E) Muscle biopsy

, ~ ~ p , ,.,,.
Previous Next Lab Values Calculator Review Help Pause
hi Comprehensive Basic Science Self-Assessment - Google Chrome x
i starttest.com/ITDVersions/19.3.0.1/ITDStart.aspx?SVC=3f50c3c0-1874-400e-af3a-5036e79dec9f

Exam Section 1: Item 26 of 50 National Board of Medical Examiners Time Remaining:


• Mark Comprehensive Basic Science Self-Assessment 4 hr 54 min 16 sec

26. A 71-year-old woman with non-Hodgkin lymphoma is diagnosed with lymphomatous meningitis. Her serum creatinine concentration is 2.3 mg/dl. A course of intrathecal
methotrexate is planned. Intravenous administration of which of the following adjuvant agents is most appropriate for this patient?

CJ A) Folic acid
B) Leucovorin
C) Vitamin B 1 (thiamine)
U D) Vitamin 8 6 (pyridoxine)
E) Vitamin C

B. Page 432 (478).


Methotrexate is a folic acid analog that competes with folate and prevents
DHF reductase from making dTMP, inhibiting DNA synthesis to treat cancers.

If you give these patients concomitant folic acid, it will not work since
methotrexate will just compete with it.
Leucovorin is 5-formyl-THF that works downstream from DHF reductase,
bypassing the step that methotrexate inhibits.
Leucovorin effectively works like folate and prevents the myelosuppression.

In comparison, you must give leucovorin the patients taking 5FU because
leucovorin increases the binding of 5FU with its target, thymidylate synthase.

, ~ ~ p , ,.,,.
Previous Next Lab Values Calculator Review Help Pause
hi Comprehensive Basic Science Self-Assessment - Google Chrome x
i starttest.com/ITDVersions/19.3.0.1/ITDStart.aspx?SVC=3f50c3c0-1874-400e-af3a-5036e79dec9f

Exam Section 1: Item 27 of 50 National Board of Medical Examiners Time Remaining:


• Mark Comprehensive Basic Science Self-Assessment 4 hr 54 min 10 sec

27. A 55-year-old woman is brought to the physician by her husband because of a change in behavior during the past 2 days. Her husband says, "Yesterday, she didn't recognize a
picture of her own mother." Neurologic examination shows that she has an inability to recognize objects unless she touches them or hears the sound that they make. These findings
most likely indicate a lesion involving the area supplied by which of the following arteries?

A) Anterior cerebral
B) Anterior choroidal
C) Lenticulostriate
D) Posterior cerebral
E) Thalamoperforating

D.
The patient is clearly having difficulty with visual association. She is seeing things but not sure what it is related to until touching it.
This is prosopagnosia due to a PCA stroke. Since its a visual problem for the most part, but neurologic exam is fine otherwise, you can choose PCA.

ACA has loss of sensation and paralysis in legs, while MCA has loss of sensation and paralysis in the face and arms.
Lenticulostriate is related to the striatum and internal capsule and often has lacunar infarcts due to hypertension. Would just have paralysis.
Anterior choroidal would have loss of sensation and paralysis also.

, ~ ~ p , ,.,,.
Previous Next Lab Values Calculator Review Help Pause
hi Comprehensive Basic Science Self-Assessment - Google Chrome x
i starttest.com/ITDVersions/19.3.0.1/ITDStart.aspx?SVC=3f50c3c0-1874-400e-af3a-5036e79dec9f

Exam Section 1: Item 28 of 50 National Board of Medical Examiners Time Remaining:


• Mark Comprehensive Basic Science Self-Assessment 4 hr 54 min 5 sec

28. A 24-year-old woman who has diffuse toxic goiter (Graves disease) undergoes a partial thyroidectomy. Several months later, she develops muscle cramps, tetany, and
hypocalcemia. Which of the following sets of laboratory findings in serum is most consistent with these findings?

Parathyroid
Phosphate Hormone 25-Hydroxyvitamin D
A) i i l
B) i l normal
C) i l l
D) l i normal
E) l l l

B. Page 330/360 discusses PTH. Page 341/373 discusses PTH disease.


This patient's parathyroid glands were probably removed along with the thyroidectomy.
PTH is phosphate trashing hormone, so without PTH the phosphate will be high. PTH is obviously gonna be low.
Vitamin D is not related to PTH levels.

, ~ ~ p , ,.,,.
Previous Next Lab Values Calculator Review Help Pause
hi Comprehensive Basic Science Self-Assessment - Google Chrome x
i starttest.com/ITDVersions/19.3.0.1/ITDStart.aspx?SVC=3f50c3c0-1874-400e-af3a-5036e79dec9f

Exam Section 1: Item 29 of 50 National Board of Medical Examiners Time Remaining:


• Mark Comprehensive Basic Science Self-Assessment 4 hr 53 min 56 sec

29. A 56-year-old man has a 3-year history of progressive memory loss followed by aphasia, visuospatial disorientation, and inappropriate behavior. His father died in a nursing home at
age 65, 6 years after the onset of progressive dementia. His 55-year-old brother has had some difficulty remembering recent events and recalling familiar names. The patient dies 5
years after the onset of these symptoms. At autopsy, examination of the brain shows cerebral atrophy, predominantly in the mesial temporal and parietal cortices; in these areas
there are neurofibrillary tangles , neuritic plaques, and marked neuronal loss. This disorder most likely indicates a mutation in a gene encoding for which of the following molecules?

A) Amyloid A protein
B) ~ 2-Microglobulin
u C) Neurofilament protein
U D) Presenilin

D. Page 508/568.
This patient has the familial form of Alzheimer's disease (neurofibrillary tangles of tau protein, neuritic plaques of beta amyloid,
and neuronal loss due to cortical atrophy). The familial form is due to presenilin1 and 2 as well as APP.

, ~ ~ p , ,.,,.
Previous Next Lab Values Calculator Review Help Pause
hi Comprehensive Basic Science Self-Assessment - Google Chrome x
i starttest.com/ITDVersions/19.3.0.1/ITDStart.aspx?SVC=3f50c3c0-1874-400e-af3a-5036e79dec9f

Exam Section 1: Item 30 of 50 National Board of Medical Examiners Time Remaining:


• Mark Comprehensive Basic Science Self-Assessment 4 hr 53 min 48 sec

30. A 60-year-old woman is receiving cisplatin therapy for advanced transitional cell bladder cancer. She develops paresthesias, and the medication
is stopped. The medication likely damaged the largest cells in the region labeled "B" in the drawing of the spinal cord shown. The damage to
these cells would most likely lead to which of the following motor signs?

A) Babinski sign
B) Clonus
U C) Fibrillations
0 D) Hyporeflexia
U E) Muscle atrophy

D. Page 518/601.
Area B is the dorsal root ganglion.
This area carries afferents.
This is basically what you check for with the knee jerk.
Hence, that reflex would be missing in severe DRG damage.

, ~ ~ p , ,.,,.
Previous Next Lab Values Calculator Review Help Pause
hi Comprehensive Basic Science Self-Assessment - Google Chrome x
i starttest.com/ITDVersions/19.3.0.1/ITDStart.aspx?SVC=3f50c3c0-1874-400e-af3a-5036e79dec9f

Exam Section 1: Item 31 of 50 National Board of Medical Examiners Time Remaining:


• Mark Comprehensive Basic Science Self-Assessment 4 hr 53 min 41 sec

31. A 30-year-old woman comes to the physician for follow-up treatment of chronic renal failure and normocytic, normochromic anemia. Following the administration of recombinant
human erythropoietin, her hemoglobin concentration increases in part due to increased activity of 5-aminolevulinic acid (o-ALA) synthase in erythrocytes. This enzyme regulates the
reaction involving the condensation of which of the following compounds?

A) Acetate and hexacarboxylic porphyrinogen


B) Ferrous ion and protoporphyrin
C. Page 417/461.
C) Glycine and succinyl CoA
Very straightforward question about heme synthesis.
D) Lysine and o-ALA
The first step involving ALA synthase makes ALA by combining glycine and succinyl-CoA.
E) Zinc and porphobilinogen

, ~ ~ p , ,.,,.
Previous Next Lab Values Calculator Review Help Pause
hi Comprehensive Basic Science Self-Assessment - Google Chrome x
i starttest.com/ITDVersions/19.3.0.1/ITDStart.aspx?SVC=3f50c3c0-1874-400e-af3a-5036e79dec9f

Exam Section 1: Item 32 of 50 National Board of Medical Examiners Time Remaining:


• Mark Comprehensive Basic Science Self-Assessment 4 hr 53 min 35 sec

32. An investigator is studying an outbreak of Escherichia coli 0157:H7 among university students who had eaten at the on-site cafeteria. The investigators administer a food
questionnaire to 100 students with E. coli 0157:H7 and 100 healthy control subjects. Two exposures emerge as possible concerns: cookies made from frozen commercial dough
(odds ratio=6.0; p<0.001) and milk from a local farm vendor (odds ratio=3.9; p<0.001 ). The exposure data is stratified. Results show:
Drank Milk Did Not Drink Milk
Patients Controls Patients Controls
72 36 8 4
Ate Cookies Yes
No 2 6 18 54
Odds ratio=1.0 Odds ratio=6.0
MISTAKE. OR= 6.0

Ate Cookies Did Not Eat Cookies


Patients Controls Patients Controls
Yes 72 36 2 6
Drank Milk
No 8 4 18 54
Odds ratio=1 .0 Odds ratio=1.0

Which of the following conclusions can be drawn from these data?

A) Both milk and cookies are independently associated with E. coli cases
B) No confounding exists between milk and cookies
C) Only cookies are independently associated with E. coli cases
D) Only milk is independently associated with E. coli cases

C. Page 258/269.
The odds ratio tells you the odds of something being related to another thing. Used in case-control studies. (Case-contORl).
When the odds ratio is 1, then both things are equally likely. That means, that eating a cookie, or drinking milk are equally likely to cause E
coli.
When the odds ratio is more than 1, then the relevant exposure is related to the outcome.

, ~ ~ p , ,.,,.
Previous Next Lab Values Calculator Review Help Pause
hi Comprehensive Basic Science Self-Assessment - Google Chrome x
i starttest.com/ITDVersions/19.3.0.1/ITDStart.aspx?SVC=3f50c3c0-1874-400e-af3a-5036e79dec9f

Exam Section 1: Item 33 of 50 National Board of Medical Examiners Time Remaining:


• Mark Comprehensive Basic Science Self-Assessment 4 hr 53 min 28 sec

33. A 68-year-old woman comes to the physician because of a 9-month history of abdominal pain after meals; she also has had an 11-kg (24-lb) weight loss during this period. She says
she avoids eating because of cramping pain that lasts for approximately 1 hour after meals. She underwent coronary artery bypass grafting for angina 6 years ago. Current
medications include aspirin, atenolol, and simvastatin. Her pulse is 62/min, respirations are 16/min, and blood pressure is 160/90 mm Hg. Physical examination shows a thin body
frame and a soft, nontender abdomen; there are no masses and no organomegaly. A CT scan of the abdomen shows a 3-cm ectatic aorta. Which of the following pairs of arteries is
most likely involved in this patient's condition?

A) Celiac and inferior mesenteric


B) Inferior mesenteric and hepatic
C) Inferior mesenteric and superior mesenteric
lJ D) Inferior phrenic and celiac
U E) Superior mesenteric and hepatic

C. Page 357/397 FA2019.

The mesenteric circulation consists of three primary vessels that supply blood to the small and large bowel: the celiac artery,
superior mesenteric artery (SMA), and inferior mesenteric artery (IMA).
Blood flow through these arteries increases within an hour after eating due to an increase in metabolic demand of the intestinal mucosa.
Chronic occlusion of a single vessel allows collateral blood flow to compensate, thus symptoms do not typically present until at least two
primary vessels are occluded.

The 3-cm ectatic aorta means she has an abdominal aortic aneurysm.
Generally, they tend to occur below the renal branches so the two closest branches are SMA (slightly above renal) and IMA (below renal).
Also, splenic flexure is a common watershed area supplied by SMA and IMA so involvement of both can cause her symptoms.

, ~ ~ p , ,.,,.
Previous Next Lab Values Calculator Review Help Pause
hi Comprehensive Basic Science Self-Assessment - Google Chrome x
i starttest.com/ITDVersions/19.3.0.1/ITDStart.aspx?SVC=3f50c3c0-1874-400e-af3a-5036e79dec9f

Exam Section 1: Item 34 of 50 National Board of Medical Examiners Time Remaining:


• Mark Comprehensive Basic Science Self-Assessment 4 hr 53 min 17 sec

34. A 12-year-old boy is brought to the physician by his mother because of a persistent rash around his nose and mouth for 4 months. He also has had a decreased appetite and an
unintentional 4.5-kg (10-lb) weight loss during the past 3 months. His mother states, "The rash developed shortly after he started at his new school. I wonder if he keeps getting into
something. I've tried hydrocortisone cream, but it didn't work." She adds, "And now his grades are falling, and he's not eating as much as he used to. I hope he's not depressed
because of the rash." The rash began as mild redness and then became dry and started to flake. The patient says there is no pain or discomfort with the rash. Physical examination
shows injection of both conjunctivae, an eczematous eruption extending inferiorly from the nasolabial fold to the upper lip, and an erythematous nasal mucosa that is friable with gold
stippling. When asked, he states that his mood is fine. Which of the following is the most likely cause of this patient's condition?

A) Atopic dermatitis
C. NOT in FA.
B) Impetigo
NBME likes asking about kids abusing glue inhalation.
C) Inhalant abuse
Any child with an unexplainable suspicious rash on the face and a decline in intelligence is most likely inhaling glue.
() D) Mercury poisoning Below are people who were caught huffing inhalants such as glue, paint, markers, and other solvents.
u E) Wegener granulomatosis

, ~ ~ p , ,.,,.
Previous Next Lab Values Calculator Review Help Pause
hi Comprehensive Basic Science Self-Assessment - Google Chrome x
i starttest.com/ITDVersions/19.3.0.1/ITDStart.aspx?SVC=3f50c3c0-1874-400e-af3a-5036e79dec9f

Exam Section 1: Item 35 of 50 National Board of Medical Examiners Time Remaining:


• Mark Comprehensive Basic Science Self-Assessment 4 hr 53 min 10 sec

35. A 26-year-old man is brought to the emergency department 40 minutes after being involved in a motor vehicle collision. His blood pressure is 90/60 mm Hg. Systolic blood pressure
decreases 20 mm Hg with inspiration. Physical examination shows jugular venous distention. There are distant heart sounds. Which of the following is the most likely diagnosis?

CJ A) Hemothorax
B) Myocardial infarction C. Pg 309/337.
C) Pericardia! tamponade The patient is presenting with pulsus paradoxus, which is a drop in systolic BP by >10mmHg during inspiration.
U D) Pneumothorax
Recall that during inspiration, blood returns to the right heart.
In cardiac tamponade, fluid around the heart prevents expansion of the heart so it is unable to sustain the increased return.
E) Pulmonary edema
This is why the patient has JVD as well.

Tamponade is french for compression. All the chambers are being compressed
by fluid in the pericardial space (red arrows in image at bottom). As this continues
the cardiac chambers decrease in size and diastolic filling is impaired, one part of
Beck triad; hypotension. Back up of blood causes distended neck veins.
Distant heart sounds are due to muffling of sounds by fluid.

Usually when we take a breath in, increase in venous return to the right heart
increases right heart volume. Usually the increased volume of RA distends into
pericardial space, but in tamponade that is not possible so the RV pushes the
IV septum into the LV, decreasing volume in the LV and that means you have
significant drop in systolic BP of more than 10mmHg because the LV will have
trouble pumping into the aorta. You also note JVD. In asthma and other
obstructive lung diseases, there is amplification of intrathoracic pressure
variation during respiration, which is then transmitted to the distal arteries.

, ~ ~ p , ,.,,.
Previous Next Lab Values Calculator Review Help Pause
hi Comprehensive Basic Science Self-Assessment - Google Chrome x
i starttest.com/ITDVersions/19.3.0.1/ITDStart.aspx?SVC=3f50c3c0-1874-400e-af3a-5036e79dec9f

Exam Section 1: Item 36 of 50 National Board of Medical Examiners Time Remaining:


• Mark Comprehensive Basic Science Self-Assessment 4 hr 53 min 3 sec

36. Forty of 100 attendees at a company Christmas party develop explosive, watery diarrhea followed by abdominal cramps and vomiting 12 to 48 hours after the party. About half of the
affected individuals also had headache. All of the affected individuals reported feeling better within several days. All of the affected individuals, and 6 of the 60 unaffected individuals,
consumed raw oysters. There was no other significant association between the consumption of specific foods and symptoms. Which of the following is the most likely causal
organism?

A) Bacillus cereus
B) Campy/obacter jejuni
u C) C/ostridium pertringens
U D) Staphylococcus aureus
E) Vibrio parahaemolyticus

E. Page 146/148.
V cholerae other V vulnificus as well as V parahaemolyticus are all related to consumption of undercooked seafood like shellfish or oysters.

B cereus is related to reheated rice and usually causes vomiting but can also cause diarrhea within 8-18 hours.
C jejuni is related to undercooked meat or contact with animals. Plus this causes bloody diarrhea.
C perfringens is related to food left standing, allowing the spores to germinate and create heat-labile enterotoxin. Gas gangrene is a complication.
S aureus is related to mayonnaise and dairy products and would have rapid onset diarrhea.

, ~ ~ p , ,.,,.
Previous Next Lab Values Calculator Review Help Pause
hi Comprehensive Basic Science Self-Assessment - Google Chrome x
i starttest.com/ITDVersions/19.3.0.1/ITDStart.aspx?SVC=54cd3e5f-e4bf-49d0-8371-ba261251f2a1

Exam Section 2: Item 37 of 50 National Board of Medical Examiners Time Remaining:


• Mark Comprehensive Basic Science Self-Assessment 4 hr 55 min 34 sec

37. A 57-year-old woman requires intubation in the intensive care unit after developing progressive dyspnea and hypoxemia 45 minutes after the initiation of a transfusion of packed red
blood cells. The patient has cirrhosis and was admitted to the hospital because of melena; the transfusion was initiated after she was found to have a hemoglobin concentration of
6.4 g/dl. Medical history includes hepatitis C diagnosed at the age of 41 years. She has a 25-year history of illicit intravenous drug use. The patient is intubated and sedated. Her
temperature is 36.9°C (98.4 °F), pulse is 92/min , respirations are 16/min, and blood pressure is 94/58 mm Hg. There is no jugular venous distention. Bilateral basilar crackles are
heard on inspiration; there are no wheezes. Physical examination shows spider angiomata on the chest and abdomen, minimal ascites, and trace bilateral pedal edema. Chest x-ray
shows bilateral diffuse airspace disease without pleural effusions or cardiomegaly. This patient most likely developed which of the following types of transfusion reactions?

A) Acute hemolytic reaction B. Page 114 FA2019.


B) Acute lung injury This patient clearly had transfusion related acute lung injury (TRALI).
C) Anaphylactic reaction TRALI occurs due to a 2 hit mechanism with neutrophil priming and then neutrophil activation leading to pulmonary edema.
() D) Rh incompatibility The patient is otherwise fine.
u E) Serum sickness This topic was in NBME 21 and NBME 22 as well.

, ~ ~ p , ,.,,.
Previous Next Lab Values Calculator Review Help Pause
hi Comprehensive Basic Science Self-Assessment - Google Chrome x
i starttest.com/ITDVersions/19.3.0.1/ITDStart.aspx?SVC=3f50c3c0-1874-400e-af3a-5036e79dec9f

Exam Section 1: Item 38 of 50 National Board of Medical Examiners Time Remaining:


• Mark Comprehensive Basic Science Self-Assessment 4 hr 52 min 26 sec

38. An investigator is conducting a study of a novel protein in an experimental animal model. It is found that this protein interferes with the normal structure of desmosomes (maculae
adherentes). Which of the following cell junctions in skin are most likely to be affected by the presence of this protein?

CJ A) Basal keratinocyte:lamina lucida


B) Basal keratinocyte:suprabasal keratinocyte
C) Granular keratinocyte:stratum corneum
U D) Lamina lucida:lamina densa
E) Melanocyte:basal keratinocyte

B. Page 471/529.
This question about desmosomes is relevant to pemphigus vulgaris, which has antibdoies against desmogleins that make desmosomes.
Loss of desmosomes separates the keratinocytes from the stratum spinosum.

, ~ ~ p , ,.,,.
Previous Next Lab Values Calculator Review Help Pause
hi Comprehensive Basic Science Self-Assessment - Google Chrome x
i starttest.com/ITDVersions/19.3.0.1/ITDStart.aspx?SVC=3f50c3c0-1874-400e-af3a-5036e79dec9f

Exam Section 1: Item 39 of 50 National Board of Medical Examiners Time Remaining:


• Mark Comprehensive Basic Science Self-Assessment 4 hr 52 min 11 sec

Herpangina

39. An otherwise healthy 45-year-old man comes to the office because of a 10-year history of episodes of painful sores in his mouth that recur every few months. He has had no skin
lesions, genital or anal ulcers, gastrointestinal or joint problems, or fever. Physical examination shows the findings in the photograph. Which of the following is the most likely cause of
this patient's condition?
A. Page 310/338.
A) Aphthous ulcers
This patient most likely has Behcet syndrome. The image is perfect. So is the question stem.
B) Herpangina Herpangina is due to coxsackie A and causes oral ulcers on the palates.
C) Herpes zoster Lack of genital or skin lesions rules out HSV but that is not even an answer choice, how nice of them.
D) Secondary syphilis Herpes zoster would have a rash following a half-body dermatome in some immunocompromised elderly person.
Secondary syphilis causes gummas, irrelevant here.

This patient simply has recurrent aphthous ulcers that come and go and could possibly be due to HSV or parvovirus.

, ~ ~ p , ,.,,.
Previous Next Lab Values Calculator Review Help Pause
hi Comprehensive Basic Science Self-Assessment - Google Chrome x
i starttest.com/ITDVersions/19.3.0.1/ITDStart.aspx?SVC=3f50c3c0-1874-400e-af3a-5036e79dec9f

Exam Section 1: Item 40 of 50 National Board of Medical Examiners Time Remaining:


• Mark Comprehensive Basic Science Self-Assessment 4 hr 52 min 3 sec

40. A 25-year-old woman is brought to the emergency department by her husband 30 minutes after losing consciousness at home. She had been home from work for the past 5 days
because of fever. She has a history of recurrent upper urinary tract infections. Her temperature is 39.4 °C (103°F), pulse is 135/min, and blood pressure is 82/48 mm Hg. Diffuse
crackles are heard over both lung fields. Arterial blood gas analysis on room air shows:
pH 7.18
Pco 2 32 mm Hg
Po 2 64 mm Hg

A blood culture grows gram-negative rods. A chest x-ray shows diffuse bilateral infiltrates. Which of the following is the most likely cause of this patient's pulmonary symptoms?

u A) Hypoventilation
E. Page 664
B) Hypoxic vasoconstriction
Even with incomplete understanding of what is happening, you come down to choices B and E.
C) Left ventricular failure
The gram negative rods causing her UTI most likely are E coli. She might have E coli sepsis leading to ARDS.
lJ D) Paradoxical thromboembolism The ARDS is due to pulmonary capillary leakage. Noncardiogenic pulmonary edema.
E) Pulmonary capillary leakage The patient clearly has a low pH (acidosis) and the low pCO2 is due to hyperventilation as a result of the hypoxemia.

, ~ ~ p , ,.,,.
Previous Next Lab Values Calculator Review Help Pause
hi Comprehensive Basic Science Self-Assessment - Google Chrome x
i starttest.com/ITDVersions/19.3.0.1/ITDStart.aspx?SVC=3f50c3c0-1874-400e-af3a-5036e79dec9f

Exam Section 1: Item 41 of 50 National Board of Medical Examiners Time Remaining:


• Mark Comprehensive Basic Science Self-Assessment 4 hr 51 min 56 sec

41. A 13-year-old girl who has a 6-year history of type 1 diabetes mellitus has had poor glucose control for the past 2 months. Previously, the diabetes was well controlled. Her parents
say that the stress of her illness is causing family problems and that they have been fighting with each other and their daughter, especially at meals. The parents argue with each
other while in the physician's office. The patient takes her insulin as prescribed but is having difficulty following the prescribed diet. Which of the following is the most appropriate
next step?

A) Advise the parents to stop bickering


B) Discuss further the impact of the patient's illness on the family
u C) Readjust the patient's sliding-scale insulin coverage
U D) Recommend individual psychotherapy for the patient
E) Refer both the patient and her parents to a dietician
0 F) Refer the patient to a dietician

B. Ethics.
Give away question.
More discussion is needed as to why there are continuous fights during meals and if the parents might be eventually divorcing.
Is the issue a lack of understanding how to manage the childs disease? Or are the parents falling apart?

, ~ ~ p , ,.,,.
Previous Next Lab Values Calculator Review Help Pause
hi Comprehensive Basic Science Self-Assessment - Google Chrome x
i starttest.com/ITDVersions/19.3.0.1/ITDStart.aspx?SVC=3f50c3c0-1874-400e-af3a-5036e79dec9f

Exam Section 1: Item 42 of 50 National Board of Medical Examiners Time Remaining:


• Mark Comprehensive Basic Science Self-Assessment 4 hr 51 min 51 sec

42. A 90-year-old man has a 1-week history of constipation and abdominal swelling. He is afebrile. A 4-cm mass is palpated in the right groin. The mass is not reducible, but moderately
increased tympanitic bowel sounds are audible over it. Which of the following is the most likely diagnosis?

CJ A) Appendiceal abscess
B) Carcinoma of the colon
C) Diverticulitis
U D) Fecal impaction
E) Incarcerated inguinal hernia

E. Page 364/405 FA2019.


This patient definitely does not have A, B, or C.
Fecal impaction is the only other plausible thought amidst the exhaustion of an exam and amidst this terrible NBME board of question writers.
However, fecal impaction would be in the colon or rectum, not the groin as in this patient. The groin is where inguinal hernias occur.
This is an extremely old patient so this might be a direct inguinal hernia.
However, given the fact that this was not reducible and it is incarcerated, this might be a femoral hernia.

, ~ ~ p , ,.,,.
Previous Next Lab Values Calculator Review Help Pause
hi Comprehensive Basic Science Self-Assessment - Google Chrome x
i starttest.com/ITDVersions/19.3.0.1/ITDStart.aspx?SVC=3f50c3c0-1874-400e-af3a-5036e79dec9f

Exam Section 1: Item 43 of 50 National Board of Medical Examiners Time Remaining:


• Mark Comprehensive Basic Science Self-Assessment 4 hr 51 min 44 sec

43. A 45-year-old man with hypercholesterolemia (LDL 260 mg/dl) requires pharmacotherapy. A 3-hydroxy-3-methylglutaryl (HMG) CoA reductase inhibitor will be prescribed. Which of
the following is the mechanism of action of this treatment?

CJ A) Decreased synthesis of HDL cholesterol


B) Enhanced activity of lipoprotein lipase
C) Increased synthesis of bile acids
U D) Increased synthesis of LDL receptors
E) Sequestration of bile acids in the gastrointestinal tract

D. Page 315/344 FA2019.


This is a straightforward question about the MOA of statins.
Statins inhibit the conversion of HMG CoA to mevalonate, a cholesterol precursor.
Now you start running out of that and cant make cholesterols in the liver, so you make more LDL receptors to pick up cholesterols into the liver.
You also ramp up HMGCoA reductase production. This medication can be hepatotoxic.

, ~ ~ p , ,.,,.
Previous Next Lab Values Calculator Review Help Pause
hi Comprehensive Basic Science Self-Assessment - Google Chrome x
i starttest.com/ITDVersions/19.3.0.1/ITDStart.aspx?SVC=3f50c3c0-1874-400e-af3a-5036e79dec9f

Exam Section 1: Item 44 of 50 National Board of Medical Examiners Time Remaining:


• Mark Comprehensive Basic Science Self-Assessment 4 hr 51 min 38 sec

44. A 19-year-old man has had weakness of the muscles of his neck and extremities for the past week. Bladder and bowel function is normal. Position sense is impaired. Which of the
following is the most likely diagnosis?

CJ A) Amyotrophic lateral sclerosis


B) Parkinson disease
C) Poliomyelitis
U D) Polyneuropathy
E) Syringomyelia
D. Page 518/578.
This question is simply a matter of eliminating the other choices.
ALS would have no sensory or bowel/bladder deficits, so this was an excellent bait. However, ALS is not about weakness of muscles.
ALSO has UMN and LMN degeneration that causes a large array of symptoms, not just neck and extremity muscle weakness.
Parkinson is irrelevant in this question. Patient would have tremor or rigidity.
Poliomyelitis is irrelevant (pg 19/579). This would present with asymmetric LMN weakness and respiratory failure.
Syringomyelia is irrelevant. Patient would have symmetrical loss of pain and temperature in the cape distribution.

, ~ ~ p , ,.,,.
Previous Next Lab Values Calculator Review Help Pause
hi Comprehensive Basic Science Self-Assessment - Google Chrome x
i starttest.com/ITDVersions/19.3.0.1/ITDStart.aspx?SVC=3f50c3c0-1874-400e-af3a-5036e79dec9f

Exam Section 1: Item 45 of 50 National Board of Medical Examiners Time Remaining:


• Mark Comprehensive Basic Science Self-Assessment 4 hr 51 min 30 sec

45. A 72-year-old woman with coronary artery disease comes to the physician because of a 2-month history of progressive angina symptoms with exertion. Five months ago, the patient
underwent stent placement for significant stenoses of the proximal anterior interventricular (left anterior descending) and right coronary arteries. She remained symptom-free for
3 months. Her pulse is 76/min and regular, and blood pressure is 135/85 mm Hg. An ECG at rest shows no abnormalities; an exercise stress test shows ST-segment changes in
leads II, Ill , and aVF. Which of the following is the most likely cause of this patient's recurrent angina symptoms?

A) Aneurysm of the right coronary artery


B) Dissection of the right coronary artery
u C) Neointima formation in the right coronary stent
U D) Thrombosis of the right coronary stent
E) Vasospasm in the stented right coronary artery

C. Page 304/330.
Not the worst question but definitely intimidating due to lack of experience with such a case.
The patient had RCA stenosis that was dealt with and she was fine for 3 months.
Something must have been going on in those 3 months because then she started a 2 month period of angina with exertion.
Its like her RCA stenosis came back again! And only choice C fits that explanation.
Choices A and B fail because she is fine when at rest. Same for choices D and E.
Restenosis occurs from gradual narrowing of the stent lumen due to neointimal proliferation.
This is why patients are put on a regiment of antiplatelet therapy after stent placement.

, ~ ~ p , ,.,,.
Previous Next Lab Values Calculator Review Help Pause
hi Comprehensive Basic Science Self-Assessment - Google Chrome x
i starttest.com/ITDVersions/19.3.0.1/ITDStart.aspx?SVC=3f50c3c0-1874-400e-af3a-5036e79dec9f

Exam Section 1: Item 46 of 50 National Board of Medical Examiners Time Remaining:


• Mark Comprehensive Basic Science Self-Assessment 4 hr 51 min 24 sec

46. A 63-year-old woman develops flank pain, tachycardia , and hypotension while receiving a blood transfusion in the intensive care unit. Her urine is wine-colored. Which of the
following is the most likely cause of these findings?

CJ A) ABO incompatibility
Page 114 FA2019.
B) Bacterial contamination of the transfused blood
Quesiton 37 was the one about transfusion related acute lung injury.
C) Extravascular hemolysis
This patient here has the acute hemolytic transfusion reaction due to ABO incompatibility.
U D) Immediate (type I) hypersensitivity reaction This is a type 2 HSR (antibodies and antigens) not a type 1 (allergic/anaphylactic).
E) Rh incompatibility

, ~ ~ p , ,.,,.
Previous Next Lab Values Calculator Review Help Pause
hi Comprehensive Basic Science Self-Assessment - Google Chrome x
i starttest.com/ITDVersions/19.3.0.1/ITDStart.aspx?SVC=3f50c3c0-1874-400e-af3a-5036e79dec9f

Exam Section 1: Item 47 of 50 National Board of Medical Examiners Time Remaining:


• Mark Comprehensive Basic Science Self-Assessment 4 hr 51 min 17 sec

47. A 55-year-old woman comes to the physician for a routine health maintenance examination. She has never smoked cigarettes. Physical examination shows no abnormalities. A
chest x-ray shows a 2-cm left pulmonary nodule (coin lesion). Microscopic examination of the excised lesion shows nests and cords of regular cells with uniform round nuclei and
rare mitoses. Focally, the cells are arranged in trabeculae, and scattered rosettes are seen. Electron microscopy shows dense-core, membrane-bound granules in the cytoplasms of
these cells. These cells are most likely to closely resemble which of the following types of normal respiratory tract cells?

A) Alveolar capillary endothelial cell


B) Alveolar macrophage
u C) Chondrocyte
U D) Ciliated columnar epithelial cell
E) Goblet cell
0 F) Neuroendocrine cell
u G) Squamous epithelial cell
H) Type I pneumocyte

F. Page 689/767 FA2019.


This patient never smoked and is a female, so that fits the profile for adenocarcinoma.
The incidental finding of her coin lesion (rather than her presenting because of unexplained weight loss and cough) fits the profile for a hamartoma.

The microscopic examination is diagnostic. The rosettes are almost always indicative of a neuroendocrine turmor.
They are most likely referring to a bronchial carcinoid tumor. The question did not want a precise diagnosis anyway so its not 100% clear.

, ~ ~ p , ,.,,.
Previous Next Lab Values Calculator Review Help Pause
hi Comprehensive Basic Science Self-Assessment - Google Chrome x
i starttest.com/ITDVersions/19.3.0.1/ITDStart.aspx?SVC=3f50c3c0-1874-400e-af3a-5036e79dec9f

Exam Section 1: Item 48 of 50 National Board of Medical Examiners Time Remaining:


• Mark Comprehensive Basic Science Self-Assessment 4 hr 51 min 11 sec

48. A 40-year-old man who recently immigrated to the USA from Kenya is brought to the emergency department because of a 3-week
history of diarrhea. He appears disoriented and cachectic. He recalls zero of three objects after 5 minutes. A photograph of the head,
neck, and upper chest is shown. Similar findings are seen over the dorsal aspects of the hands. The diagnosis of a vitamin deficiency
is made. The deficient vitamin in this patient is part of a cofactor that contains which of the following components?

A) Adenine
U B) Alanine
C) Apolipoprotein
U D) Arachidonic acid
E) Aspartate

A. Page 67.
This is clearly an extreme presentation of the "Casal necklace" rash that occurs in the C3/C4 dermatome.
This patient has vitamin B3 deficiency. Niacin is a part of NAD. The A is adenine.

, ~ ~ p , ,.,,.
Previous Next Lab Values Calculator Review Help Pause
hi Comprehensive Basic Science Self-Assessment - Google Chrome x
i starttest.com/ITDVersions/19.3.0.1/ITDStart.aspx?SVC=3f50c3c0-1874-400e-af3a-5036e79dec9f

Exam Section 1: Item 49 of 50 National Board of Medical Examiners Time Remaining:


• Mark Comprehensive Basic Science Self-Assessment 4 hr 51 min 4 sec

49. A 64-year-old man undergoes surgical repair of an abdominal aortic aneurysm. During the repair, the left testicular artery is ligated. Anastomotic supply from which of the following
arteries will maintain adequate arterial supply to the left testis in this patient?

CJ A) Artery of the ductus deferens


B) Inferior vesical artery
C) Obturator artery
U D) Posterior scrotal artery
E) Superficial circumflex iliac artery

A. Page 610/686.
This artery was previously tested in some NBME as the
deferential artery.
You can answer this question by simply thinking of what
other artery would be able to supply the testis.
The other arteries labelled do not reach the testis.

, ~ ~ p , ,.,,.
Previous Next Lab Values Calculator Review Help Pause
hi Comprehensive Basic Science Self-Assessment - Google Chrome x
i starttest.com/ITDVersions/19.3.0.1/ITDStart.aspx?SVC=3f50c3c0-1874-400e-af3a-5036e79dec9f

Exam Section 1: Item 50 of 50 National Board of Medical Examiners Time Remaining:


• Mark Comprehensive Basic Science Self-Assessment 4 hr 50 min 57 sec

50. A full-term male newborn has lethargy, poor feeding , and vomiting 48 hours after delivery. He appears flaccid. Serum studies show increased concentrations of ammonia and orotic
acid. Which of the following labeled steps in the metabolic pathway is the most likely cause of these findings?

~
H 4 + 2ATP 2ADP+ P i
N~
_ \ ./ , Carbamoyl- - -
_,l phosphate ~----..._,

C02 \:;:>
N-Acetylglutamate
Ornithine Citrulline
CoA-SHl(!l
Acetyl CoA + glutamate

Mitochondrial inner membrane

Mitochondrial outer membrane

Ornithine Citrulline
C. Page 82 and 83 FA2019.
This is a straightforward question on OTC deficiency.
High orotate and hyperammonemia with a low BUN. AMP + PPj
The excess carbamoyl phosphte upregulates
pyrimidine synthesis, forming excess orotate. Arginine Argininosucc in ate

Fumarate

0 A) B) u C) D) E)

, ~ ~ p , ,.,,.
Previous Next Lab Values Calculator Review Help Pause
hf Comprehensive Basic Science Self-Assessment - Google Chrome x
i starttest.com/ITDVersions/19.3.0.1/ITDStart.aspx?SVC=54cd3e5f-e4bf-49d0-8371-ba261251f2a1

Exam Section 2: Item 1 of 50 National Board of Medical Examiners Time Remaining:


• Mark Comprehensive Basic Science Self-Assessment 4 hr 59 min 50 sec

1. An 11-year-old boy is brought to the emergency department by ambulance with his mother 30 minutes after she was unable to awaken him from a nap. The mother states that he was
recently diagnosed with type 1 diabetes mellitus and she has been having difficulty adjusting his insulin regimen. He is unconscious, and his breath smells of acetone. He is
responsive only to sharp pain. A rapid test shows a serum glucose concentration greater than 500 mg/dl. Which of the following best explains this patient's altered state of
consciousness?

A) Altered affinity of oxygen from hemoglobin A 1c


B) Diminished transport of glucose across the blood-brain barrier
u C) Excess free fatty acids in the cerebrospinal fluid
U D) Inability of neurons to perform glycolysis
E) Intracellular and extracellular dehydration

E. Page 346/379.
This patient has diabetic ketoacidosis, which is most commonly seen in DM1 patients that missed their insulin.
The body starts breaking down fats to increase ketogenesis, which is great for them.
However, the extremely high glucose levels cause severe dehydration that can lead to delirium.
The osmotic diuresis needs to be managed with IV fluids along with the IV insulin and K+.

~ ~ p, ,.,,.
Next Lab Values Calculator Review Help Pause
hi Comprehensive Basic Science Self-Assessment - Google Chrome x
i starttest.com/ITDVersions/19.3.0.1/ITDStart.aspx?SVC=54cd3e5f-e4bf-49d0-8371-ba261251f2a1

Exam Section 2: Item 2 of 50 National Board of Medical Examiners Time Remaining:


• Mark Comprehensive Basic Science Self-Assessment 4 hr 59 min 35 sec

2. A researcher hypothesizes that exposure to more than 50 µg/L of arsenic in drinking water is associated with an increased risk for development of a common cancer compared with
persons whose drinking water contains less than 5 µg/L of arsenic. Which of the following is the most time-efficient experimental design to investigate this hypothesis?

CJ A) Case-control study A. Page 256/266.


B) Case series study Classic NBME trash. The question asks for a time-efficient, experimental design.
C) Cohort study While case-control studies are time-efficient, they are observational studies.
U D) Open-label, dose-ranging study You would need a case (people drinking high arsenic) and control (people with no arsenic).
E) Randomized, double-blind, controlled trial
Experimental studies are usually randomized controlled trials, where E would be the best answer.

, ~ ~ p , ,.,,.
Previous Next Lab Values Calculator Review Help Pause
hi Comprehensive Basic Science Self-Assessment - Google Chrome x
i starttest.com/ITDVersions/19.3.0.1/ITDStart.aspx?SVC=54cd3e5f-e4bf-49d0-8371-ba261251f2a1

Exam Section 2: Item 3 of 50 National Board of Medical Examiners Time Remaining:


• Mark Comprehensive Basic Science Self-Assessment 4 hr 59 min 26 sec

3. A 43-year-old woman with a 2-year history of labile hypertension comes to the physician for a follow-up examination. Her blood pressure has ranged from 123/84 mm Hg to 165/ 112
mm Hg since being diagnosed. One day after propranolol therapy is started, she develops a severe headache and her blood pressure is 214/132 mm Hg. Serum and urine
catecholamine concentrations are four times the upper limit of the reference ranges. A CT scan shows a 2-cm nodule in the right adrenal gland. Which of the following is most likely
responsible for the increased blood pressure in this patient after propranolol therapy was started?

A) Anxiety reaction
B) Impaired degradation of catecholamines
u C) Increased adrenocorticotropic hormone release
U D) Infarction of the adrenal tumor
E) Stimulation of aldosterone production
0 F) Unopposed a-adrenergic tone

F. Page 335/366.
This patient must have a pheochromocytoma, which is treated with an irreversible alhpa blocker (to stop vasoconstriction), followed by a
beta blocker (to decrease HR but these also cause vasoconstriction so they are given a week after the alpha blocker is initiated).

, ~ ~ p , ,.,,.
Previous Next Lab Values Calculator Review Help Pause
hi Comprehensive Basic Science Self-Assessment - Google Chrome x
i starttest.com/ITDVersions/19.3.0.1/ITDStart.aspx?SVC=54cd3e5f-e4bf-49d0-8371-ba261251f2a1

Exam Section 2: Item 4 of 50 National Board of Medical Examiners Time Remaining:


• Mark Comprehensive Basic Science Self-Assessment 4 hr 59 min 17 sec

4. A 4-year-old girl is brought to the physician because of a 6-month history of foul-smelling stools and failure to gain weight appropriately. She is at the 3rd percentile for height and
weight. Abdominal examination shows distention. Stool analysis shows an increased fat concentration. Serum concentrations of anti-endomysial and anti-tissue transglutaminase
antibodies are increased. Which of the following findings in the gastrointestinal tract is the most likely cause of the disorder in this patient?

A) Eosinophilic infiltration of the mucosa of the small intestine


B) Erosions of epithelial cells in the small intestine
C) Lactase deficiency
D) Periodic acid-Schiff (PAS)-positive granules in macrophages in the lamina propria of the small intestine
E) Villous atrophy

E. Page 375/416.
This patient's serum antibodies are diagnostic of Celiac disease.
The answer choice was very easy.

A -- Probably some parasitic infection.


B -- Erosions are irrelevant. Small intestine erosion is related to IBD.
C -- Irrelevant.
D -- Whipple disease.

, ~ ~ p , ,.,,.
Previous Next Lab Values Calculator Review Help Pause
hi Comprehensive Basic Science Self-Assessment - Google Chrome x
i starttest.com/ITDVersions/19.3.0.1/ITDStart.aspx?SVC=54cd3e5f-e4bf-49d0-8371-ba261251f2a1

Exam Section 2: Item 5 of 50 National Board of Medical Examiners Time Remaining:


• Mark Comprehensive Basic Science Self-Assessment 4 hr 59 min 10 sec

5. A 26-year-old man comes to the physician for a follow-up examination. One month ago, his total serum cholesterol concentration was 325 mg/dl, and the physician prescribed a 3-
hydroxy-3-methylglutaryl (HMG) CoA reductase inhibitor. At the time of the present examination, his total serum cholesterol concentration is 235 mg/dl. Which of the following is the
most likely cause of the decrease?

A) Decreased absorption of dietary cholesterol


B) Decreased conversion of chylomicrons to VLDL E. Page 315/344.
C) Increased concentration of HDL in plasma Statins work by indirectly causing an increase in hepatic LDL receptors.
D) Increased intracellular esterification of cholesterol Same question as question 43 block 1.
E) Increased number of LDL receptors on hepatocytes
F) Increased synthesis of apolipoprotein B-100

, ~ ~ p , ,.,,.
Previous Next Lab Values Calculator Review Help Pause
hi Comprehensive Basic Science Self-Assessment - Google Chrome x
i starttest.com/ITDVersions/19.3.0.1/ITDStart.aspx?SVC=54cd3e5f-e4bf-49d0-8371-ba261251f2a1

Exam Section 2: Item 6 of 50 National Board of Medical Examiners Time Remaining:


• Mark Comprehensive Basic Science Self-Assessment 4 hr 59 min 1 sec

6. A 28-year-old man with AIDS has fever, headache, and lethargy. Examination shows papilledema and nuchal rigidity. A mucicarmine-
stained smear of his cerebrospinal fluid is shown. Which of the following is the most likely primary site of infection with this organism?

A) Eyes
B) Gastrointestinal tract
C) Lungs
CJ D) Middle ear C. 153/155.
E) Skin This was the best answer to pick given the image and patient's AIDS history.
This image shows budding but it is not broad-based, its narrow budding.
Crypto is stained with india ink or mucicarcmine and can also be found with latex agglutination.
This is a heavily encapsulated yeast that causes meningitis, encephalitis, and cryptococcosis.
Patients inhale this so its found in the bronchoalveolar lavage.

, ~ ~ p , ,.,,.
Previous Next Lab Values Calculator Review Help Pause
hi Comprehensive Basic Science Self-Assessment - Google Chrome x
i starttest.com/ITDVersions/19.3.0.1/ITDStart.aspx?SVC=54cd3e5f-e4bf-49d0-8371-ba261251f2a1

Exam Section 2: Item 7 of 50 National Board of Medical Examiners Time Remaining:


• Mark Comprehensive Basic Science Self-Assessment 4 hr 58 min 51 sec

7. A 68-year-old woman with acute myelogenous leukemia begins treatment with standard induction chemotherapy. After completion of the regimen , her leukocyte count is 1200/mm 3
with an absolute neutrophil count of 475/mm 3. Which of the following is the most appropriate pharmacotherapy?

CJ A) Bcr-Abl tyrosine kinase inhibitor


B) Erythropoietin
C) Granulocyte-macrophage colony-stimulating factor
U D) lnterleukin-2 (IL-2)
E) Thrombopoietin
F) Transforming growth factor-13

C. Page 121 and page 424/469.


This patient's leukocyte count and neutrophil count are very low.
She simply needs something that will help her make new WBCs.
AML is treated with vitamin A and arsenic (if its the APL variant with t(15;17).
The problem in AML is the high amounts of myeloblasts not becoming mature, so vitamin A and arsenic induce differentiation.

, ~ ~ p , ,.,,.
Previous Next Lab Values Calculator Review Help Pause
hi Comprehensive Basic Science Self-Assessment - Google Chrome x
i starttest.com/ITDVersions/19.3.0.1/ITDStart.aspx?SVC=54cd3e5f-e4bf-49d0-8371-ba261251f2a1

Exam Section 2: Item 8 of 50 National Board of Medical Examiners Time Remaining:


• Mark Comprehensive Basic Science Self-Assessment 4 hr 58 min 43 sec

8. A 70-year-old man is brought to the emergency department because of a 1-week history of progressive shortness of breath with mild exertion, cough, fatigue, and weakness. The
shortness of breath frequently awakens him from sleep and is partly relieved by sitting or standing. He has a 20-year history of hypertension treated with diuretics and ACE inhibitors;
however, he does not always take his medications. His pulse is 85/min, respirations are 20/min, and blood pressure is 180/110 mm Hg. Crackles are heard on auscultation. A chest x-
ray shows cardiomegaly. Echocardiography shows decreased left ventricular systolic function. Which of the following patterns best characterizes this patient's cardiovascular
variables at this time?

Left Ventricular
Stroke End-Diastolic Cardiac
Volume Volume Output
0 A) i i i
B) i i l
C) i l i
U D) i l l
E) l i i
U F) l i l
G) l l i
H) l l l

F. Page 306/332.
This patient clearly has left heart failure. The fact that the x ray shows cardiomegaly and auscultation reveals crackles, it is decompensated.
He has orthopnea, paroxysmal nocturnal dyspnea, and the pulmonary edema.
Since the heart is not working well, obviously stroke volume and cardiac output will be low.
The blood remains in theheart so his EDV will be high.

, ~ ~ p , ,.,,.
Previous Next Lab Values Calculator Review Help Pause
hi Comprehensive Basic Science Self-Assessment - Google Chrome x
i starttest.com/ITDVersions/19.3.0.1/ITDStart.aspx?SVC=54cd3e5f-e4bf-49d0-8371-ba261251f2a1

Exam Section 2: Item 9 of 50 National Board of Medical Examiners Time Remaining:


• Mark Comprehensive Basic Science Self-Assessment 4 hr 58 min 35 sec

9. A 52-year-old woman comes to the physician because of severe pain in her midback for 2 weeks. She has a history of left breast cancer treated with mastectomy and chemotherapy.
Physical examination shows tenderness to palpation over the thoracic spine. An MRI of the back shows metastases in thoracic vertebral bodies. Which of the following veins is the
most likely path for tumor cells in the breast to obtain access to the vertebral bodies in this patient?

A) Axillary
B) lntercostal
C) Internal thoracic
D) Lateral thoracic
E) Subclavian

B. Pg 636/716.
NBME has asked about this before too.
I explained that one using this same image.
The anatomical basis of the spread of breast cancer is high-yield.
First, recall that bone mets can be lytic or blastic (2 breasts, 2 lesion types).

The intercostal veins drain into the azygos, which drains some into the SVC
but a lot of it goes to Batson's vertebral venous plexus as well, causing the mets.
C - Goes to brachiocephacli v.
D- Goes to A, which goes to E, ultimately coming to brachiocephalic also.

Regardless, axillary lymph mets is the most important prognostic factor.


This is by the lateral mammary-> lat thoracic -> axillary veins.
, ~ ~ p , ,.,,.
Previous Next Lab Values Calculator Review Help Pause
hi Comprehensive Basic Science Self-Assessment - Google Chrome x
i starttest.com/ITDVersions/19.3.0.1/ITDStart.aspx?SVC=54cd3e5f-e4bf-49d0-8371-ba261251f2a1

Exam Section 2: Item 10 of 50 National Board of Medical Examiners Time Remaining:


• Mark Comprehensive Basic Science Self-Assessment 4 hr 58 min 28 sec

10. An 80-year-old woman with poorly controlled type 2 diabetes mellitus and hypertension comes to the physician for a follow-up examination. She also has a 5-year history of chronic
kidney disease with increasing proteinuria and a recent creatinine clearance of 30 ml/min. Her blood pressure is 165/95 mm Hg. Physical examination shows edema over the lower
extremities. Pulmonary and cardiac examinations show no abnormalities. Compared with a healthy individual, which of the following sets of laboratory findings is most likely in this
patient?

Serum
Hematocrit Parathyroid Hormone Serum Calcitriol
A) Increased normal increased
B) Increased normal decreased
C) Normal increased decreased
D) Normal decreased normal
E) Decreased increased decreased
U F) Decreased decreased decreased

E. Page 590/663.
The patient has CKD.
Without kidney function, you can assume the hormones that work through the kidney will not be able to carry out their tasks.
For this question, EPO will not work, so hematocrit will be low.
PTH will not work so your body will make more PTH in attempts to get a result. However, PTH function will fail due to the CKD.
The renal 1 alpha hydroxylase will not function, so 1,25 dihydroxy vitamin D (calcitriol) will be low.

, ~ ~ p , ,.,,.
Previous Next Lab Values Calculator Review Help Pause
hi Comprehensive Basic Science Self-Assessment - Google Chrome x
i starttest.com/ITDVersions/19.3.0.1/ITDStart.aspx?SVC=54cd3e5f-e4bf-49d0-8371-ba261251f2a1

Exam Section 2: Item 11 of 50 National Board of Medical Examiners Time Remaining:


• Mark Comprehensive Basic Science Self-Assessment 4 hr 58 min 20 sec

11. An experimental study is conducted to examine the function of the pro-opiomelanocortin gene. The gene is found to encode an mRNA from which protein products are generated.
This protein serves as the precursor of adrenocorticotropic hormone and f3-lipotropin. Which of the following processes is necessary in order to generate these hormones?

CJ A) Alternative splicing
B) Gene rearrangement
C) Methylation of the gene
U D) Post-transcriptional modification
E) Post-translational modification

E. Page 41 and page 323/352.


This is just a question about POMC, a prohormone peptide chain, yields the opio beta-endorphin, melano MSH, and cortin ACTH.
This is all a result of post-translational modification.

This is not an example of alternative splicing, where one gene can yield different mRNA based on how that mRNA's exons are reorganized.
This is not due to post-transcriptional modification, which is practically what alternative splicing is.

, ~ ~ p , ,.,,.
Previous Next Lab Values Calculator Review Help Pause
hi Comprehensive Basic Science Self-Assessment - Google Chrome x
i starttest.com/ITDVersions/19.3.0.1/ITDStart.aspx?SVC=54cd3e5f-e4bf-49d0-8371-ba261251f2a1

Exam Section 2: Item 12 of 50 National Board of Medical Examiners Time Remaining:


• Mark Comprehensive Basic Science Self-Assessment 4 hr 58 min 12 sec

12. A 24-year-old man receives an injection of histamine in the brachia! artery. Which of the following changes is most likely to occur in his forearm in response to the histamine?

A) Decreased capillary filtration rate


E. Page 332/362.
U B) Decreased interstitial hydrostatic pressure Histamine is a vasodilator that causes vasodilation through the H1 receptor via IP3 and Gq.
U C) Decreased interstitial oncotic pressure It is released from basophils and mast cells and is a major part of inflammation.
D) Increased arteriole resistance The vasodilation and increased vascular permeability are key characteristics that all choices A-D fail.
E) Increased lymph flow

, ~ ~ p , ,.,,.
Previous Next Lab Values Calculator Review Help Pause
hi Comprehensive Basic Science Self-Assessment - Google Chrome x
i starttest.com/ITDVersions/19.3.0.1/ITDStart.aspx?SVC=54cd3e5f-e4bf-49d0-8371-ba261251f2a1

Exam Section 2: Item 13 of 50 National Board of Medical Examiners Time Remaining:


• Mark Comprehensive Basic Science Self-Assessment 4 hr 58 min 5 sec

13. A 24-year-old woman at 28 weeks' gestation is brought to the emergency department because of a 3-hour history of shortness of breath. She has asthma, but she currently takes no
medications. Her pulse is 100/min, respirations are 32/min, and blood pressure is 120/83 mm Hg. Physical examination shows the use of accessory muscles of respiration. Diffuse
inspiratory and expiratory wheezes are heard. An inhaled ~ 2-adrenergic agonist is administered. Which of the following findings is most likely in this patient after this therapy?

A) Bradycardia
E. Page 672/772.
B) Diaphoresis
It is straight from the book.
C) Dry mouth
No reason to pick A, beta 2 might cause vasodilation but not bradycardia.
D) Pallor Diaphoresis and dry mouth are muscarinic effects.
E) Tremor
Think about the fact that beta blockers help treat essential tremor and can mask the tremor of hypoglycemia.
So a beta agonist must be capable of causing tremors.

, ~ ~ p , ,.,,.
Previous Next Lab Values Calculator Review Help Pause
hi Comprehensive Basic Science Self-Assessment - Google Chrome x
i starttest.com/ITDVersions/19.3.0.1/ITDStart.aspx?SVC=54cd3e5f-e4bf-49d0-8371-ba261251f2a1

Exam Section 2: Item 14 of 50 National Board of Medical Examiners Time Remaining:


• Mark Comprehensive Basic Science Self-Assessment 4 hr 57 min 58 sec

14. A 4-year-old boy is brought to the emergency department by his mother 6 hours after she noticed that his urine was red. He is otherwise feeling well. Fifteen days ago, the patient
had a sore throat, fever, and cough. His mother thought he had the flu and treated him symptomatically with rest and analgesics, and his status improved until now. His temperature
is 38.5°C (101.3°F), pulse is 110/ min, respirations are 22/min, and blood pressure is 100/50 mm Hg. Physical examination shows normal breath and cardiac sounds, no
organomegaly, and 1 + lower extremity edema, bilaterally. Laboratory studies show:
Serum
Urea nitrogen 40 mg/dl
Creatinine 2 mg/dl
Albumin 3.6 g/dl
Urine
Color red-brown
Blood 3+
Protein 2+
Ketones negative
RBC 30-50/hpf
RBC casts few

Which of the following is the most likely diagnosis?


D. Page 582/653 and page 591/664.
A) Membranous nephropathy
This 4yo kid most likely had a Strep infection and now has PSGN.
B) Minimal change disease The only GN answer choice is D.
C) Papillary necrosis Choices A and B are nephrotic so they are wrong.
D) Proliferative glomerulonephritis Choice E is basically acute interstitial nephritis and choice C is a papillary problem (not glomerular).
E) Tubulointerstitial nephritis Those two choices are related to various medications or conditions that are irrelevant here.

, ~ ~ p , ,.,,.
Previous Next Lab Values Calculator Review Help Pause
hi Comprehensive Basic Science Self-Assessment - Google Chrome x
i starttest.com/ITDVersions/19.3.0.1/ITDStart.aspx?SVC=54cd3e5f-e4bf-49d0-8371-ba261251f2a1

Exam Section 2: Item 15 of 50 National Board of Medical Examiners Time Remaining:


• Mark Comprehensive Basic Science Self-Assessment 4 hr 57 min 52 sec

15. A 1616-g (3-lb 9-oz) male newborn is delivered to a 16-year-old primigravid patient at 28 weeks' gestation. He develops severe respiratory distress within 1 hour and is treated with
intubation and 100% inspired oxygen with continuous positive end-expiratory pressure. One month later, a chest x-ray shows persistently increased densities in all lung fields. He
develops pneumonia and dies. At autopsy, the lungs are firm and poorly aerated. Microscopic examination of lung tissue shows prominent interstitial fibrosis surrounding irregular,
dilated airspaces with bronchiolar metaplasia. Which of the following is the most likely diagnosis?

A) Bronchial asthma D. Page 647/728.


B) Bron ch iectasis This baby was born early (28 weeks) and has developed NRDS.
u C) Bronchiolitis obliterans After being given therapeutic 100% O2, the RIB problems can come up.
U D) Bronchopulmonary dysplasia Straightforward question about the B, bronchopulmonary dysplasia.
E) Chronic bronchitis
0 F) Cystic adenomatoid malformation Bronchial asthma is irrelevant.
Bronchiectasis and chronic bronchitis are COPD related.
Bronchiolitis obliterans is the result of chronic lung transplant rejection.

Choice F is random. Its a malformation where a lobe forms a cyst that does not function as normal lung tissue.
https://en.wikipedia.org/wiki/Congenital_pulmonary_airway_malformation

Both images show cystic


changes in the lung.

, ~ ~ p , ,.,,.
Previous Next Lab Values Calculator Review Help Pause
hi Comprehensive Basic Science Self-Assessment - Google Chrome x
i starttest.com/ITDVersions/19.3.0.1/ITDStart.aspx?SVC=54cd3e5f-e4bf-49d0-8371-ba261251f2a1

Exam Section 2: Item 16 of 50 National Board of Medical Examiners Time Remaining:


• Mark Comprehensive Basic Science Self-Assessment 4 hr 57 min 45 sec

16. The 35-year-old woman indicated by the arrow has a family history of retinitis pigmentosa. Which of the following modes of inheritance can be
eliminated based on this pedigree?

A) Autosomal dominant
B) Autosomal recessive
C) Multifactorial
CJ D) X-linked recessive

D. Page 59 FA2019.
Men pass their Y chromosome to sons, so there definitely is not X-linked inheritance here since all men are having it.

, ~ ~ p , ,.,,.
Previous Next Lab Values Calculator Review Help Pause
hi Comprehensive Basic Science Self-Assessment - Google Chrome x
i starttest.com/ITDVersions/19.3.0.1/ITDStart.aspx?SVC=54cd3e5f-e4bf-49d0-8371-ba261251f2a1

Exam Section 2: Item 17 of 50 National Board of Medical Examiners Time Remaining:


• Mark Comprehensive Basic Science Self-Assessment 4 hr 57 min 37 sec

17. A 35-year-old man comes to the physician because of a 1-week history of severe daily headaches of sudden onset. The pain is often located around his left eye and is associated
with excessive tearing and redness of the eye. The headaches last from 30 minutes to 2 hours. He has had similar episodes during the past 7 years, often with periods of up to 1
year during which no headaches occur. Vital signs are normal. Funduscopic examination shows no abnormalities. Neurologic examination shows no focal findings. Which of the
following is the most likely diagnosis?

A) Acute meningitis
B) Cluster headache
u C) Idiopathic intracranial hypertension B. Page 506/566.
U D) Migraine This description of pain around one eye is classic for a cluster headache.
E) Subarachnoid hemorrhage The lacrimation this patient has is also key in the diagnosis.
0 F) Temporal arteritis
u G) Tension-type headache

, ~ ~ p , ,.,,.
Previous Next Lab Values Calculator Review Help Pause
hi Comprehensive Basic Science Self-Assessment - Google Chrome x
i starttest.com/ITDVersions/19.3.0.1/ITDStart.aspx?SVC=54cd3e5f-e4bf-49d0-8371-ba261251f2a1

Exam Section 2: Item 18 of 50 National Board of Medical Examiners Time Remaining:


• Mark Comprehensive Basic Science Self-Assessment 4 hr 57 min 31 sec

18. A 3-year-old boy is brought to the emergency department because of epigastric pain and bile-stained vomitus. The child had been well prior to this episode and has reached all
milestones normally. Which of the following developmental abnormalities is most likely to be the cause of this patient's problem?

CJ A) Annular pancreas
B) Esophageal atresia A. Page 354/388.
The only way to get bilious vomiting is if there is an obstruction past the 2nd part of the duodenum.
C) Extrahepatic biliary atresia
Careful. Annular pancreas does not always present with bilious vomiting, so it is not a critical characteristic.
U D) Hypertrophic pyloric stenosis

, ~ ~ p , ,.,,.
Previous Next Lab Values Calculator Review Help Pause
hi Comprehensive Basic Science Self-Assessment - Google Chrome x
i starttest.com/ITDVersions/19.3.0.1/ITDStart.aspx?SVC=54cd3e5f-e4bf-49d0-8371-ba261251f2a1

Exam Section 2: Item 19 of 50 National Board of Medical Examiners Time Remaining:


• Mark Comprehensive Basic Science Self-Assessment 4 hr 57 min 24 sec

19. A 7-year-old boy has metabolic acidosis and persistent phosphaturia. This patient most likely has a primary defect of which of the following labeled sites in the drawing of the
nephron shown?

B. Page 574/642.
The PCT reabsorbs most things including phosphate.
This child probably has Fanconi.
Very important to spend some time and master these.
Of the various renal tubular defects, only fanconi
causes metabolic acidosis.

This was not a question about the RTAs on page 581/651.


Those cause acidosis of the urine but not metabolic acidosis.

U A) B) U C) D) E)

, ~ ~ p , ,.,,.
Previous Next Lab Values Calculator Review Help Pause
hi Comprehensive Basic Science Self-Assessment - Google Chrome x
i starttest.com/ITDVersions/19.3.0.1/ITDStart.aspx?SVC=54cd3e5f-e4bf-49d0-8371-ba261251f2a1

Exam Section 2: Item 20 of 50 National Board of Medical Examiners Time Remaining:


• Mark Comprehensive Basic Science Self-Assessment 4 hr 57 min 16 sec

20. A 65-year-old woman comes to the physician because of blood in her urine with no pain for 5 days. Physical examination
shows no abnormalities. Cystoscopy depicts multiple lesions as shown in the photomicrograph. Which of the following is
the strongest predisposing risk factor for the development of this lesion?

A) Alcoholism
B) Cigarette smoking
U C) Human papillomavirus infection
0 D) Schistosomiasis
U E) Vinyl chloride exposure

B. Page 226/236.
If it is ever the case that smoking might cause the problem, then it is most likely right.
The patient most likely has transitional cell carcinoma (urothelial carcinoma).

Schistosoma haematobium can lead to squamous cell carcinoma of the bladder and
would also present with painless hematuria. But there would be fever.
The image shows transitional cell carcinoma.

, ~ ~ p , ,.,,.
Previous Next Lab Values Calculator Review Help Pause
hi Comprehensive Basic Science Self-Assessment - Google Chrome x
i starttest.com/ITDVersions/19.3.0.1/ITDStart.aspx?SVC=54cd3e5f-e4bf-49d0-8371-ba261251f2a1

Exam Section 2: Item 21 of 50 National Board of Medical Examiners Time Remaining:


• Mark Comprehensive Basic Science Self-Assessment 4 hr 57 min 11 sec

21. After completing 6 courses of chemotherapy for cancer of the breast, a 45-year-old woman has shortness of breath, dyspnea on exertion, and orthopnea. Basilar crackles are heard
bilaterally, and there is an S 3 gallop. X-rays of the chest show an enlarged heart, bilateral interstitial and alveolar edema, and bilateral pleural effusions. Which of the following is the
chemotherapeutic drug most likely to have caused these findings?

A) Bleomycin
B) Cytosine arabinoside
C. Page 431/477.
Memorizing these takes a bit of time and chemo-tox man but its fairly easy.
C) Doxorubicin
This patient has symptoms of dilated cardiomyopathy caused by the cardiotoxic doxorubicin.
D) Methotrexate
Shouldve given her concomitant dexrazoxane.
E) Vincristine

, ~ ~ p , ,.,,.
Previous Next Lab Values Calculator Review Help Pause
hi Comprehensive Basic Science Self-Assessment - Google Chrome x
i starttest.com/ITDVersions/19.3.0.1/ITDStart.aspx?SVC=54cd3e5f-e4bf-49d0-8371-ba261251f2a1

Exam Section 2: Item 22 of 50 National Board of Medical Examiners Time Remaining:


• Mark Comprehensive Basic Science Self-Assessment 4 hr 57 min 5 sec

22. A 10-year-old girl with a lifelong history of intermittent dyspnea is brought to the physician for a follow-up examination. During an episode of dyspnea , she turns blue. The episode
resolves after she squats. A systolic murmur was heard at birth. Today, a grade 4/6 systolic murmur is heard at the pulmonic area, radiating widely to the back and left side. Cardiac
catheterization shows:
Location Pressure (mm Hg) 0 2 Saturation
Aorta 120/80 82%
Vena cava 610 70%
Pulmonary artery 20/8 70%
Right atrium 4 70%
Left atrium 6 95%
Right ventricle 120/6 70%
Left ventricle 120/6 82%

Which of the following is most likely in this patient? D. Page 297/321 FA2019.
The patient clearly has a VSD. Remember my "VAPE later" mnemonic.
U A) Diastolic murmur Squatting increases pressures in the right heart, sending blood back into the LV and allowing the
B) Enlarged left ventricle on chest x-ray blood to then go into the aorta, fixing the cyanosis.
0 C) Hypervascular lung fields on chest x-ray A VSD would cause a systolic murmur whenever the LV contracts, cuz itll send blood into the RV.
D) Palpable right ventricular lift This is given in the question.
E) Pulsating aortic mass on CT scan of the chest
Lung problems are usually a problem related to the left heart.
Pulsating aortic mass would be maybe related to aortic issues not a VSD.

, ~ ~ p , ,.,,.
Previous Next Lab Values Calculator Review Help Pause
hi Comprehensive Basic Science Self-Assessment - Google Chrome x
i starttest.com/ITDVersions/19.3.0.1/ITDStart.aspx?SVC=54cd3e5f-e4bf-49d0-8371-ba261251f2a1

Exam Section 2: Item 23 of 50 National Board of Medical Examiners Time Remaining:


• Mark Comprehensive Basic Science Self-Assessment 4 hr 56 min 59 sec

23. A 10-year-old girl is scheduled to undergo fixation of a right femoral fracture. A neuromuscular blocking agent is administered prior to insertion of an endotracheal tube.
Subsequently, she develops severe muscle rigidity. Her temperature is 41 .1°C (106°F), pulse is 100/min, and blood pressure is unstable. Which of the following drugs most likely
caused this reaction?

A) Etomidate
B) Midazolam F. Page 537/599.
C) Nitric oxide Muscular blockade, leading to fever and muscle rigidity? Malignant hyperthermia.
D) Nitrous oxide Classic presentation. Treat with dantrolene.
E) Rocuronium
F) Succinylcholine

, ~ ~ p , ,.,,.
Previous Next Lab Values Calculator Review Help Pause
hi Comprehensive Basic Science Self-Assessment - Google Chrome x
i starttest.com/ITDVersions/19.3.0.1/ITDStart.aspx?SVC=54cd3e5f-e4bf-49d0-8371-ba261251f2a1

Exam Section 2: Item 24 of 50 National Board of Medical Examiners Time Remaining:


• Mark Comprehensive Basic Science Self-Assessment 4 hr 56 min 53 sec

*'-
24. The graph shows expiratory pressure-volume curves that compare a healthy adult with a patient who has a 9-month Normal
history of progressive shortness of breath. These findings are most consistent with which of the following?

A) Asthma
-
-
>-
100

TS 80
B) Bronchopneumonia co
a.
C) Chronic bronchitis co 60
(.)
CJ D) Diffuse pulmonary fibrosis O>
Patient
c: 40
E) Emphysema ~

D. Page 651/735.
-
co
._
0
20

This is math. The graph shows lung capacity / transpulmonary pressure, which is compliance. 10 20 30 40
Recall that compliance is volume / pressure. Transpu lmonary p ressure
(cm H20)
Choices A and B are irrelevant, and choices C and E go together (and have increased compliance).
Choice C is the only unique answer where compliance decreases.

, ~ ~ p , ,.,,.
Previous Next Lab Values Calculator Review Help Pause
hi Comprehensive Basic Science Self-Assessment - Google Chrome x
i starttest.com/ITDVersions/19.3.0.1/ITDStart.aspx?SVC=54cd3e5f-e4bf-49d0-8371-ba261251f2a1

Exam Section 2: Item 25 of 50 National Board of Medical Examiners Time Remaining:


• Mark Comprehensive Basic Science Self-Assessment 4 hr 56 min 47 sec

25. A thin 66-year-old man is brought to the emergency department because of confusion. Initial laboratory tests show severe hyponatremia. The syndrome of inappropriate ADH
(vasopressin) secretion is suspected. Which of the following findings best supports this diagnosis?

CJ A) Increased serum urea nitrogen and creatinine concentrations


B) Serum potassium concentration < 2.5 mEq/L
C) Urine osmolality > plasma osmolality
U D) Urine sodium concentration < 10 mEq/L
E) Urine specific gravity of 1.002

C. Page 344/376.
Very straightforward answer choice.
The patient has SIADH.
ADH causes anti-diuretic so you retain water.
This means your urine solute concentration (aka urine osmolality) will be higher than your plasma solute concentration.
The plasma solutes will be diluted in all the water the patient will retain.
The other answer choices all become plausible "maybe" choices if you dont understand SIADH and the terminology of osmolality very well.

, ~ ~ p , ,.,,.
Previous Next Lab Values Calculator Review Help Pause
hi Comprehensive Basic Science Self-Assessment - Google Chrome x
i starttest.com/ITDVersions/19.3.0.1/ITDStart.aspx?SVC=54cd3e5f-e4bf-49d0-8371-ba261251f2a1

Exam Section 2: Item 26 of 50 National Board of Medical Examiners Time Remaining:


• Mark Comprehensive Basic Science Self-Assessment 4 hr 56 min 41 sec

26. A 71-year-old man is brought to the emergency department because of the sudden onset of dysarthria, dysphagia, and dizziness. Examination shows markedly decreased pain
sensation on the right side of the face and complete loss of pain sensation on the left side of the body. The most likely cause is occlusion of which of the following labeled blood
vessels in the normal brain stem shown?

D. Page 502/562 FA2019.

https://www.youtube.com/watch?v=A8S3B9p1t_g

This is Wallenberg syndrome.


"Dont PICA (pick a) horse that cannot swallow."
The patient has dyshphagia and some sensory deficits.
The only label pointing at something posterior is D.
E

0 A) U B) .J C) D) U E)

, ~ ~ p , ,.,,.
Previous Next Lab Values Calculator Review Help Pause
hi Comprehensive Basic Science Self-Assessment - Google Chrome x
i starttest.com/ITDVersions/19.3.0.1/ITDStart.aspx?SVC=54cd3e5f-e4bf-49d0-8371-ba261251f2a1

Exam Section 2: Item 27 of 50 National Board of Medical Examiners Time Remaining:


• Mark Comprehensive Basic Science Self-Assessment 4 hr 56 min 35 sec

27. A 40-year-old man with AIDS develops severe diarrhea that is refractory to all standard oral therapies. His diarrhea resolves after intravenous administration of a stable analogue of
a naturally occurring hormone. The analogue administered most likely mimics which of the following hormones?

CJ A) Cortisol
C. Page 365/406.
B) Glucagon
Somatostatin has an analog known as octreotide.
C) Somatostatin The other hormones listed dont really have a commonly tested analog.
U D) Testosterone Somatostatin causes static stasis of all GIT activity and can treat acromegaly, carcinoid, and variceal bleeding.
E) Vitamin D

, ~ ~ p , ,.,,.
Previous Next Lab Values Calculator Review Help Pause
hi Comprehensive Basic Science Self-Assessment - Google Chrome x
i starttest.com/ITDVersions/19.3.0.1/ITDStart.aspx?SVC=54cd3e5f-e4bf-49d0-8371-ba261251f2a1

Exam Section 2: Item 28 of 50 National Board of Medical Examiners Time Remaining:


• Mark Comprehensive Basic Science Self-Assessment 4 hr 56 min 28 sec

28. A 25-year-old woman at 16 weeks' gestation comes to the physician for a routine examination. One year ago, she had a serum thyroxine (T 4 ) concentration of 8.2 µg/dl and a serum
thyroid-stimulating hormone (TSH) concentration of 2.1 µU/ml. At that time, she was not taking any medications and she was not pregnant. At this visit, serum T 4 concentration is
13.2 µg/dl and serum TSH concentration is 2.2 µU/ml. Which of the following is most likely responsible for the increased serum T 4 concentration in this patient?

A) Diffuse toxic goiter (Graves disease)


B) High T 4 content in prenatal vitamins
C) Increased fetal production of T 4
D) Increased hepatic production of T 4-binding globulin
E) Struma ovarii
F) TSH-producing pituitary tumor

D. Page 331/361.
This kind of presentation of abnormal T4 concentrations where a patient is presenting with labs or symptoms that seem like a thyroid
imbalance is special in cases of pregnancy, OCP use, hepatic falure, steroid use, or nephrotic syndrome.

This is basically a question of thyroid binding globulin, which is high in pregnancy and OCP use (estrogen increases TBG production.
The globulin is holding on to all the thyroid, creating a scenario of hypothyroidism so the body has more TSH formation but inactive T3 and T4.

The other answer choices are irrelevant.

, ~ ~ p , ,.,,.
Previous Next Lab Values Calculator Review Help Pause
hi Comprehensive Basic Science Self-Assessment - Google Chrome x
i starttest.com/ITDVersions/19.3.0.1/ITDStart.aspx?SVC=54cd3e5f-e4bf-49d0-8371-ba261251f2a1

Exam Section 2: Item 29 of 50 National Board of Medical Examiners Time Remaining:


• Mark Comprehensive Basic Science Self-Assessment 4 hr 56 min 21 sec

29. A 3-month-old girl is brought to the emergency department by her parents because of a 2-day history of a deep cough "that sounds like a barking seal," a runny nose, and decreased
appetite. Her temperature is 37.8°C (100°F). Physical examination shows inspiratory strider. A diagnosis of croup is made, but the appropriate treatment does not result in
improvement. Examination of the airway after anesthesia is administered shows two squamous, warty lesions on the vocal cords. Which of the following viruses is the most likely
cause of these lesions?

A) Epstein-Barr virus
B) Herpes simplex virus D. Page 729 of my PDF. Pg 164/169 and pg 631/710.
U C) HIV This is a recurring questions NBME loves asking about.
U D) Human papillomavirus These warty lesions on the true vocal cords are a result of HPV 1, 2, 6, 11.
E) Poxvirus

, ~ ~ p , ,.,,.
Previous Next Lab Values Calculator Review Help Pause
hi Comprehensive Basic Science Self-Assessment - Google Chrome x
i starttest.com/ITDVersions/19.3.0.1/ITDStart.aspx?SVC=54cd3e5f-e4bf-49d0-8371-ba261251f2a1

Exam Section 2: Item 30 of 50 National Board of Medical Examiners Time Remaining:


• Mark Comprehensive Basic Science Self-Assessment 4 hr 56 min 15 sec

30. An 80-year-old woman comes to the physician because of a 1-month history of intermittent headaches and aching pain in her jaw while chew ing. She has no history of similar
headaches or pain. Neurologic examination shows no focal findings. Which of the following is the most appropriate next step in establishing the diagnosis?

CJ A) Determination of erythrocyte sedimentation rate


B) Leukocyte count
C) Chest x-ray
U D) MRI of the brain
E) Cerebrospinal fluid analysis

A. Page 310/338.
If a patient presents like this, it is important to know what tests to order even for CS and of course in real life.
This is a patient with headaches and jaw pain, which should immediately ring a bell for "jaw claudication" of giant cell temporal arteritis.
This patient should be started on prednisone asap before results of ESR return, because this can lead to irreversible blindness via opthalmic artery occlusion.

, ~ ~ p , ,.,,.
Previous Next Lab Values Calculator Review Help Pause
hi Comprehensive Basic Science Self-Assessment - Google Chrome x
i starttest.com/ITDVersions/19.3.0.1/ITDStart.aspx?SVC=54cd3e5f-e4bf-49d0-8371-ba261251f2a1

Exam Section 2: Item 31 of 50 National Board of Medical Examiners Time Remaining:


• Mark Comprehensive Basic Science Self-Assessment 4 hr 56 min 9 sec

31. A 50-year-old man comes to the physician because of diarrhea and a 13.6-kg (30-lb) weight loss over the past 2 months. He has three to four large, pale, foul-smelling stools with oil
droplets daily. He has a history of alcohol dependence and chronic abdominal pain. X-ray of the abdomen shows calcifications in the mid-upper abdomen. Which of the following
pathophysiologic mechanisms is the most likely cause of this patient's condition?

A) Generalized malabsorption
B) Motility disorder
C) Osmotic diuresis
D) Portal hypertension
E) Vitamin 8 12 (cobalamin) deficiency

A. Page 391/435.
Another excellent broad presentation. Any man with diarrhea without a fever is having GIT issues unrelated to infection.
On top of that, it is fatty diarrhea, indicative of malabsorption. Bloody diarrhea (dysentery) would be indicative of inflammation.
The alcohol dependence and abdominal calcification are indicative of pancreatitis, which seals the deal.

, ~ ~ p , ,.,,.
Previous Next Lab Values Calculator Review Help Pause
hi Comprehensive Basic Science Self-Assessment - Google Chrome x
i starttest.com/ITDVersions/19.3.0.1/ITDStart.aspx?SVC=54cd3e5f-e4bf-49d0-8371-ba261251f2a1

Exam Section 2: Item 32 of 50 National Board of Medical Examiners Time Remaining:


• Mark Comprehensive Basic Science Self-Assessment 4 hr 56 min 3 sec

32. A 58-year-old man is brought to the emergency department 1 hour after being kicked in the side by a horse. Two years ago, he was diagnosed with chronic idiopathic myelofibrosis.
He drinks two 12-ounce beers daily. His temperature is 37°C (98.6°F), pulse is 120/min, respirations are 18/min, and blood pressure is 90/50 mm Hg. Abdominal examination shows
guarding and tenderness over the left upper quadrant. Laboratory studies show:
Hemoglobin 9 g/dL
Hematocrit 27%
Leukocyte count 11 ,000/mm 3
Platelet count 280,000/mm3

Ultrasonography of the abdomen shows intraperitoneal fluid. Four hours later, his hematocrit is 24%. A laparotomy is most likely to show which of the following in this patient?

A) Mesenteric artery thrombosis


B) Pancreatic pseudocyst
C) Perforated stomach
D) Ruptured bladder
E) Ruptured spleen

E. Page 425/470.
The patient had a strong trauma from the horse kick, which is no joke.
Myelofibrosis is from the hematology chapter, which causes the bone marrow to be fibrosed due to increased fibroblast activity.
These patients have splenomegaly.
Trauma plus splenomegaly is an easy stage for splenic rupture, leading to nothing more than anemia (normal leukocyte and platelet count).

The other answer choices are irrelevant.


Mesentaric thrombosis would cause extreme bleeding inside and the patient would go into hypovolemic shock. The same for choice C.
Choice B can be seen in alcoholics and he might be an alcoholic but that is not what is the core of this question.
Choice D has nothing to do with his upper left quadrant.

, ~ ~ p , ,.,,.
Previous Next Lab Values Calculator Review Help Pause
hi Comprehensive Basic Science Self-Assessment - Google Chrome x
i starttest.com/ITDVersions/19.3.0.1/ITDStart.aspx?SVC=54cd3e5f-e4bf-49d0-8371-ba261251f2a1

Exam Section 2: Item 33 of 50 National Board of Medical Examiners Time Remaining:


• Mark Comprehensive Basic Science Self-Assessment 4 hr 55 min 58 sec

33. A 26-year-old man and his 25-year-old wife come to the physician for genetic counseling prior to conception. Both of the patients were previously found to have microcytic anemia
and an increased fraction of hemoglobin A 2. Genetic analysis shows that the man has a heterozygous null mutation of the B-globin gene, and the woman has a heterozygous
mutation known to cause a 50% decrease in B-globin gene function of one allele. Which of the following best represents the B-globin function possibilities for this couple's offspring?

A) All will have decreased B-globin function and transfusion-dependent anemia


B) One in four will have less than 10% B-globin function and transfusion-dependent anemia WOMAN
C) One in four will have 25% B-globin function and may require occasional transfusions
B B+
D) One in two will have 50% B-globin function without need for transfusions
E) One in two will have normal B-globin function without need for transfusions
B BB = 100% BB+ = 75%
beta globin beta globin
MAN

BB0 = 50% B0B+ = 25%


B0 beta globin
beta globin

C. Page 410/454.
Pretty clever question for a topic that is not really inherited in a standard pattern.
The punette square makes it very simple.
You can cancel out choices D and E because for D, with 50% beta globin you will need transfusions, and choice E says half the kids will be 100%,
which is unlikely given that the father has a null mutation and the mother has mutation too, so then only a quarter of kids will be 100%.

Not an easy or straightforward question unless you are confident and comfy with genetics and think of using one of the few genetics tools, the square.

, ~ ~ p , ,.,,.
Previous Next Lab Values Calculator Review Help Pause
hi Comprehensive Basic Science Self-Assessment - Google Chrome x
i starttest.com/ITDVersions/19.3.0.1/ITDStart.aspx?SVC=54cd3e5f-e4bf-49d0-8371-ba261251f2a1

Exam Section 2: Item 34 of 50 National Board of Medical Examiners Time Remaining:


• Mark Comprehensive Basic Science Self-Assessment 4 hr 55 min 49 sec

34. A 6-year-old girl is brought to the office by her mother because of a 2-day history of a severely itchy scalp. The mother reports that 10 children in her daughter's class also have the
same symptoms. The patient has no history of major medical illness and receives no medications. Vital signs are within normal limits. Examination of multiple hair shafts shows 1- to
2-mm, white, globular protuberances. Which of the following is the most likely cause of this patient's condition?

A) Cladosporium cladosporioides
C. Page 161/166.
B) Epidermophyton floccosum This is clearly lice related problems that commonly spread in early school years.
C) Pediculus humanus capitis Very easy straightforward question.
D) Sarcoptes scabiei
E) Trichophyton rubrum

, ~ ~ p , ,.,,.
Previous Next Lab Values Calculator Review Help Pause
hi Comprehensive Basic Science Self-Assessment - Google Chrome x
i starttest.com/ITDVersions/19.3.0.1/ITDStart.aspx?SVC=54cd3e5f-e4bf-49d0-8371-ba261251f2a1

Exam Section 2: Item 35 of 50 National Board of Medical Examiners Time Remaining:


• Mark Comprehensive Basic Science Self-Assessment 4 hr 55 min 43 sec

35. A 50-year-old man undergoes surgical transection of the obturator nerve for unbalanced muscle spasm affecting the hip. Which of the following muscles is most likely to be
paralyzed by this procedure?

CJ A) Adductor brevis
B) External oblique
C) lliopsoas
U D) Obturator internus
E) Piriformis

A. Page 444/495 FA2019.


I always remember the uworld colored images of sensory innervation and that helps.
Obturator is an adductor nerve.
It innervates adductor longus brevis and magnus, obturator externus, and the gracilis & pectinues.

The obliques are not hip muscles or even part of the legs. The internal oblique is innervated by iliohypogastric.
Iliopsoas has an iliacus portion innervated by the femoral nerve.
Obturator internus was a trick answer choice you must remember is something that is supplied by
the nerve to the obturator internus, a branch directly off the sacral plexus.
The piriformis also has a nerve to the piriformis from the sacral plexus.

, ~ ~ p , ,.,,.
Previous Next Lab Values Calculator Review Help Pause
hi Comprehensive Basic Science Self-Assessment - Google Chrome x
i starttest.com/ITDVersions/19.3.0.1/ITDStart.aspx?SVC=54cd3e5f-e4bf-49d0-8371-ba261251f2a1

Exam Section 2: Item 36 of 50 National Board of Medical Examiners Time Remaining:


• Mark Comprehensive Basic Science Self-Assessment 4 hr 55 min 38 sec

36. A 24-year-old man is brought to the office because of fever and chills for 1 day. He is a paraplegic and uses a wheelchair. He has a history of recurrent urinary tract infections treated
with different antibiotics over the past year. His temperature is 38.5°C (101.3°F). Urine cultures are ordered, and treatment with ciprofloxacin is begun . Two days later, he is still
febrile, and the initial urine cultures grow Escherichia coli resistant to ciprofloxacin. Which of the following is the most likely cause of this organism's resistance?

A) Acquisition of a plasmid encoding ciprofloxacin acetylase


B) Acquisition of a plasmid encoding 238 ribosomal RNA methylase
C) Alteration in DNA gyrase
D) Alteration in penicillin-binding proteins
E) Alteration in 308 ribosomal RNA
F) Alteration in RNA polymerase

C. Page 191/201 and 195/205.


Very easy question. Ciprofloxocin is a fluoroquinolone, which works on topoisomerase 2 and 4.
Topoisomerase 2 is DNA gyrase. Just pure memorization.

Choices A and B are very unlikely as cultures only showed E coli, so there is no other bacteria that could conjugate and send that plasmid.
Ciprofloxacin acetylase is a real mechanism of E coli resistance to -floxacins.
But the 23s ribosomal RNA Methylase is irrelevant. That would be for macrolide resistance. The 50s subunit includes the 5s and 23s subunits.

Choices D and F are completely off.


Choice E is related to aminoglycosides.

, ~ ~ p , ,.,,.
Previous Next Lab Values Calculator Review Help Pause
hi Com prehensive Basic Science Self-Assessment - Goog le Chrome x
i starttest.com/ITDVersions/19.3.0.1/ITDStart.aspx?SVC=3f50c3c0-1874-400e-af3a-5036e79dec9f

Exam Section 1: Item 37 of 50 National Board of Medical Examiners Time Remaining:


• Mark Comprehensive Basic Science Self-Assessment 4 hr 52 min 45 sec

37. A 30-year-old man who is a migrant farm worker comes to the emergency department because of a 1-week history of fever, chills, and severe headache. He immigrated to the
midwestern USA from Honduras 1 year ago. He is alert. His temperature is 40°C (104 °F), pulse is 101 /min, respirations are 20/min, and blood pressure is 105/62 mm Hg. The
lungs are clear. Cardiac examination shows no abnormalities. The abdomen is soft with active bowel sounds. Neurologic examination shows a supple neck; there is no nuchal
rigidity. He is oriented to person , place, and time. Laboratory studies show:
Hemoglobin 14 g/dL
Hematocrit 42%
Leukocyte count 4600/mm 3
Segmented neutrophils 57%
Lymphocytes 33%
Monocytes 10%
Urine
Color clear yellow
Blood 1+
Urobilinogen trace

, ~ ~ p, ,.,,.
Previous Next Lab Values Calculator Review Help Pause
hi Com prehensive Basic Science Self-Assessment - Goog le Chrome x
i starttest.com/ITDVersions/19.3.0.1/ITDStart.aspx?SVC=3f50c3c0-1874-400e-af3a-5036e79dec9f

37. A 30-year-old man who is a migrant farm worker comes to the emergency department because of a 1-week history of fever, chills, and severe headache. He immigrated to the
midwestern USA from Honduras 1 year ago. He is alert. His temperature is 40°C (104 °F), pulse is 101 /min, respirations are 20/min, and blood pressure is 105/62 mm Hg. The
lungs are clear. Cardiac examination shows no abnormalities. The abdomen is soft with active bowel sounds. Neurologic examination shows a supple neck; there is no nuchal
rigidity. He is oriented to person , place, and time. Laboratory studies show:
Hemoglobin 14 g/dL
Hematocrit 42%
Leukocyte count 4600/mm 3
Segmented neutrophils 57%
Lymphocytes 33%
Monocytes 10%
Urine
Color clear yellow
Blood 1+
Urobilinogen trace

A photomicrograph of a peripheral blood smear is shown. This patient's overall condition is most likely caused by an infectious agent with which of the following features?

A) Anaerobic metabolism D. Page 157/162.


U B) Chloroquine resistance Source of image --> http://spot.pcc.edu/~jvolpe/b/bi234/lec/2_parasites/images/P._vivax.htm
C) Exotoxin production
This is Schuffner stippling due to Plasmodium vivax.
In general you see something like this you immediately think of Plasmodium.
D) Formation of hypnozoites
It could be Chloroquine resistant (an answer choice that should make you think of Plasmodium), but not necessarily.
E) Induction of adhesion molecules
He must have had hypnozoites before he came to the US a year ago.
hi Comprehensive Basic Science Self-Assessment - Google Chrome x
i starttest.com/ITDVersions/19.3.0.1/ITDStart.aspx?SVC=54cd3e5f-e4bf-49d0-8371-ba261251f2a1

Exam Section 2: Item 38 of 50 National Board of Medical Examiners Time Remaining:


• Mark Comprehensive Basic Science Self-Assessment 4 hr 55 min 28 sec

38. A 37-year-old woman comes to the physician because of nausea and vomiting. She drank half a bottle of whiskey the previous evening because "I was in great despair because my
boyfriend broke up with me. I normally don't drink alcohol , but I had to calm my nerves." She describes her mood as depressed, angry, and unstable. She says, "There is no one I
can trust except you. I recently started a job and , from what I can tell , everyone I work with is a jerk." This patient most likely has which of the following personality traits?

A) Antisocial
B) Borderline
B. Page 553/617 of FA2019.
Very nice question.
C) Dependent
Her misery after the breakup makes you think of dependent.
D) Narcissistic
Her comment about only being able to trust the doctor and no one else makes you think of paranoid.
E) Paranoid Remember the clusters to help figure this out.

She best fits borderline due to her unstable mood, impulsive reckless drinking, emotional emptiness, and work related splitting.
Antisocial is in the same cluster B but that would be criminality.
Narcissistic is also a cluster B but that would be grandiosity that she does not exhibit at all.

, ~ ~ p , ,.,,.
Previous Next Lab Values Calculator Review Help Pause
hi Comprehensive Basic Science Self-Assessment - Google Chrome x
i starttest.com/ITDVersions/19.3.0.1/ITDStart.aspx?SVC=54cd3e5f-e4bf-49d0-8371-ba261251f2a1

Exam Section 2: Item 39 of 50 National Board of Medical Examiners Time Remaining:


• Mark Comprehensive Basic Science Self-Assessment 4 hr 55 min 23 sec

39. A 55-year-old man with type 1 diabetes mellitus comes to the physician because of intermittent burning pain of his feet during the past 4 months. Examination of the feet shows
allodynia bilaterally. Sensation to pinprick is decreased. Motor strength, deep tendon reflexes, joint position, and vibration sense are normal. Which of the following is the most likely
cause of the pain in this patient?

A) Increased activation of glutamate receptors in the dorsal root ganglia


B) Increased activity of presynaptic y-aminobutyric acid receptors in the dorsal horn
C) Increased activity of voltage-gated K+ channels in the thalamus
D) Inhibition of vanilloid receptors in the dorsal root afferents
E) Persistent activation of voltage-gated Na+ channels in the nociceptor

E.
This question was hard.
They try to present the question as a simple diabetic neuropathy question.
The allodynia (pain from things that do not normally cause pain) is key here. This patient has increased sensitivity.
Allodynia is not due to central things such as the DRG, GABA of the dorsal horn, receptors in the thalamus, or dorsal root afferents.
Choice E depicts a peripheral receptor being overactivated, as is the case in peripheral sensitization due to the diabetes.

, ~ ~ p , ,.,,.
Previous Next Lab Values Calculator Review Help Pause
hi Comprehensive Basic Science Self-Assessment - Google Chrome x
i starttest.com/ITDVersions/19.3.0.1/ITDStart.aspx?SVC=54cd3e5f-e4bf-49d0-8371-ba261251f2a1

Exam Section 2: Item 40 of 50 National Board of Medical Examiners Time Remaining:


• Mark Comprehensive Basic Science Self-Assessment 4 hr 55 min 18 sec

40. A 75-year-old man is brought unconscious to the emergency department by his wife after he collapsed 30 minutes
ago. The wife says he had abdominal pain that morning, but he did not intend to visit the physician. He has not had a
physical examination since childhood. His wife says she has been begging him to go to the physician ever since he
noticed an abdominal mass 2 years ago. He has smoked 2 packs of cigarettes daily for 60 years and drinks three
12-ounce cans of beer daily. His temperature is 36.8°C (98.2°F), pulse is 80/min and irregular with premature
ventricular contraction , respirations are 35/min , and blood pressure is 110/60 mm Hg. Physical examination shows a
pulsatile abdominal mass and minimal bowel sounds. A CT scan of the abdomen is shown. Which of the following is
the most likely underlying cause of this patient's condition?

A) Atherosclerosis
u B) Diverticulosis with abscess
C) Marfan syndrome
D) Mycotic inflammation
E) Syphilis
() F) Takayasu arteritis

A. Page 300/325.
Classic presentation of an elderly (possibly white man) with an extensive smoking history; AAA.
Atherosclerosis most commonly occurs in the abdominal aorta. This inflammatory process leads to aneurysmal dilatation of the abdominal aorta.
In the image, calcification of the aortic wall has occurred.

Good knowledge -- http://www.lumen.luc.edu/lumen/MedEd/Radio/curriculum/Surgery/aneurysm2.htm


Amazing case scenario -- https://www.ncbi.nlm.nih.gov/pmc/articles/PMC6178120/

, ~ ~ p , ,.,,.
Previous Next Lab Values Calculator Review Help Pause
hi Comprehensive Basic Science Self-Assessment - Google Chrome x
i starttest.com/ITDVersions/19.3.0.1/ITDStart.aspx?SVC=54cd3e5f-e4bf-49d0-8371-ba261251f2a1

Exam Section 2: Item 41 of 50 National Board of Medical Examiners Time Remaining:


• Mark Comprehensive Basic Science Self-Assessment 4 hr 55 min 10 sec

41. A 32-year-old man is brought to the emergency department after sustaining a deep laceration that may have injured the nerve supply to the flexor digitorum profundus muscle of the
index finger. Which of the following movements can be used to specifically test for the function of this muscle?

CJ A) Abduction of a finger
B) Adduction of a finger
C) Flexion at the distal interphalangeal joint
U D) Flexion at the metacarpophalangeal joint
E) Flexion at the proximal interphalangeal joint

C. Page 442/488 of FA2019 but this question's answer is not really in FA.

The flexor digitorum profundus has a lateral median nerve half and a medial ulnar nerve half. It inserts at the DIP. Profundus is profoundly long.
DIP - P for profundus.
This patient's lateral index finger is injured so thats a laceration of the median nerve.

Choice D. The lumbricals flex at the MCP joint and extend the PIP & DIP joints. Lateral lumbricals by median n and medial lumbricals by ulnar n.
Lumbricals lengthen.

Choice E.The flexor digitorum superficialis is innervated by the median nerve and it inserts at the middle phalanges of the medial 4 fingers.
This helps flex at the PIP.

, ~ ~ p , ,.,,.
Previous Next Lab Values Calculator Review Help Pause
hi Comprehensive Basic Science Self-Assessment - Google Chrome x
i starttest.com/ITDVersions/19.3.0.1/ITDStart.aspx?SVC=54cd3e5f-e4bf-49d0-8371-ba261251f2a1

Exam Section 2: Item 42 of 50 National Board of Medical Examiners Time Remaining:


• Mark Comprehensive Basic Science Self-Assessment 4 hr 55 min 5 sec

42. A 32-year-old woman who is 20 weeks' pregnant has had increasingly severe episodes of heartburn for the past 3 weeks. She has mild epigastric tenderness on palpation of the
abdomen. Which of the following drugs is contraindicated because of an increased risk for spontaneous abortion?

CJ A) Bismuth subsalicylate
B) Famotidine C. Page 642/723.
This is literally a medication used for abortion.
C) Misoprostol
U D) Omeprazole
E) Sucralfate

, ~ ~ p , ,.,,.
Previous Next Lab Values Calculator Review Help Pause
hi Comprehensive Basic Science Self-Assessment - Google Chrome x
i starttest.com/ITDVersions/19.3.0.1/ITDStart.aspx?SVC=54cd3e5f-e4bf-49d0-8371-ba261251f2a1

Exam Section 2: Item 43 of 50 National Board of Medical Examiners Time Remaining:


• Mark Comprehensive Basic Science Self-Assessment 4 hr 54 min 58 sec

43. A 55-year-old man is brought to the emergency department because of a 1-hour history of severe chest pain, nausea, and vomiting. He is agitated, clammy, and sweating profusely.
His temperature is 36.?°C (98°F), pulse is 130/min, respirations are 36/min, and blood pressure is 85/45 mm Hg. Jugular venous pressure is 12 cm H 20 and crackles are heard in
the lung bases bilaterally. Which of the following hemodynamic changes is most likely in this patient?

Pulmonary Capillary Wedge Cardiac Output Systemic Vascular


Pressure Resistance
U A) i i i
U B) i l i
u C) l l l
D) l l i
E) l i l

B. Page 307/335.
The severe chest pain might be indicative of a potential MI or HF.

The question gives everything needed to make the right choice but it is based on your knowledge of cardiovascular topics.
The clammy, sweating patient with normal temp but low BP seems like a case of shock.
More specifically, this must be cardiogenic shock (not septic because of normal temp, no infection).

Since the patient has crackles in the lung, blood must be backing up and pressures must be high in PCWP. This immediately brings you to A & B.
However, PCWP can be high or low in cardiogenic or obstructive shock based off of the specific issue causing the shock.

Recall that the only shock where SVR is low is distributive shock, so you know for SURE that SVR must be high.
I speak in my book about how no matter what shock we discuss, the underlying physiology is always based off of CO x SVR = MAP.
In distributive, SVR is very low so CO tries to be increased to maintain MAP.
In other shocks, SVR is high and CO is low, which brings us to choices B and D. 50% chance of guessing right.

Figuring out whether PCWP would be high or low comes from what i initially said, the normal crackles in the lung.
This patients JVP is high due to poor forward-flow of blood in this patients specific case. So their PCWP must be high too for the same poor forward-flow.
If the infarct were only on the right heart, blood would back up and the JVP would be even higher and not much blood would make it to the left
heart so the PCWP would be low in that case. But most cardiogenic shock cases have infarct of both sides of the heart.

, ~ ~ p , ,.,,.
Previous Next Lab Values Calculator Review Help Pause
hi Comprehensive Basic Science Self-Assessment - Google Chrome x
i starttest.com/ITDVersions/19.3.0.1/ITDStart.aspx?SVC=54cd3e5f-e4bf-49d0-8371-ba261251f2a1

Exam Section 2: Item 44 of 50 National Board of Medical Examiners Time Remaining:


• Mark Comprehensive Basic Science Self-Assessment 4 hr 54 min 53 sec

44. A 62-year-old man comes to the physician for a follow-up examination. One month ago, he was diagnosed with atrial and ventricular arrhythmia, and appropriate pharmacotherapy
was initiated at that time. Today, his pulse is 64/min, and blood pressure is 136/88 mm Hg; 1 month ago, his pulse was 78/min, and blood pressure was 152/95 mm Hg. Physical
examination shows no other abnormalities. An ECG shows normal sinus rhythm with a prolonged QT interval. Which of the following drugs is the most likely cause of these findings?

A) Adenosine
B) Flecainide
C) Metoprolol
D) Phenytoin
E) Sotalol

E. Page 318/347 FA2019.


Memorization of the antiarrhythmics takes time and practice.
The class 3 AIDS drugs are potassium channel blockers. Amiodarone is very high-yield.
I once saw a UW question that tested on ONE SPECIFIC thing. As bad as amiodarone's side effects might be, it does not cause torsades.
The other drugs, Sotalol and Ibuitlide cause torsades (QT prolongation).
On top of that, sotalol also has beta blocker activity that reduced the patients pulse and BP.

Adenosine is a high-yield cool medication i review extensively in my book.


Flecainide (eat more fries please) is one of the class 1c meds. In class 1, only thet class 1a cause torsades.
Metoprolol is just a beta blocker, so that is irrelevant.
Phenytoin can be placed in class 1b (lettuce mayo tomato please).

, ~ ~ p , ,.,,.
Previous Next Lab Values Calculator Review Help Pause
hi Com prehensive Basic Science Self-Assessment - Goog le Chrome x
i starttest.com/ITDVersions/19.3.0.1/ITDStart.aspx?SVC=54cd3e5f-e4bf-49d0-8371-ba261251f2a1

Exam Section 2: Item 45 of 50 National Board of Medical Examiners Time Remaining:


• Mark Comprehensive Basic Science Self-Assessment 4 hr 54 min 46 sec

45. A 34-year-old woman with a long history of rheumatoid arthritis refractory to treatment with corticosteroids and methotrexate comes to the physician because of a 2-week history of
increasingly severe low back pain, decreased appetite, and malaise. She has never had low back pain in the past. She has not had fever, lower extremity weakness, constipation,
bladder or bowel incontinence, weight loss, or sensory changes. Six weeks ago, she began treatment with infliximab because of bilateral wrist, hand, and knee pain. Her
temperature is 37°C (98.6°F). Physical examination shows mild tenderness over the L 1-2 vertebrae. Laboratory studies show:
Hemoglobin 11.3 g/dL
Hematocrit 32%
Leukocyte count 7800/mm 3
Platelet count 290,000/mm3

An MRI of the spine shows osteomyelitis of L1-2 with destruction of the intervertebral disc space and a 1.2-cm adjacent abscess. The abscess is debrided, and samples of the
abscess and bone tissue are sent for analysis. A Gram stain is negative. Pathologic examination of the bone specimen shows acute and chronic inflammation with granuloma
formation. Which of the following is the most likely cause of these findings?

A) Ankylosing spondylitis
U B) Multiple myeloma
C) Progression of rheumatoid arthritis
() D) Sarcoidosis
E) Tuberculous osteomyelitis

E. Page 122 for TNF, and page 140/141 of FA2019 for TB.
First of all, choices A-D make no sense.
Second, remember your TNF alpha inhibitors. This patient started infliximab, which is one of the "In Ada Ce Goli" meds from the Hindi mnemonic I made.
The mnemonic says "In Ada Ce Goli marti, totally not fair." It is a corny phrase from an old Indian song. It means you're shooting at me with your beauty,
totally not fair. Something like that but it gets the job done. Totally Not Fair is for TNF.

The patient must have latent TB that was previously locked in the granuloma. With the TNFa inhibitor, that granuloma got weak and the TB reactivated.
Now she has Potts. Disseminated TB causing osteomyelitis. And TB, Mycoplasma, is acid fast so it does not gram stain.

, ~ ~ p , ,.,,.
Previous Next Lab Values Calculator Review Help Pause
hi Comprehensive Basic Science Self-Assessment - Google Chrome x
i starttest.com/ITDVersions/19.3.0.1/ITDStart.aspx?SVC=54cd3e5f-e4bf-49d0-8371-ba261251f2a1

Exam Section 2: Item 46 of 50 National Board of Medical Examiners Time Remaining:


• Mark Comprehensive Basic Science Self-Assessment 4 hr 54 min 41 sec

46. A 32-year-old woman comes to the physician because of a 7-day history of sneezing, nasal stuffiness, and watery eyes. She has a history of similar symptoms each spring while
gardening. Her temperature is 37°C (98.6°F). Which of the follow ing types of cells are most likely to be increased in her nasal secretions as a result of this reaction?

CJ A) Basophils B. Page 400/444 for eosinophils, and page


B) Eosinophils Great question. Tough choices.
C) Lymphocytes Read the question carefully. They want the result of this reaction.
U D) Mast cells The reaction itself is initiated by mast cell degranulation (allergies).
E) Monocytes Mast cells release many things and the histamine and eosinophil chemotactic factors are crucial.

The result is from later activity by eosinophils, which are activated by IL5. There is IgE clustering and cross linking.
Type 1 HSR.

, ~ ~ p , ,.,,.
Previous Next Lab Values Calculator Review Help Pause
hi Comprehensive Basic Science Self-Assessment - Google Chrome x
i starttest.com/ITDVersions/19.3.0.1/ITDStart.aspx?SVC=54cd3e5f-e4bf-49d0-8371-ba261251f2a1

Exam Section 2: Item 47 of 50 National Board of Medical Examiners Time Remaining:


• Mark Comprehensive Basic Science Self-Assessment 4 hr 54 min 36 sec

47. A 35-year-old man comes to the physician 1 week after he had a high blood pressure reading taken while he was donating blood. His pulse is 76/min, and blood pressure is
180/100 mm Hg. Physical examination shows no other abnormalities. Treatment with losartan is started. Which of the following is most likely to occur in this patient?

CJ A) Decreased plasma renin activity


B) Decreased serum bradykinin concentration
C) Increased serum aldosterone concentration
U D) Increased serum angiotensin II concentration
E) Increased serum norepinephrine concentration

D. Page 576/645.
Losartan is an ARB.
This is given instead of an ACEinhibitor if there is a chance the patient might get bradykinin induced cough. Bradykinin can be high with ACEinhib use.

Normally, RAAS causes eventual angiotensin 2 production which works to cause hypertension.
Without angiotensin 2 activity, there will be a feedback that will cause more renin to be made (A is wrong) and more angiotensin 1 and 2 to be made.
There will not be more aldosterone made.

Angiotensin 2 uses AT1 receptors to induce more aldosterone secretion from zona glomerulosa, but this receptor will ALSO be blocked.
If there was more aldosterone, the patient would still have hypertension with losartan treatment so C is ridiculous.

Angiotensin 2 is the final messenger that causes all the effects, and it will not be able to anymore.

, ~ ~ p , ,.,,.
Previous Next Lab Values Calculator Review Help Pause
hi Comprehensive Basic Science Self-Assessment - Google Chrome x
i starttest.com/ITDVersions/19.3.0.1/ITDStart.aspx?SVC=54cd3e5f-e4bf-49d0-8371-ba261251f2a1

Exam Section 2: Item 48 of 50 National Board of Medical Examiners Time Remaining:


• Mark Comprehensive Basic Science Self-Assessment 4 hr 54 min 30 sec

48. Immediately after a spontaneous vaginal delivery, a full-term male newborn is found to have a tuft of hair on the midline in the lumbar region. Palpation of the area shows an
absence of spinous processes in that region. Imaging studies show no additional abnormalities. The most likely cause of this defect was failure of which of the following events
during gestation?

A) Development of the notochord


B) Formation of the neural tube
C) Formation of the yolk sac
D) Fusion of the sclerotomes
E) Migration of the neural crests
D. Page 479/537 of FA2019.
Witty NBME wording. The tuft of hair and lack of spinous processes is diagnostic of SBO.

Patient has Spina bifida occulta which is a neural tube defect (failure of fusion of the neuropores).
Sclerotomes are the part of each somite in a vertebrate embryo giving rise to bone or other skeletal tissue.
Since a part of this patient's spina bifida included "absense of spinous processes" then a sclerotome was involved.
Knowing that neural tube defects are an issue with fusion should be enough to get to the right answer.

Choices A and B are nonsense because of the notochord or neural tube did not develop/form, the entire CNS would not form.
The yolk sac has nothing to do with this scenario.
Neural crest migration is a bait but it is irrelevant to neural tube defects.

Sclerotomes -> vertebrae, rib cage, occipital bone (part)


Myotomes -> skeletal muscle of the back, ribs, limbs
Syndetomes -> cartilage, tendons
Dermatomes -> skin of the back

, ~ ~ p , ,.,,.
Previous Next Lab Values Calculator Review Help Pause
hi Comprehensive Basic Science Self-Assessment - Google Chrome x
i starttest.com/ITDVersions/19.3.0.1/ITDStart.aspx?SVC=54cd3e5f-e4bf-49d0-8371-ba261251f2a1

Exam Section 2: Item 49 of 50 National Board of Medical Examiners Time Remaining:


• Mark Comprehensive Basic Science Self-Assessment 4 hr 54 min 24 sec

49. A 35-year-old woman comes to the physician because of a 1-year history of fatigue and lethargy; she also has had a 9-kg (20-lb) weight gain during this period. She says that she
has to wear a coat in her office because she is always cold. Physical examination shows coarse skin. Examination of the neck shows no palpable thyroid gland tissue. Neurologic
examination shows a delayed relaxation phase of the calcaneal (Achilles) reflexes. Serum concentrations of thyroxine (T 4) and triiodothyronine (T 3) are decreased. A thyroid scan
shows localization of the radioiodine in the midline of the inferior aspect of the oral cavity; no radioactivity is detected in the neck. The radioiodine is most likely localized inferior to
which of the following structures?

A) Foramen cecum
B) Frenulum of the tongue
C) Median glossoepiglottic fold
lJ D) Sublingual caruncle
U E) Tracheobronchial bifurcation

A. Page 322/351.
The patient has a failed migration of the thyroid gland?
Not sure why symptoms would appear after age 30.
Either way, the other choices are far from related to the thyroid.

, ~ ~ p , ,.,,.
Previous Next Lab Values Calculator Review Help Pause
hi Comprehensive Basic Science Self-Assessment - Google Chrome x
i starttest.com/ITDVersions/19.3.0.1/ITDStart.aspx?SVC=54cd3e5f-e4bf-49d0-8371-ba261251f2a1

Exam Section 2: Item 50 of 50 National Board of Medical Examiners Time Remaining:


• Mark Comprehensive Basic Science Self-Assessment 4 hr 54 min 17 sec

50. Results of a 5-year screening program for HIV infection in a population of 10,000 commercial sex workers are shown:
Time Number Remaining in Population Number of New Patients With Positive Serology
Intake 10,000 4000
1 year 6000 400 2yrs --> 6.7%
2 years 5600 250 3yrs --> 4.5%
3 years 5350 300
4yrs --> 5.6%
4 years 5050 300
5 years 4800 250
5yrs --> 5.2%

Which of the following percentages is closest to the average annual incidence of infection in this population?

A) 1% B.
U B) 5% Just simple math. Nothing insane. They didnt even try to confuse you with incidence vs prevalence.
C) 15%
U D) 20%
lJ E) 35%
U F) 40%

, ~ ~ p , ,.,,.
Previous Next Lab Values Calculator Review Help Pause
hi Comprehensive Basic Science Self-Assessment - Google Chrome x
i starttest.com/ITDVersions/19.3.0.1/ITDStart.aspx?SVC=54cd3e5f-e4bf-49d0-8371-ba261251f2a1

Exam Section 3: Item 1 of 50 National Board of Medical Examiners Time Remaining:


• Mark Comprehensive Basic Science Self-Assessment 4 hr 59 min 41 sec

1. An investigator is studying a new drug for the treatment of patients undergoing adjuvant radiation therapy. The drug is designed to be administered prior to irradiation to minimize
localized tissue damage at the irradiated site. This drug most likely inhibits which of the following effects of external beam radiation?

CJ A) Depurination
E. Page 40
B) DNA ligase inactivation Tricky question where many will fall for choice D.
C) DNA polymerase activation There are generally to types of radiation.
U D) Formation of pyrimidine dimers Radiotherapy is ionizing and causes free radical formation.
E) Free radical formation The non-ionizing UVB from sunlight causes pyrimidine dimer formation that requires nucleotide excision repair.
F) Tautomerization Thing NERD- Nucletide Excision Repair of Dimers.

Base excision repair is for non-bulky DNA base alterations, while mismatch repair is for Lynch syndrome.

~ ~ p , ,.,,.
Next Lab Values Calculator Review Help Pause
hi Comprehensive Basic Science Self-Assessment - Google Chrome x
i starttest.com/ITDVersions/19.3.0.1/ITDStart.aspx?SVC=54cd3e5f-e4bf-49d0-8371-ba261251f2a1

Exam Section 3: Item 2 of 50 National Board of Medical Examiners Time Remaining:


• Mark Comprehensive Basic Science Self-Assessment 4 hr 59 min 31 sec

2. A 70-year-old man comes to the physician because of a 1-year history of progressive shortness of breath and nonproductive cough. He is now unable to tolerate even mild activity.
Physical examination shows clubbing of the fingers. lnspiratory crackles are heard at both lung bases. A CT scan of the chest shows patchy subpleural reticular opacities.
Examination of a lung biopsy specimen shows a heterogeneous pattern with alternating areas of normal lung and interstitial inflammation and fibrosis. On questioning, he has not
taken any medications or had environmental exposures associated with pulmonary fibrosis. Which of the following pulmonary function tests in this patient will most likely show a result
greater than the predicted range?

A) Alveolar-arterial Po 2 difference
B) Diffusing capacity measured with carbon monoxide
C) Functional residual capacity
lJ D) FVC
U E) Residual volume
U F) Tidal volume

A. Page 661/754 while page 655/742 discusses A-a.


The patient simply has pulmonary fibrosis, which can often be idiopathic.
This is a restrictive lung disease so all lung volumes generally decrease, making C-F all wrong.
Choice B is a trick answer. The diffusion capacity of of CO would be LOW not increased.
DLCO is low in interstitial lung diseases (all restrictive lung diseases that are not due to poor breathing mechanics) and emphysema.
DLCO is higher sometimes in asthma.

Not a hard question at all.


The A-a gradient increases in interstitial restrictive lung diseases due to more oxygen being in the alveoli but faulty diffusion into arteries.

, ~ ~ p , ,.,,.
Previous Next Lab Values Calculator Review Help Pause
hi Comprehensive Basic Science Self-Assessment - Google Chrome x
i starttest.com/ITDVersions/19.3.0.1/ITDStart.aspx?SVC=54cd3e5f-e4bf-49d0-8371-ba261251f2a1

Exam Section 3: Item 3 of 50 National Board of Medical Examiners Time Remaining:


• Mark Comprehensive Basic Science Self-Assessment 4 hr 59 min 1 sec

3. A 4-year-old boy has a 1-cm round mid line mass just inferior to the hyoid bone. The mass was present at birth and remains unchanged. The mass is most likely derived from tissue
that originated from which of the following structures?

CJ A) Pharyngeal arch D. Page 322/351.


B) Submandibular gland The patient's mass is a midline mass that is below the hyoid bone.
C) Thymus This most likely is not thyroid tissue since the question is not hinting anything about it.
U D) Tongue
So then it must be a thyroglossal duct cyst (based on the presentation).
This mass would move with swallowing (compared to a pharyngeal cleft cyst, not pharyngeal arch like choice A).
E) Trachea

The ectopic thyroid tissue would be an answer choice of thyroid, which is not given.
The thyroglossal duct cyst is a derivative of the tongue, which is why it moves.

Good osmosis video -- https://www.youtube.com/watch?v=snwEs0u99sY&feature=youtu.be

, ~ ~ p , ,.,,.
Previous Next Lab Values Calculator Review Help Pause
hi Comprehensive Basic Science Self-Assessment - Google Chrome x
i starttest.com/ITDVersions/19.3.0.1/ITDStart.aspx?SVC=54cd3e5f-e4bf-49d0-8371-ba261251f2a1

Exam Section 3: Item 4 of 50 National Board of Medical Examiners Time Remaining:


• Mark Comprehensive Basic Science Self-Assessment 4 hr 58 min 30 sec

4. A 35-year-old man with quadriplegia develops a urinary tract infection. He was admitted to the neurology unit of the hospital 7 months ago. He has had an intermittent indwelling
urinary catheter during this entire period. A culture of urine grows an organism that is susceptible only to aminoglycosides. Gentamicin is administered. If nephrotoxicity occurs in this
patient, it is most likely to originate in which of the following labeled sites depicted in the drawing of the nephron shown?

B. Page 191/201.
Aminoglycosides have a risk for nephrotoxicity, neuromuscular
blockade, ototoxicity, and teratogenicity.

The nephrotoxicity targets the PCT.


Nephrotoxic drugs cause acute tubular necrosis, which is an
ischemic event. The two portions of a nephron that are
susceptible to ischemia are the PCT and the thick
ascending limb (not labelled). E

U A) lJ B) C) D) E)

, ~ ~ p , ,.,,.
Previous Next Lab Values Calculator Review Help Pause
hi Comprehensive Basic Science Self-Assessment - Google Chrome x
i starttest.com/ITDVersions/19.3.0.1/ITDStart.aspx?SVC=54cd3e5f-e4bf-49d0-8371-ba261251f2a1

Exam Section 3: Item 5 of 50 National Board of Medical Examiners Time Remaining:


• Mark Comprehensive Basic Science Self-Assessment 4 hr 58 min 20 sec

5. A 52-year-old man is admitted to the hospital for treatment of a massive acute myocardial infarction. Treatment with oxygen, (3-adrenergic blockers, aspirin, and reperfusion therapy is
initiated. Eighteen days later, he develops ventricular fibrillation , from which he cannot be resuscitated. A photomicrograph of cardiac tissue from the site of infarct obtained at autopsy
is shown. Which of the following best describes the appearance of this patient's heart?

U A) Abscess D. Page 302/327 of FA2019.


0 B) Coagulation necrosis This question is about the evolution of an MI.
C) Dense fibrous scar It has been 18 days since the patient had his MI.
D) Granulation tissue Memorizing this evolution takes revision and time, as usual. But it is important to understand that fibrosis takes lots of time.
u E) Granuloma The contracted, type 1 collagen, fibrotic scar tissue forms after a month.

, ~ ~ p , ,.,,.
Previous Next Lab Values Calculator Review Help Pause
hi Comprehensive Basic Science Self-Assessment - Google Chrome x
i starttest.com/ITDVersions/19.3.0.1/ITDStart.aspx?SVC=54cd3e5f-e4bf-49d0-8371-ba261251f2a1

Exam Section 3: Item 6 of 50 National Board of Medical Examiners Time Remaining:


• Mark Comprehensive Basic Science Self-Assessment 4 hr 58 min 13 sec

6. An 80-year-old man is admitted to the hospital because of a 2-week history of a severe, persistent urinary tract infection. An 80-mg dose of Drug X is administered intravenously.
Thirty minutes after the infusion is complete, his serum concentration of Drug Xis 4 µg/ml. Assuming a distribution half-life of 3 minutes and an elimination half-life of 24 hours, the
volume of distribution of this drug (in L) in the peripheral compartment is closest to which of the following?

A) 2 E. Page 233/243.
B) 5
C) 10 Administered 80mg dose
D) 15 Distribution half life = 3min
E) 20 Elimination half life = 24 hours
F) 50
30 minutes have passed and serum has 4ug/mL.

This question gave so much information. Intimidation factor.


Just read the question. They just want volume of distritbution, which is drug administered divided by plasma drug concentration.

So that would be 80mg and 4ug/mL. Now converting it becomes a thing.


(4 micrograms /mL)(1000mL/L)(1g/1million micrograms) = .004g/L
(80mg)(1g/1000mg)=.08g
So .08 / .004 = 20L

, ~ ~ p , ,.,,.
Previous Next Lab Values Calculator Review Help Pause
hi Comprehensive Basic Science Self-Assessment - Google Chrome x
i starttest.com/ITDVersions/19.3.0.1/ITDStart.aspx?SVC=54cd3e5f-e4bf-49d0-8371-ba261251f2a1

Exam Section 3: Item 7 of 50 National Board of Medical Examiners Time Remaining:


• Mark Comprehensive Basic Science Self-Assessment 4 hr 58 min 7 sec

7. A 68-year-old man has loss of pain and temperature sensations on the right side of the face and left side of the body, paralysis of the vocal cord on the right, and absence of the gag
reflex on the right. Which of the following regions of the brain stem is most likely to be involved?

CJ A) Left ventromedial medulla


D. Page 497/555 and pg 502/562 of FA2019.
B) Right ventromedial medulla The patient is presenting with PICA infarct (same as q76, q 26 of block 2).
C) Left dorsolateral medulla Since the patient lost sensation on the right face, the lesion must be on the right so that knocks out half the answers.
U D) Right dorsolateral medulla Now you are left with B, D, F, and H.
E) Left ventrolateral pons The first losses the patient has are mainly pain and temperature, meaning the spinothalamic nucleus is involved.
F) Right ventrolateral pons The loss of his vocal cords and gag reflex are indiciative of CN9 and 10, which are on the medulla.
G) Left ventromedial midbrain This brings you to just choices B or D.
H) Right ventromedial midbrain You pick dorsolateral if you studied the anatomy of the images below.
The spinothalamic tract runs dorsolaterally.

, ~ ~ p , ,.,,.
Previous Next Lab Values Calculator Review Help Pause
hi Comprehensive Basic Science Self-Assessment - Google Chrome x
i starttest.com/ITDVersions/19.3.0.1/ITDStart.aspx?SVC=54cd3e5f-e4bf-49d0-8371-ba261251f2a1

Exam Section 3: Item 8 of 50 National Board of Medical Examiners Time Remaining:


• Mark Comprehensive Basic Science Self-Assessment 4 hr 58 min 3 sec

8. A 56-year-old man who is admitted to the hospital for treatment of pneumonia is found to have advanced lung cancer. His respiratory status improves with intravenous antibiotic
therapy. Before therapy is completed , he refuses palliative radiation therapy and wants to be discharged. He states that he is all alone, that no one cares about him, and that he does
not want to waste someone else's money for treatment. He promises that he will return in 6 months after he makes money from selling his invention that will cure arthritis. Which of
the following is the most appropriate next step?

A) Arrange for ambulatory administration of antibiotics


B) Determine whether the patient has decision-making capacity
u C) Institute home-visit care
U D) Seek a court order to force the patient to stay in the hospital
E) Transfer the patient to a psychiatric outpatient facility

B. Page 264/276.
This is a question of autonomy. We must respect the patients choice to not accept medical care.
However, he does not seem psychiatrically stable as he mentioned he has an invention that will cure arthritis.

Ambulatory administration of antibiotics would be forcing treatment he does not want.


Home-visit care is also something he has not asked for.
Transferring to the psych outpatient facility is unnecessary and extra.
Psych transfer is rarely dont when a patient is extremely depressed, suicidal, or expressing schizophrenic/mood disorders.

, ~ ~ p, ,.,,.
Previous Next Lab Values Calculator Review Help Pause
hi Comprehensive Basic Science Self-Assessment - Google Chrome x
i starttest.com/ITDVersions/19.3.0.1/ITDStart.aspx?SVC=54cd3e5f-e4bf-49d0-8371-ba261251f2a1

Exam Section 3: Item 9 of 50 National Board of Medical Examiners Time Remaining:


• Mark Comprehensive Basic Science Self-Assessment 4 hr 57 min 58 sec

9. A 68-year-old man comes to the physician because he is concerned about changes in his sexual performance during the past year. He noticed the changes when he started dating a
40-year-old woman after 25 years of living alone. "My orgasms are shorter and less intense than when I was married. I have to wait 2 or 3 hours before having sex again." His pulse is
72/min and regular, and blood pressure is 138/78 mm Hg. Physical examination shows hair growth on the tops of the feet and toes. Dorsalis pedis and posterior tibial pulses are +2
bilaterally, and his capillary refill time is 1 to 2 seconds. There is symmetric enlargement of the prostate without nodules. Laboratory studies show a hemoglobin A 1c of 5.5%, a serum
glucose concentration of 121 mg/dl, and a serum prostate-specific antigen concentration of 4.5 ng/ml (N<4 ). Which of the following is the most likely cause of this patient's changes
in sexual function?
F. Page 268/280.
A) Atherosclerosis
B) Benign prostatic hyperplasia VERY tricky question that truly tests your understanding of deriving information from each sentence.
C) Diabetic neuropathy The patient is doing well! No diabetes, no atherosclerosis or other arterial inflammation, and no concerns.
() D) Prostate cancer The patient has a very minimal increase in his PSA and some growth of his prostate, but he has not
u E) Psychogenic erectile dysfunction complained about issues regarding urination. It definitely is not prostate cancer either (no nodules).
U F) Normal aging
A- False. Dorsalis pedis and pulses are normal.
B- Possibly but not the cause of his sexual dysfunction.
C- False, not diabetic.
D- False, no nodules or metastatic symptoms.
E- False, he is having erections and having sex, but he just does not find his orgasms pleasing.
F- Correct. Normal aging causes men to have a weaker ejaculation and longer refractory period.

, ~ ~ p , ,.,,.
Previous Next Lab Values Calculator Review Help Pause
hi Comprehensive Basic Science Self-Assessment - Google Chrome x
i starttest.com/ITDVersions/19.3.0.1/ITDStart.aspx?SVC=54cd3e5f-e4bf-49d0-8371-ba261251f2a1

Exam Section 3: Item 10 of 50 National Board of Medical Examiners Time Remaining:


• Mark Comprehensive Basic Science Self-Assessment 4 hr 57 min 51 sec

10. A public health consultant is contacted by a health maintenance organization for recommendations about primary health prevention techniques for a population of women ages 30 to
40 years. Which of the following is the most appropriate primary preventative recommendation for this group?

CJ A) Abstinence from alcohol


B) Low-carbohydrate diet
C) Regular exercise
U D) HIV testing every 6 months
E) Annual mammography

C. Page 269/281.
This question is about primary prevention.
Examples include health promotion through vaccinations, exercise, diet, and smoking cessation.
Women ages 30-40 need to invest their time into weight-bearing exercises so that they can prepare for the skeletal issues revolving around menopause.

B and C would be relevant to an overweight group but that is irrelevant here.


A would be for groups known to fall into alcoholism.
D is critical for heavily sexually active teens. On top of that, "testing" for disease is a form of screening, which is for secondary prevention.
The same goes for annual mammography, which is a screening for breast cancer.

, ~ ~ p , ,.,,.
Previous Next Lab Values Calculator Review Help Pause
hi Comprehensive Basic Science Self-Assessment - Google Chrome x
i starttest.com/ITDVersions/19.3.0.1/ITDStart.aspx?SVC=54cd3e5f-e4bf-49d0-8371-ba261251f2a1

Exam Section 3: Item 11 of 50 National Board of Medical Examiners Time Remaining:


• Mark Comprehensive Basic Science Self-Assessment 4 hr 57 min 46 sec

11. A 17-year-old girl comes to the physician because she has never had a menstrual period. She is not sexually active. She is 180 cm(5 ft 11 in) tall and weighs 50 kg (110 lb); BMI is
15 kg/m2. Breast development is Tanner stage 5, and pubic and axillary hair development is Tanner stage 1. Pelvic examination shows a blind vaginal pouch. Which of the following
is most likely to be found on further testing?

A) Decreased serum estrogen concentration


B) Decreased serum luteinizing hormone concentration
C) Hyperplastic adrenal cortices
D) Increased serum testosterone concentration
E) Streak ovaries
F) Uterus

D. Page 625/703. Sexual differentiation on page 608/684.


This patient most likely has the X-linked recessive androgen insensitivity syndrome (AIS).
This patient is probably a male (46XY) with a disorder of sexual development.
The blind ending vagina means that the lower vagina (external) developed, but the upper vagina (internal) did not.
The upper vagina develops from the Mullerian duct in the absence of MIF. This patient most likely has testes (in the labias).
The testes make MIF and cause the mullerian duct to disintegrate.
Testosterone, estrogen, and LH will be high.

The factor differentiating this from mullerian agenesis is the lack of pubic and axillary hair in AIS.

, ~ ~ p , ,.,,.
Previous Next Lab Values Calculator Review Help Pause
hi Comprehensive Basic Science Self-Assessment - Google Chrome x
i starttest.com/ITDVersions/19.3.0.1/ITDStart.aspx?SVC=54cd3e5f-e4bf-49d0-8371-ba261251f2a1

Exam Section 3: Item 12 of 50 National Board of Medical Examiners Time Remaining:


• Mark Comprehensive Basic Science Self-Assessment 4 hr 57 min 38 sec

12. A 50-year-old woman is admitted to the hospital for management of an acute exacerbation of heart failure. She has been admitted to the hospital more than eight times during the
past year for similar episodes. On admission, intravenous administration of a diuretic is begun, and her fluid intake is restricted to 2 L daily. Her temperature is 36.3°C (97.3°F), pulse
is 88/min, respirations are 20/min, and blood pressure is 140/72 mm Hg. Bilateral basilar crackles are heard on pulmonary examination. There is pitting edema of the lower
extremities. Her fluid balance and weight are closely monitored. Two days later, the patient's weight and clinical status are unchanged. The nurse finds the patient in the bathroom
drinking directly from the faucet and holding a full pitcher of water, despite having already reached her fluid limit for the day. Which of the following is the most likely explanation for
this patient's lack of clinical improvement?

A) Antisocial personality disorder


B) Conversion disorder
C) Diabetes insipidus
() D) Factitious disorder
u E) Inappropriate secretion of ADH (vasopressin)

D. Page 554/618.
Awkward right? I always think of this as "She is making it a fact that she has a disorder."
Factitious disorder. Symptoms are intentional, motivation is unconscious. The patient just wants to be sick and get sympathy.

, ~ ~ p , ,.,,.
Previous Next Lab Values Calculator Review Help Pause
hi Comprehensive Basic Science Self-Assessment - Google Chrome x
i starttest.com/ITDVersions/19.3.0.1/ITDStart.aspx?SVC=54cd3e5f-e4bf-49d0-8371-ba261251f2a1

Exam Section 3: Item 13 of 50 National Board of Medical Examiners Time Remaining:


• Mark Comprehensive Basic Science Self-Assessment 4 hr 57 min 35 sec

13. A 65-year-old woman with well-controlled type 2 diabetes mellitus comes to the physician for a follow-up examination. At her last office visit 1 year ago, physical examination and
laboratory studies showed no abnormalities. Her temperature is 37.2°C (99°F), pulse is 82/min, respirations are 18/min, and blood pressure is 135/80 mm Hg. Physical examination
shows no other abnormalities. Laboratory studies show:
Hemoglobin 11.2 g/dl
Hematocrit 33%
Serum
Urea nitrogen 30 mg/dl
Creatinine 2.1 mg/dl

Abdominal ultrasonography shows decreased size of both kidneys. MR aortography shows bilateral proximal renal artery stenoses. Which of the following is the most likely
diagnosis?

A) Atherosclerosis
B) Congenital renal artery hypoplasia
U C) Fibromuscular dysplasia
D) Takayasu arteritis
U E) Temporal arteritis

A. Page 298/323 for fibromuscular dysplasia and 2 pages down discusses atherosclerosis.
There is a lot to work with in this question as well.
She has bilateral proximal renal artery stenosis.
You can cancel out choices B D and E. B is unreal. D and E affect large proximal vessels of the ascending aorta.

Atherosclerosis mainly affects the PROXIMAL 1/3 of renal arteries and is the #1 cause of renal artery stenosis.
Fibromuscular dysplasia affects the DISTAL 2/3 of renal arteries and is the #2 cause of renal artery stenosis.
This patient was fine in her previous visit 1 year ago, which also begs to think that this year she might have developed atherosclerosis.
Fibromuscular dysplasia usually occurs in women of child-bearing age and this patient is well into menopause.

, ~ ~ p , ,.,,.
Previous Next Lab Values Calculator Review Help Pause
hi Comprehensive Basic Science Self-Assessment - Google Chrome x
i starttest.com/ITDVersions/19.3.0.1/ITDStart.aspx?SVC=54cd3e5f-e4bf-49d0-8371-ba261251f2a1

Exam Section 3: Item 14 of 50 National Board of Medical Examiners Time Remaining:


• Mark Comprehensive Basic Science Self-Assessment 4 hr 57 min 29 sec

14. A 59-year-old man has repeated episodes of gouty arthritis unresponsive to preventive therapy. Which of the following drugs is most likely to increase uric acid excretion in this man?

A) Acetylsalicylic acid
U B) Colchicine
U C) Ketorolac
D) Penicillin
E) Probenecid

E.
Very nice question with a good take home lesson. Dosage matters.
You can remove ketorolac (an NSAID) and penicillin (an antibiotic) from the choices.
Ketorolac would help but help as it is a good NSAID and salicylate for acute gout but must be given in high dose for urinary excretion of urate.
Same for choice A, they didnt say high or low dose so that makes it hard.

Probenecid is a medication for chronic gout and it inhibits reabsorption of uric acid in the PCT so its perfect for what theyre asking for.
The question mentions his current preventive therapy is not working (maybe hes on allopurinol).
So this would be his new preventive therapy.

, ~ ~ p , ,.,,.
Previous Next Lab Values Calculator Review Help Pause
hi Comprehensive Basic Science Self-Assessment - Google Chrome x
i starttest.com/ITDVersions/19.3.0.1/ITDStart.aspx?SVC=54cd3e5f-e4bf-49d0-8371-ba261251f2a1

Exam Section 3: Item 15 of 50 National Board of Medical Examiners Time Remaining:


• Mark Comprehensive Basic Science Self-Assessment

15. A 35-year-old African American man comes to the physician for a routine examination. He recently read a pamphlet at a health fair on the importance of screening for skin cancer.
He enjoys sailing and usually goes out on his boat every weekend when the weather is nice. He does not use sunscreen , but he states that he does not "burn." Physical examination
shows no abnormalities. This patient is at increased risk for melanoma at which of the following locations?

A) Back
B) Chest
C) Forehead
D) Palms D. Page 473/531 of FA2019.
E) Scalp
Recurring theme. NBME has asked multiple times about melanoma occurring in the palms/soles of AA patients.

https://www.aimatmelanoma.org/melanoma-risk-factors/melanoma-in-people-of-color/acral-lentiginous-melanoma-alm/
Acral lentiginous melanoma is the melanoma on the palms, soles, and nail beds.
It is uncommon but it is the most common melanoma in people of darker skin or Asian descent.

Melanin in darkly pigmented skin can provide an SPF (sun protective factor) of 1.5 - 4.
Amount of UV light getting through is 1/SPF, so a SPF of 4 reduces UV radiation by 75%.
So the areas without this protective feature are most at risk (palms/soles).

, ~ ~ p , ,.,,.
Previous Next Lab Values Calculator Review Help Pause
hi Comprehensive Basic Science Self-Assessment - Google Chrome x
i starttest.com/ITDVersions/19.3.0.1/ITDStart.aspx?SVC=54cd3e5f-e4bf-49d0-8371-ba261251f2a1

Exam Section 3: Item 16 of 50 National Board of Medical Examiners Time Remaining:


• Mark Comprehensive Basic Science Self-Assessment 4 hr 57 min 18 sec

16. A patient with cancer who is being treated with high-dose chemotherapy has severe bone marrow suppression. Which of the following cytokines is most likely to be beneficial for the
bone marrow suppression?

CJ A) Granulocyte colony-stimulating factor


B) Interferon-a
Page 121.
C) lnterferon-y
Question 7 block 2 here was also on this same topic and had the same answer.
U D) lnterleukin-2 (IL-2) GMCSF will help with the bone marrow suppression.
E) lnterleukin-4 (IL-4)
F) lnterleukin-6 (IL-6) For revision, you should know exactly what the rest of these are for.
G) Transforming growth factor-B
H) Tumor necrosis factor INFa is used for chronic hep C, hep B, and RCC. (Think abc arcc).
INFb is used for multiple sclerosis while INFg is used for CGD and IL12 receptor deficiency.

IFNg and IL12 cause differentiation into Th1 cells.


IFNg also stimulates macrophages.

IL2 and IL4 cause differentiation into TH2 cells. (page 101).
IL4 also causes class switching to IgE and IgG.
IL6 is for acute phase reactants (Hot T Bone stEAK).

TGFb and IL10 attenuate the immune response and work with Tregs.

TNFa is for cachexia and maintenance of granulomas.

, ~ ~ p , ,.,,.
Previous Next Lab Values Calculator Review Help Pause
hi Comprehensive Basic Science Self-Assessment - Google Chrome x
i starttest.com/ITDVersions/19.3.0.1/ITDStart.aspx?SVC=54cd3e5f-e4bf-49d0-8371-ba261251f2a1

Exam Section 3: Item 17 of 50 National Board of Medical Examiners Time Remaining:


• Mark Comprehensive Basic Science Self-Assessment 4 hr 57 min 12 sec

17. The breakdown of dipeptides and tripeptides to free amino acids takes place primarily in which of the following areas in the gastrointestinal tract?

A) Intestinal mucosa
A. Page 367/408.
U B) Lumen of the duodenum NBME trash strikes again.
U C) Lumen of the large intestine So the duodenum is where pancreatic proteaseses like trypsin and chymotrypsin break down proteins to peptides.
D) Lumen of the stomach These peptides get further broken down into amino acids in the brush border of the intestines
E) Mouth

, ~ ~ p , ,.,,.
Previous Next Lab Values Calculator Review Help Pause
hi Comprehensive Basic Science Self-Assessment - Google Chrome x
i starttest.com/ITDVersions/19.3.0.1/ITDStart.aspx?SVC=54cd3e5f-e4bf-49d0-8371-ba261251f2a1

Exam Section 3: Item 18 of 50 National Board of Medical Examiners Time Remaining:


• Mark Comprehensive Basic Science Self-Assessment 4 hr 57 min 6 sec

18. A 55-year-old woman comes to the physician because of a 3-month history of difficulty using her hands and a 3-week history of muscle cramps. She has had a 9-kg (20-lb) weight
loss during the past 5 weeks. Neurologic examination shows tongue fasciculations, and lower extremity weakness and atrophy. Sensory examination is normal. A lesion at which of
the following sites is the most likely cause of these findings?

A) Cerebral white matter


B) Corticospinal tract in the spinal cord
C) Internal capsule
D) Lower motoneurons
E) Peripheral nerves

D. Page 517-518/577 discusses motor neuron signs and spinal lesions.


This patient has fasciculations, lower extremity weakness, and atrophy. Sensory stuff is fine. These are signs of LMN lesions.
This could be ALS but the question is not asking for a specific diagnosis.
This question does a good job testing what you understand about various parts of the CNS.

Choice A-- The cerebral white matter is full of myelin (lipid) and forms the deep parts of the brain in between
grey matter such as the basal ganglia and brainstem nuclei.
It serves to help coordinate communication between different brain regions. Multiple sclerosis affects white matter.

Choice B-- In contrast, gray matter comprises the main components of the brain (pinkish due to capillaries).
Composed of neuronal cell bodies, dendrites, glial cells, and synapses.
The anterior grey column is for motor neurons (pyramidal tract so corticospinal + corticobulbar),
while the posterior grey column is for sensory neurons (dorsal and spinothalamic).

C-- The internal capsule is a high-yield part of the brain. It is related to UMN lesions (q on my pdf page 577).

D-- Peripheral nerves would have sensory involvement as well.

, ~ ~ p , ,.,,.
Previous Next Lab Values Calculator Review Help Pause
hi Comprehensive Basic Science Self-Assessment - Google Chrome x
i starttest.com/ITDVersions/19.3.0.1/ITDStart.aspx?SVC=54cd3e5f-e4bf-49d0-8371-ba261251f2a1

Exam Section 3: Item 19 of 50 National Board of Medical Examiners Time Remaining:


• Mark Comprehensive Basic Science Self-Assessment 4 hr 57 min 1 sec

19. A 4 7-year-old man comes to the physician because of recurrent episodes of joint pain during the past 3 years. He
describes these as abrupt in onset and involving principally his ankles or knees, usually only one joint at a time. He does
not take any medications. Physical examination shows yellow-white nodules at the tips of several fingers , as well as over
the soles of the feet. His left knee is warm, tender, and swollen with dusky, erythematous overlying skin. A photomicrograph
of joint fluid aspirate is shown. The crystals shown are most likely composed of which of the following?

U A) Basic calcium phosphate (hydroxyapatite)


B) Calcium pyrophosphate dihydrate
u C) Cholesterol
D) Corticosteroid ester
E) Methylmethacrylate
F) Monosodium urate

F. Page 459/517.
This is like looking at a black and white image of the urate of gout.
You must remember that the crystals in pseudogout are rhomboid.
Plus the question explained the tophi this patient is getting, which further indicates gout.

Needle shaped crystals = gout = urate. Done.

, ~ ~ p , ,.,,.
Previous Next Lab Values Calculator Review Help Pause
hi Comprehensive Basic Science Self-Assessment - Google Chrome x
i starttest.com/ITDVersions/19.3.0.1/ITDStart.aspx?SVC=54cd3e5f-e4bf-49d0-8371-ba261251f2a1

Exam Section 3: Item 20 of 50 National Board of Medical Examiners Time Remaining:


• Mark Comprehensive Basic Science Self-Assessment 4 hr 56 min 55 sec

20. A 28-year-old man with chronic hepatitis C comes to the physician because of fever and progressive shortness of breath during the past 2 days. His temperature is 39.3°C (102.?°F),
pulse is 110/min, respirations are 28/min, and blood pressure is 120/70 mm Hg. Physical examination shows wasting and intercostal retractions. Crackles are heard over both lung
fields. A chest x-ray shows diffuse hazy infiltrates. His leukocyte count is 2000/mm 3 (90% segmented neutrophils, 5% lymphocytes, and 5% monocytes). To help explain the cause of
his illness, this patient should be tested for which of the following other chronic viral infections?

A) Cytomegalovirus D. Page 175/185.


B) Epstein-Barr virus First thing is that the patient has an infection. It seems like a very severe lung infection for the past 2 days.
u C) Hepatitis D The most notable thing is his leukocyte count is low and the breakdown is not distributed uniformly.
U D) HIV 90% neutrophils but he seems to have lost lymphocytes and monocytes. 5% of 2000 is 100. Very low CD4 count.
E) Human herpes virus-6 Could even be pneumocystis jirovecii.
The HepC is related to IV drug abuse, which can also lead to HIV infection.

, ~ ~ p , ,.,,.
Previous Next Lab Values Calculator Review Help Pause
hi Comprehensive Basic Science Self-Assessment - Google Chrome x
i starttest.com/ITDVersions/19.3.0.1/ITDStart.aspx?SVC=54cd3e5f-e4bf-49d0-8371-ba261251f2a1

Exam Section 3: Item 21 of 50 National Board of Medical Examiners Time Remaining:


• Mark Comprehensive Basic Science Self-Assessment 4 hr 56 min 43 sec

21 . A 73-year-old woman has had easy fatigability for 2 years. She had an ileal resection for Crohn disease 10 years ago. She has
extremely pale oral mucosa. A peripheral blood smear is shown . Which of the following is the most likely mechanism of this
disorder?

A) Decreased serum transferrin concentration


B) Deficiency of glucose 6-phosphate dehydrogenase
U C) Deficiency of protoporphyrinogen oxidase
0 D) Failure of conversion of N 5-methyltetrahydrofolate to tetrahydrofolate

D. Pg 376/418 for Crohn and page 409/453 for heme.


Crohns disease has many issues (ulcers, fistulas, bleeding) but it
does not cause IDA. UC has bloody diarrhea and also rarely has
IDA but it is possible.

The peripheral smear is showing megaloblastic anemia.


There are many macrocytic RBC with pale centers.

Choices B and C are irrelevant.

Choice D, the correct answer, is explained by the pathway here.


Vitamin B12 is necessary for convertiing 5M-THF into THF, the
active form. Without B12 this conversion will not occur and you
will not get THF to be able to make more DNA. So this patient
must have B12 deficiency due to involvement of injury to the
terminal ileum, which is common in Crohn.

, ~ ~ p , ,.,,.
Previous Next Lab Values Calculator Review Help Pause
hi Comprehensive Basic Science Self-Assessment - Google Chrome x
i starttest.com/ITDVersions/19.3.0.1/ITDStart.aspx?SVC=54cd3e5f-e4bf-49d0-8371-ba261251f2a1

Exam Section 3: Item 22 of 50 National Board of Medical Examiners Time Remaining:


• Mark Comprehensive Basic Science Self-Assessment 4 hr 56 min 9 sec

22. A 35-year-old woman undergoes a left oophorectomy because of a 5 x 5-cm ovarian mass. During this procedure, which of the following structures is at greatest risk for injury when
dividing the suspensory ligament?

CJ A) Left internal iliac artery D. Page 611/687.


B) Left renal vein This is a very easy question. There are two cases that can occur, an oophorectomy or a hysterectomy.
C) Umbilical artery
During an oophorectomy (as in our patient), the suspensory infundibulopelvic ligament that suspends the ovarian vessels is ligated.
During a hysterectomy, the cardinal transverse cervical ligament holding the uterine vessels is ligated.
U D) Ureter
E) Uterine artery
Both cases carry a risk of ureteral injury that often presents with unilateral flank pain (if bilateral, patient cant void either).

, ~ ~ p , ,.,,.
Previous Next Lab Values Calculator Review Help Pause
hi Comprehensive Basic Science Self-Assessment - Google Chrome x
i starttest.com/ITDVersions/19.3.0.1/ITDStart.aspx?SVC=54cd3e5f-e4bf-49d0-8371-ba261251f2a1

Exam Section 3: Item 23 of 50 National Board of Medical Examiners Time Remaining:


• Mark Comprehensive Basic Science Self-Assessment 4 hr 55 min 53 sec

23. A 45-year-old man comes to the physician because of progressive daytime sleepiness and fatigue during the past 2 years. He says that he frequently wakes up in the night and has
been told he snores loudly. He is 180 cm (5 ft 11 in) tall and weighs 159 kg (350 lb); BMI is 49 kg/m 2. Physical examination shows peripheral edema below the knees. A loud S 2 is
heard on auscultation of the chest. This patient's symptoms are most likely caused by an episodic decrease in which of the following?

A) Lung volume
B) Oxygen saturation
C) Pulmonary vascular resistance
D) Systemic vascular resistance
E) Venous return

B. Page 665/761.
The patient fits the presentation of someone who has sleep apnea.
I always think of sleep apnea as "you arent even breathing." This is one reason why this is under respiratory, because it causes hypoxia.
Try snoring for an hour straight while youre up and about on your day.
You will run out of breath and get weak and tired. Apnea is a real issue. Patients just dont notice because theyre asleep.

, ~ ~ p , ,.,,.
Previous Next Lab Values Calculator Review Help Pause
hi Comprehensive Basic Science Self-Assessment - Google Chrome x
i starttest.com/ITDVersions/19.3.0.1/ITDStart.aspx?SVC=54cd3e5f-e4bf-49d0-8371-ba261251f2a1

Exam Section 3: Item 24 of 50 National Board of Medical Examiners Time Remaining:


• Mark Comprehensive Basic Science Self-Assessment 4 hr 55 min 44 sec

24. A 9-year-old boy who was adopted from an African country 2 weeks ago is brought to the physician because of difficulty walking and fatigue. Femoral, popliteal, posterior tibial , and
dorsalis pedis pulses are equal and full. Neurologic examination shows weakness of plantar dorsiflexion and foot intrinsics. He has a broad-based, ataxic gait. Sensation to light
touch is decreased over the lower extremities; vibration sense is absent. Laboratory studies show:
Hemoglobin decreased
Hematocrit decreased
Mean corpuscular hemoglobin normal
Mean corpuscular hemoglobin concentration normal
Mean corpuscular volume normal
Platelet count normal
Serum lactate dehydrogenase increased

This patient most likely has a deficiency of which of the following vitamins?

A) 8 2 (riboflavin)
U 8) 8 12 (cobalamin)
E. Page 70 FA2019.
First of all. WTF is plantar dorsiflexion? NBME Trash.
C) C
() D) D
This patient has neurological deficits but without megaloblastic macrocytic anemia.
E) E This patient has anemia that is most likely due to hemolysis.
Vitamin E is an antioxidant that protects RBCs from free radical damage.
Deficiency can lead to hemolytic anemia and demyelination of the posterior columns and spinocerebellar tract.

Often confused with vitamin b12 deficiency which is similar in presentation.


Deficiency of b12 leads to macrocytic megaloblastic anemia along with subacute combined degeneration
involving the lateral corticospinal tract as well.

, ~ ~ p , ,.,,.
Previous Next Lab Values Calculator Review Help Pause
hi Comprehensive Basic Science Self-Assessment - Google Chrome x
i starttest.com/ITDVersions/19.3.0.1/ITDStart.aspx?SVC=54cd3e5f-e4bf-49d0-8371-ba261251f2a1

Exam Section 3: Item 25 of 50 National Board of Medical Examiners Time Remaining:


• Mark Comprehensive Basic Science Self-Assessment 4 hr 55 min 38 sec

25. A 13-year-old boy is brought to the physician by his mother for an examination prior to participating on the school bowling team. His mother says that he has asked to be excused
from gym class this year because of chest swelling that began 6 months ago. He is reluctant to take off his shirt. Physical examination shows bilateral , 1-cm, mildly tender, rubbery
nodules palpable under the areolar region. Sexual development is Tanner stage 3. After empathizing with the patient about how he may be embarrassed by his condition , it is most
appropriate for the physician to state which of the following?

A) "Most boys experience this between the ages of 9 and 10 years."


B) "This will typically resolve within the next 12 to 18 months."
u C) "We need to do a few blood tests to rule out hormonal disorders."
U D) "You'll gradually get more comfortable with your own body."
E) "You're the only one who really notices it."

B. Ethics
This is just typical gynecomastia due to puberty related spikes in testosterone and estrogen.
Choice A is wrong since that is not when puberty occurs.
Choice C is unnecessary.
Choice D is not something you can tell someone. People will get comfortable when they get comfortable. Some never do.
Choice E is a lie lol.

Although I must admit I would hate to tell a little boy hes gonna have big boobs for another year and a half...

, ~ ~ p , ,.,,.
Previous Next Lab Values Calculator Review Help Pause
hi Comprehensive Basic Science Self-Assessment - Google Chrome x
i starttest.com/ITDVersions/19.3.0.1/ITDStart.aspx?SVC=54cd3e5f-e4bf-49d0-8371-ba261251f2a1

Exam Section 3: Item 26 of 50 National Board of Medical Examiners Time Remaining:


• Mark Comprehensive Basic Science Self-Assessment 4 hr 55 min 32 sec

26. A 3-year-old boy with AIDS develops giant cell pneumonia 3 months after exposure to an unimmunized cousin who had a morbilliform rash , conjunctivitis, rhinitis, and Koplik spots.
The virus responsible for the pneumonia has which of the following types of genomes?

CJ A) Double-stranded DNA
B) Negative-stranded RNA
C) Positive-stranded RNA
U D) Single-stranded DNA

B. Page 170/179.
The patient has AIDS which is important because this patient cannot be immunized, so they are susceptible to everything.
The cousin is unimmunized (maybe theyre from a 3rd world country) so this cousin can have any disease (MMR, TDaP, etc).
The koplik spots are practically pathognomonic for measles (a paramyxovirus, remember sketchy's paranormal mixer).

, ~ ~ p , ,.,,.
Previous Next Lab Values Calculator Review Help Pause
hi Comprehensive Basic Science Self-Assessment - Google Chrome x
i starttest.com/ITDVersions/19.3.0.1/ITDStart.aspx?SVC=54cd3e5f-e4bf-49d0-8371-ba261251f2a1

Exam Section 3: Item 27 of 50 National Board of Medical Examiners Time Remaining:


• Mark Comprehensive Basic Science Self-Assessment 4 hr 55 min 25 sec

27. A 17-year-old girl is brought to the hospital by ambulance 45 minutes after sustaining a closed-head injury during a motor vehicle collision. She is in a coma. Physical examination
shows multiple bleeding lacerations over the head. An MRI of the brain shows a 4-cm, right-sided intracranial hematoma with possible brain-stem herniation. Emergency craniotomy
is indicated, but several attempts to contact the patient's parents to obtain permission for the procedure are unsuccessful. The physician decides to proceed with the operation
without permission. This decision by the physician is most consistent with which of the following ethical principles?

A) Beneficence
B) Common good
u C) Integrity
U D) Nonmaleficence
E) Respect for autonomy

A. Page 264/276.
This patient is in need of this life saving surgery.
Beneficence is acting in the benefit of the patient.

Common good and integrity are just filler answer choices.


Autonomy involves listening to the patient.
Nonmaleficence involves not hurting the patient. An example for this scenario would be to NOT do a surgery that might possibly kill the patient.

, ~ ~ p , ,.,,.
Previous Next Lab Values Calculator Review Help Pause
hi Comprehensive Basic Science Self-Assessment - Google Chrome x
i starttest.com/ITDVersions/19.3.0.1/ITDStart.aspx?SVC=54cd3e5f-e4bf-49d0-8371-ba261251f2a1

Exam Section 3: Item 28 of 50 National Board of Medical Examiners Time Remaining:


• Mark Comprehensive Basic Science Self-Assessment 4 hr 55 min 20 sec

28. An obese 57-year-old man comes to the physician for a routine examination. He has smoked 2Y2 packs of cigarettes daily for 40 years. Physical examination shows mild dyspnea. A
chest x-ray shows a mass in one lobe of the right lung. Examination of a biopsy specimen obtained on bronchoscopy shows squamous metaplasia of the bronchial mucosa. Which
of the following best describes the changes in this patient's bronchial mucosa?

A) Activation of c-Fos resulting in increased proliferation of atypical squamous epithelial cells


B) Chronic irritation leading to decreased blood flow and atrophy of the mucosal lining
C) Irreversible change in the basal cell layer leading to development of abnormal squamous epithelial cells
D) Respiratory epithelial hyperplasia with compression into a flattened squamous pattern
E) Normal ciliated columnar epithelium replaced by normal squamous epithelium

E. Page 206/216.
They really tried to make the answer choices come off extremely intimidating.
But overall its clear that the patient has damage from the stressor; 40 years of extensive smoking.
Metaplasia is a normal response to stressors. The mass in the lobe of the right lung could be from the smoking.
This patient might even have GERD related to his obesity.

Regardless, they said he has squamous metaplasia which means that the ciliated columnar epithelium is becoming stratified sqamous epithelium.
The columnar cells are normal, and they get replaced by normal looking squamous cells. The fact that this is happening is abnormal but it is reversible.
Good question to test on the understanding of pathology.

, ~ ~ p , ,.,,.
Previous Next Lab Values Calculator Review Help Pause
hi Comprehensive Basic Science Self-Assessment - Google Chrome x
i starttest.com/ITDVersions/19.3.0.1/ITDStart.aspx?SVC=54cd3e5f-e4bf-49d0-8371-ba261251f2a1

Exam Section 3: Item 29 of 50 National Board of Medical Examiners Time Remaining:


• Mark Comprehensive Basic Science Self-Assessment 4 hr 55 min 14 sec

29. A 58-year-old man comes to the physician because of a 3-day history of progressive malaise, increased urinary frequency, dribbling of urine, a feeling of incomplete bladder
emptying, and deep, dull pelvic pain. He has not had scrotal or testicular pain or blood in his urine, and there has been no trauma. He now urinates once hourly. He has a history of
mild asthma well controlled with inhaled albuterol as needed. He takes no other medications. He does not smoke, drink alcohol, or use illicit drugs. He is sexually active with one
female partner. The penis and scrotum appear normal. Digital rectal examination shows an enlarged, exquisitely tender prostate. Which of the following infectious agents is the most
likely cause of these findings?

A) Chlamydia trachomatis
B) Escherichia coli
C) Mumps virus
lJ D) Neisseria gonorrhoeae
U E) Ureaplasma urealyticum

B. Page 639/720.
Very nice question. The patient has urinary incontinence (dribbling of urine), prostatitis (tender prostate), and might even have a fever.
E coli is the #1 cause of a UTI but this question is about more than that.
E coli is also the #1 cause of acute bacterial prostatitis in older men (younger men would be C trachomatis or N gonorrhoeae along with sexual activity).

Dont think of picking C trachomatis or N gonorrhoeae without a urine culture being mentioned or something

, ~ ~ p , ,.,,.
Previous Next Lab Values Calculator Review Help Pause
hi Comprehensive Basic Science Self-Assessment - Google Chrome x
i starttest.com/ITDVersions/19.3.0.1/ITDStart.aspx?SVC=54cd3e5f-e4bf-49d0-8371-ba261251f2a1

Exam Section 3: Item 30 of 50 National Board of Medical Examiners Time Remaining:


• Mark Comprehensive Basic Science Self-Assessment 4 hr 55 min 9 sec

30. An 80-year-old woman , gravida 4, para 4 , comes to the office because of a 6-month history of decreased frequency of bowel movements. She now has one or two bowel
movements weekly. Increased fluid and dietary fiber intake has not resulted in relief. She also reports a sensation of incomplete evacuation of her bowels and having to manually
support the posterior vaginal wall in order to defecate. Colonoscopy at the age of 75 years showed extensive diverticula. Digital rectal examination shows no evidence of
gastrointestinal bleeding, fissure, mass, or hemorrhoids. Rectal tone and reflexes are normal. Which of the following is the most likely cause of this patient's symptoms?

A) Absence of ganglionic cells in the colon


B) Damage to the rectovaginal septum
u C) External anal sphincter spasm
U D) Inhibition of the gastrocolic reflex
E) Transmural mucosa! inflammation

B. Page 688 of my annotated FA.


This is a recurring theme in NBME.
This patient has a rectocele. The wall of the rectum is bulging towards the vagina.

When a rectocele becomes large, stool can become trapped within it, making it difficult to have a bowel
movement or creating a sensation of incomplete evacuation. Symptoms are usually due to stool trapping,
difficulty passing stool, and protrusion of the back of the vagina through the vaginal opening.
During bowel movements, women with large, symptomatic rectoceles may describe the need to put their
fingers into their vagina and push back toward the rectum to allow the stool to pass.
Rectoceles are more common in women who have delivered children vaginally.

, ~ ~ p , ,.,,.
Previous Next Lab Values Calculator Review Help Pause
hi Comprehensive Basic Science Self-Assessment - Google Chrome x
i starttest.com/ITDVersions/19.3.0.1/ITDStart.aspx?SVC=54cd3e5f-e4bf-49d0-8371-ba261251f2a1

Exam Section 3: Item 31 of 50 National Board of Medical Examiners Time Remaining:


• Mark Comprehensive Basic Science Self-Assessment 4 hr 55 min 4 sec

31. A 29-year-old man is brought to the emergency department because of a 1-hour history of bilateral jaw pain that began immediately after he tried to bite into a th ick double
cheeseburger. Physical examination shows excessive drooling and an inability to elevate the mandible. An x-ray of the skull shows bilateral anterior dislocation of the
temporomandibular joints. Reduction of the temporomandibular joints is recommended. Relaxation of which of the following muscles is most likely to facilitate this procedure?

A) Anterior belly of digastric


B) Buccinator
C) Lateral pterygoid
D) Levator veli palatini
E) Mylohyoid

C. Page 495/553 FA2019.


This patient has temporomandibular joint dysfunction, which is not a topic in FA.
However, this page does mention the muscles of mastication.
The other muscles listed are irrelevant and you should have a decent idea of where they are.

The levator veli palatini elevates the soft palate during swallowing to prevent food from entering the nasopharynx.

, ~ ~ p , ,.,,.
Previous Next Lab Values Calculator Review Help Pause
hi Comprehensive Basic Science Self-Assessment - Google Chrome x
i starttest.com/ITDVersions/19.3.0.1/ITDStart.aspx?SVC=54cd3e5f-e4bf-49d0-8371-ba261251f2a1

Exam Section 3: Item 32 of 50 National Board of Medical Examiners Time Remaining:


• Mark Comprehensive Basic Science Self-Assessment 4 hr 54 min 57 sec

32. A 3-month-old boy is brought to the physician because of yellow eyes and skin and weakness since birth. Physical examination shows jaundice, large fontanels, a flat midfacial area,
hypotonia, and hepatomegaly. Serum studies show:
Total bilirubin (mainly direct) increased
AST increased
ALT increased
Very-long-chain fatty acids increased

A liver biopsy specimen shows foamy, lipid-filled hepatocytes, necrosis, and absence of a specific organelle. This organelle is most likely which of the following?

u A) Golgi complex
B) Lysosomes
C) Mitochondria
u D) Peroxisomes
E) Smooth endoplasmic reticulum

D. Page 47 FA2019.
When you first see this question it should NOT ring a bell for a lysosomal storage disorder. The LSDs have neurological symptoms.
All the LSDs have involvement of other organs, histological findings, and involvement of many pathologies other than hepatic problems.
The lipid-filled hepatocytes were a bait to make you think of "lipid-laden macrophages" of Gaucher and Niemann Pick disease.

This patient has Zellweger syndrome, an autosomal recessive peroxisomal disorder due to mutated PEX genes critical for peroxisome function.
The presence of VLCFA is an indication of failure of beta oxidation, one of the key functions of peroxisomes.
Patients will have elevated VLCFAs and there is no treatment so death is imminent by age of 1 year.

, ~ ~ p , ,.,,.
Previous Next Lab Values Calculator Review Help Pause
hi Comprehensive Basic Science Self-Assessment - Google Chrome x
i starttest.com/ITDVersions/19.3.0.1/ITDStart.aspx?SVC=54cd3e5f-e4bf-49d0-8371-ba261251f2a1

Exam Section 3: Item 33 of 50 National Board of Medical Examiners Time Remaining:


• Mark Comprehensive Basic Science Self-Assessment 4 hr 54 min 52 sec

33. A 25-year-old man comes to the emergency department because of a 3-day history of abdominal cramps and diarrhea. He appears anxious. Physical examination shows
piloerection and diffuse abdominal tenderness. Neurologic examination shows no abnormalities. He is oriented to person, place, and time. The most likely cause of this patient's
condition is withdrawal from which of the following substances?

A) Alcohol D. Page 558/623.


B) Benzodiazepine This patient is presenting with opioid withdrawal.
C) Cocaine The fact that heroin is an opioid drug made from morphine is something that catches IMGs off-gaurd.
D) Heroin The "piloerection" is the giveaway for opioid withdrawal (cold turkey).
E) Marijuana Patients present with flu-like symptoms.

Mnemonic.
Opioids are for peace and euphoria.
Opioid intoxication puts you to sleep, so its dark, and pupils constrict.
Opioid withdrawal puts you in pain, so its fight/flight mode, and pupils dilate.

, ~ ~ p , ,.,,.
Previous Next Lab Values Calculator Review Help Pause
hi Comprehensive Basic Science Self-Assessment - Google Chrome x
i starttest.com/ITDVersions/19.3.0.1/ITDStart.aspx?SVC=54cd3e5f-e4bf-49d0-8371-ba261251f2a1

Exam Section 3: Item 33 of 50 National Board of Medical Examiners


• Mark Comprehensive Basic Science Self-Assessment Please Walt

34. A 35-year-old woman has difficulty urinating 2 days postpartum. She has a long history of asthma. Which of the following mechanisms is both an indication and a contraindication
for bethanechol in this patient?

A) r3-Adrenergic antagonism
B) Anticholinesterase action
C) Histaminergic stimulation
D) Nicotinic antagonism
E) Parasympathomimetic stimulation

E. Page 240/250.
Very nice and fair question! The patient has postpartum urinary retention. The increased progesterone can inhibit bladder muscle activity.
Bethanechol is a cholinomimetic that acts on M3 receptors to activate the bladder smooth muscle.
The M3 receptor also causes bronchoconstriction, which would be bad for the patient's asthma.

The other answer choices are not related to bethanechol.

, ~ ~ p , ,.,,.
Previous Next Lab Values Calculator Review Help Pause
hi Comprehensive Basic Science Self-Assessment - Google Chrome x
i starttest.com/ITDVersions/19.3.0.1/ITDStart.aspx?SVC=54cd3e5f-e4bf-49d0-8371-ba261251f2a1

Exam Section 3: Item 35 of 50 National Board of Medical Examiners Time Remaining:


• Mark Comprehensive Basic Science Self-Assessment 4 hr 54 min 36 sec

35. A 75-year-old man with a 10-year history of progressive renal failure comes to the physician for a follow-up examination. Laboratory studies show a serum urea nitrogen
concentration of 40 mg/dl, and a serum creatinine concentration of 3 mg/dl. Ultrasonography of the urinary tract shows a solitary hydronephrotic kidney and a dilated ureter. The
most likely cause of this patient's renal failure is an increase in which of the following?

A) Hydrostatic pressure in Bowman space


B) Hydrostatic pressure in glomerular capillaries
C) Hydrostatic pressure in peritubular capillaries
D) Oncotic pressure in peritubular capillaries
E) Oncotic pressure in renal tubules

A. Page 571/638 of FA2019.


This question is also very straightforward as long as you have a strong understanding of renal physiology.
The image on this page is critical. The thickness of the arrows pointing to or from the glomerulus determines their intensity.
Pgc is the strongest. It is the glomerular capillary hydrostatic pressure, which pushes fluid into the bowman space.
Pbs is the 2nd strongest, which pushes fluid into the glomerulus (our answer).

The oncotic pressures are weaker.


PIgc is the oncotic pressure inside the glomerulus and it pulls fluid from the bowman space.
PIbs is the weakest and it pulls fluid into the bowman space.

Our patient has a hydronephrotic kidney and a dilated ureter. This indicates there is blockade so urine is backing up.
Thus there is a ton of pressure up the ureter all the way up the collecting tubules and into the bowman's space!
Pretty simple and clear. This is the opposite flow of filtration so GFR decreases.

, ~ ~ p , ,.,,.
Previous Next Lab Values Calculator Review Help Pause
hi Comprehensive Basic Science Self-Assessment - Google Chrome x
i starttest.com/ITDVersions/19.3.0.1/ITDStart.aspx?SVC=54cd3e5f-e4bf-49d0-8371-ba261251f2a1

Exam Section 3: Item 36 of 50 National Board of Medical Examiners Time Remaining:


• Mark Comprehensive Basic Science Self-Assessment 4 hr 54 min 29 sec

36. A 55-year-old man is diagnosed with coronary artery disease. He begins treatment with 81-mg aspirin to prevent thrombus formation and vessel occlusion in areas of injured
vascular endothelium caused by atherosclerosis. Which of the following effects of aspirin on platelet function is most likely to decrease this patient's risk for thrombosis?

CJ A) Accelerated breakdown of vascular endothelial growth factors


B) Decreased adherence
C) Inhibition of glycoprotein Ilb/Illa receptors
U D) Potentiation of contraction by thrombosthenin
E) Release of calcium ions from the endoplasmic reticulum

B. Page 474/532.
ALERT- NBME Trash.
So you can definitely cancel out C because that is irrelevant to aspirin.
Choice E is also irrelevant to aspirin's mechanism. Same for choice A, aspirin does not break VEGF.
This leaves choices B and D.
Never heard of thrombosthenin so I chose B and this is BULLSHIT because platelet adherence and aggregation are DIFFERENT things.

Aspirin inhibits platelet aggregation and produces a mild bleeding defect by inhibiting COX enzymes.
Loss of TXAs messes up platelet aggregation. Not adherence. Get your shit straight NBME.

Also, for completeness, thrombasthenin is just a contractile protein inside platelets.

, ~ ~ p , ,.,,.
Previous Next Lab Values Calculator Review Help Pause
hi Comprehensive Basic Science Self-Assessment - Google Chrome x
i starttest.com/ITDVersions/19.3.0.1/ITDStart.aspx?SVC=54cd3e5f-e4bf-49d0-8371-ba261251f2a1

Exam Section 3: Item 37 of 50 National Board of Medical Examiners Time Remaining:


• Mark Comprehensive Basic Science Self-Assessment 4 hr 54 min 23 sec

37. A 35-year-old man is brought to the emergency department because he is disoriented and hallucinating. He has a 20-year history of alcoholism. On admission to the hospital, he has
a seizure. His blood pressure is 180/ 100 mm Hg. Serum potassium concentration is 2.5 mEq/L, and urine potassium concentration is 40 mEq/L. Alcohol withdrawal is suspected.
Which of the following is the most likely cause of the hypokalemia?

A) Catecholamine-mediated intracellular shifts of K +


B) Decreased dietary intake of K+
C) Decreased release of renin
D) Decreased urine flow rate
E) Increased muscle breakdown

A. Page 238/248.
Intricate question. I dont agree on the way this is being tested and it is quite unreal.
Not the way this will be tested on STEP 1.

Regardless, you should be able to cancel out B-E and then find some way A is even possible.
This depends on how much youve been studying and how close you are to taking step 1.

I discussed the g-protein linked second messengers and receptors in pharmacology a lot and ive always advised to revisit often.
This patient has alcohol withdrawal.
He had a seizure, which must have lead to an increase in catecholamines.
The catecholamines increased his BP.
His serum K is low but his urine K is normal, so the K must have shifted into cells.
Insulin is notorious for intracellular shift of K. So what happened here?

Well, beta 2 receptors increase insulin release. M3 also increases insulin release.
And alpha 2, the receptor that decreases everything, decreases insulin release.

, ~ ~ p , ,.,,.
Previous Next Lab Values Calculator Review Help Pause
hi Comprehensive Basic Science Self-Assessment - Google Chrome x
i starttest.com/ITDVersions/19.3.0.1/ITDStart.aspx?SVC=54cd3e5f-e4bf-49d0-8371-ba261251f2a1

Exam Section 3: Item 38 of 50 National Board of Medical Examiners Time Remaining:


• Mark Comprehensive Basic Science Self-Assessment 4 hr 54 min 18 sec

38. A 35-year-old man with small cell carcinoma of the lung has systemic hypertension and hypokalemia. The most likely cause of these findings is ectopic secretion of which of the
following hormones?

CJ A) ACTH
B) ADH (vasopressin)
C) Epinephrine
U D) Parathyroid hormone-related peptide
E) Vasoactive intestinal polypeptide

A. Page 669/768.
You will only miss this question if you are very new to step prep or have not revised respiratory and done UW.
The lung cancers are notorious for ectopic secretion, notably small cell (ACTH, ADH, and antibodies causing lambert eaton and more).
This patient must be secreting ACTH, which ramps up cortisol release and can also lead to more insulin release (which caused the hypokalemia).

, ~ ~ p , ,.,,.
Previous Next Lab Values Calculator Review Help Pause
hi Comprehensive Basic Science Self-Assessment - Google Chrome x
i starttest.com/ITDVersions/19.3.0.1/ITDStart.aspx?SVC=54cd3e5f-e4bf-49d0-8371-ba261251f2a1

Exam Section 3: Item 39 of 50 National Board of Medical Examiners Time Remaining:


• Mark Comprehensive Basic Science Self-Assessment 4 hr 54 min 11 sec

39. A 57-year-old man has had hoarseness and difficulty swallowing for 2 days. He is unable to elevate the right side of the palate. Which of the following labeled cranial nerves in the
photograph of the brain stem is most likely damaged?

F. Page 492/550.
These images are never perfect enough.
A = CN III
B = CN V
C = CN VI
D = CN VII
E = CN VIII
F = CN X

Remember the rule of 4's.


1-4 in midbrain
5-8 in pons
9-12 in medulla.
And 3, 4, 6 ,12 are medial.

Our patient's difficulty swallowing and hoarseness


are indicative of CN10 vagus injury, so that must
be F since it is the only one coming out of the
medulla. It also has a thick trunk since its big.

They have used this same image repeatedly. U A) B) C) D) E) F)

, ~ ~ p , ,.,,.
Previous Next Lab Values Calculator Review Help Pause
hi Comprehensive Basic Science Self-Assessment - Google Chrome x
i starttest.com/ITDVersions/19.3.0.1/ITDStart.aspx?SVC=54cd3e5f-e4bf-49d0-8371-ba261251f2a1

Exam Section 3: Item 40 of 50 National Board of Medical Examiners Time Remaining:


• Mark Comprehensive Basic Science Self-Assessment 4 hr 54 min 6 sec

40. A 2-year-old boy is brought to the physician because of chronic bacterial respiratory infections since birth. He is currently asymptomatic. Physical examination shows no
abnormalities. T- and B-lymphocyte counts and serum antibody concentrations are within the reference ranges. Natural killer cell count and function are normal. Analysis of cellular
expression of human leukocyte antigen by flow cytometry shows absence of class I MHC-expressing cells. A diagnosis of bare lymphocyte syndrome, type I, is made. This patient
most likely has mutations in the genes encoding which of the following?

A) Adenosine deaminase
B) Fas ligand (CD178)
u C) lnterleukin-2 (IL-2) receptor a chain (CD25)
U D) Peptide transporter (TAP)

D. Not in FA.
I had never heard of peptide transporter TAP.
I do know adenosine deaminase deficiency can cause SCID, which is irrelevant here.
Bare lymphocyte syndrome is a form of SCID, but the mechanism is different.
Fas ligand is related to apoptosis (pg 208/218 fa2019) and is of the TNF family, so that is irrelevant here.
And IL2 is for making Th2 cells and that is irrelevant here.
So I just picked D.

TAP deficiency is the BLS1. Symptoms can include recurrent bacterial infections of the respiratory tract and chronic skin lesions.
Why you would know this is beyond me. It is not in FA.

This is how NBME induces crippling depression in medical students who pay for an exam that comes with no explanation.

, ~ ~ p , ,.,,.
Previous Next Lab Values Calculator Review Help Pause
hi Comprehensive Basic Science Self-Assessment - Google Chrome x
i starttest.com/ITDVersions/19.3.0.1/ITDStart.aspx?SVC=54cd3e5f-e4bf-49d0-8371-ba261251f2a1

Exam Section 3: Item 41 of 50 National Board of Medical Examiners Time Remaining:


• Mark Comprehensive Basic Science Self-Assessment 4 hr 54 min 1 sec

41. A 50-year-old woman with HIV infection comes to the physician for a follow-up examination. For the past 6 months, she has been receiving antiretroviral therapy that includes the
nucleoside analogues zidovudine (AZT) and lamivudine (3TC), as well as the protease inhibitor nelfinavir. She is also receiving prophylaxis with pentamidine for pneumocystosis and
azithromycin for Mycobacterium avium complex. Her leukocyte count 2 weeks ago was 1200/mm 3 (50% segmented neutrophils). Her plasma HIV viral load remains undetectable.
Drug-induced bone marrow suppression is suspected. Which of the following drugs is the most likely cause?

A) Azithromycin
B) Lamivudine
U C) Nelfinavir
U D) Pentamidine
E) Zidovudine

E. Page 203/213.
NBME trash alert. All NRTI can cause BMS and they listed two..

HIV therapy is a common NBME topic and will be on your step as well.
The NRTIs have weird names that need to be revised often to memorize.
The special ones are tenofovir (T for nucleoTide, while others are all nucleosides), zidovudine (can be used in pregnancy), didanosine (pancreatitis), and
abacavir which is contraindicated in HLAB5701 patients.
It was wrong of them to list both zidovudine and lamivudine as answer choices because all of the NRTIs can cause bone marrow suppression.
However, "Zero bone marrow wiith Zidovudine" is a sketchy memorization mnemonic.

The NNRTIs cause rash and hepatotoxicity and efavirenzzz gives vivid dreams when you zzz sleep.
Protease inhibitors have various GI side effects and should not be taken with rifampin so take rifabutin instead if you have TB.
Integrase inhibitors can raise CK. And thats about it overall.

, ~ ~ p , ,.,,.
Previous Next Lab Values Calculator Review Help Pause
hi Comprehensive Basic Science Self-Assessment - Google Chrome x
i starttest.com/ITDVersions/19.3.0.1/ITDStart.aspx?SVC=54cd3e5f-e4bf-49d0-8371-ba261251f2a1

Exam Section 3: Item 42 of 50 National Board of Medical Examiners Time Remaining:


• Mark Comprehensive Basic Science Self-Assessment 4 hr 53 min 56 sec

42. A 65-year-old man comes to the physician because of difficulty sleeping for the past month. He says, "I have been awakening between 3 and 4 AM every morning and I can't get
back to sleep." He has impaired concentration and a decreased energy level. He used to read a lot and play cards with friends, but he has lost interest in these activities. He has lost
10 kg (22 lb) over the past 3 months, and he has had recurrent thoughts of death since having a myocardial infarction and undergoing a five-vessel coronary bypass 5 months ago.
His current medications include aspirin, metoprolol, and lisinopril. An ECG shows a heart rate of 72/min with a PR interval of 0.26 sec (N =0.12-0.20 sec), a ORS interval of 0.08 sec
(N <0.12 sec), and Q waves in leads II, Ill , and AVF. Which of the following drugs is most appropriate to add to this patient's regimen?

A) Alprazolam
B) Amitriptyline
C) Buspirone
lJ D) Carbamazepine
U E) Haloperidol
U F) Methylphenidate
G) Paroxetine
G. Page 549/612.
This is an elderly patient. Depression presents differently in the elderly.
This couldve been atypical depression. The two key features are mood reactivity and leaden paralysis.

He meets the SIGECAPS criteria of MDD.


He has sleep disturbance, anhedonia, and loss of energy. His weight loss could be due to appetite changes, the A of SIGECAPS.
If both MDD and atypical depression were answer choices, I would pick the MDD since neither leaden paralysis nor mood reactivity are mentioned.
However, one of the important diagnostic requirements of MDD is a self-reported depressed mood or anhedonia, and he mentions that.
So MDD is treated with SSRIs and paroxetine is perfect for that.

Amitriptyline - TCA. The C's of side effects. Cardiotoxic. Not safe in elderly according to Beer's criteria.
Buspirone - "Im anxious I will miss my bus." For GAD.
Carbamazepine - Neuro drug. "CARBs are bad for blood" because of agranulocytosis and aplastic anemia. Mainly for trigeminal neuralgia.
Haloperidol - Typical antipsychotic, not an antidepressant.

, ~ ~ p , ,.,,.
Previous Next Lab Values Calculator Review Help Pause
hi Comprehensive Basic Science Self-Assessment - Google Chrome x
i starttest.com/ITDVersions/19.3.0.1/ITDStart.aspx?SVC=54cd3e5f-e4bf-49d0-8371-ba261251f2a1

Exam Section 3: Item 42 of 50 National Board of Medical Examiners


• Mark Comprehensive Basic Science Self-Assessment Please Walt

43. In a culture of motile spore-forming bacteria, which of the following inhibits bacterial growth by causing double-stranded breaks in DNA?

A) Addition of phenol
B) Addition of streptomycin
C) Irradiation with ultraviolet light
D) Irradiation with x-rays
E) Starvation for a carbon source

D. Page 40.
You should have been able to cut this down to C or D.
Then, realize that the question is not about pyrimidine dimers or xeroderma pigmentosum.
Thus, this must be about x-rays, which cause double-strand breaks.

Simple. Just keep it simple.

, ~ ~ p , ,.,,.
Previous Next Lab Values Calculator Review Help Pause
hi Comprehensive Basic Science Self-Assessment - Google Chrome x
i starttest.com/ITDVersions/19.3.0.1/ITDStart.aspx?SVC=54cd3e5f-e4bf-49d0-8371-ba261251f2a1

Exam Section 3: Item 44 of 50 National Board of Medical Examiners Time Remaining:


• Mark Comprehensive Basic Science Self-Assessment 4 hr 53 min 46 sec

44. A 3-year-old boy has short stature and extremities and a relatively normal-sized trunk, a large head with a prominent forehead and low nasal bridge, and exaggerated lordosis. The
most likely cause of these findings is a genetic abnormality in which of the following?

CJ A) Calcium uptake
B) Endochondral ossification
C) Growth hormone synthesis
U D) Osteoclast activity
E) Tendon formation

B. Pg 450/506 and pg 454/511.


Straight up honest description of achondroplasia, like Peter Dinklage (Tyrion) of Game of Thrones.
The description of a normal trunk and large head relative to body size are key.
The mutation causes an activation of FGFR3, inhibiting chondrocyte proliferation.
The autosomal dominant form is lethal so everyone alive mustve had a sporadic mutation.
One cool fact is that it is associated with a high paternal age.

A-chondro -- Without endochondrol ossification.

, ~ ~ p , ,.,,.
Previous Next Lab Values Calculator Review Help Pause
hi Comprehensive Basic Science Self-Assessment - Google Chrome x
i starttest.com/ITDVersions/19.3.0.1/ITDStart.aspx?SVC=54cd3e5f-e4bf-49d0-8371-ba261251f2a1

Exam Section 3: Item 45 of 50 National Board of Medical Examiners Time Remaining:


• Mark Comprehensive Basic Science Self-Assessment 4 hr 53 min 40 sec

45. A 73-year-old man has an incurable malignant neoplasm of the lung, and his condition is slowly deteriorating. He is virtually incapable of movement and unable to breathe without
mechanical respiration ; however, he remains mentally competent. He requests that the respirator be removed, indicating that he no longer wants to endure the constant suffering.
His children want the respirator to be continued. After discussing the matter with the children, the physician chooses to order removal of the respirator. Which of the following best
describes the physician's action?

A) Legal but not ethical


B) Ethical but not legal
u C) Both legal and ethical
U D) Neither legal nor ethical

C. Ethics
The third question about autonomy this NBME.
The mentally capable and competent patient does not want to live this meaningless life of being hooked up.
Plus he has what seems like a terminal disease.

, ~ ~ p , ,.,,.
Previous Next Lab Values Calculator Review Help Pause
hi Comprehensive Basic Science Self-Assessment - Google Chrome x
i starttest.com/ITDVersions/19.3.0.1/ITDStart.aspx?SVC=54cd3e5f-e4bf-49d0-8371-ba261251f2a1

Exam Section 3: Item 46 of 50 National Board of Medical Examiners Time Remaining:


• Mark Comprehensive Basic Science Self-Assessment 4 hr 53 min 30 sec

46. In which of the following stages of the cell cycle are mitotic cyclins synthesized?

A) G0
U B) G1
U C) S
D) G2
E) M

D. Page 46 FA2019.
This is a straightforward question about making mitotic cyclins before mitosis occurs.
G2 is before mitosis, so thats it.
G2 is very irrelevant overall and rarely an answer, specially because even in pharmacotherapy
only bleomycin works at G2 (and topisomerase inhibitors but they also work in S phase).

There are G1/S cyclins that FA discusses and these control the G1 to S phase transition.
Then there are G2/M cyclins which are synthesized at G2 and are responsible for transition from G2 to M phase.

, ~ ~ p , ,.,,.
Previous Next Lab Values Calculator Review Help Pause
hi Comprehensive Basic Science Self-Assessment - Google Chrome x
i starttest.com/ITDVersions/19.3.0.1/ITDStart.aspx?SVC=54cd3e5f-e4bf-49d0-8371-ba261251f2a1

Exam Section 3: Item 47 of 50 National Board of Medical Examiners Time Remaining:


• Mark Comprehensive Basic Science Self-Assessment 4 hr 53 min 25 sec

47. A 40-year-old man who goes horseback riding 3 to 4 times weekly develops a painful swollen mass in his left inner thigh. Over the next 2 weeks, the mass becomes circumscribed
and very firm. Which of the following is the most likely diagnosis?

CJ A) Arteriovenous fistula
B) Dermatomyositis
C) Fasciitis
U D) Ganglion
E) Hemangioma
F) Myositis ossificans
G) Sarcoma
H) Synovial cyst

F. Not in FA.
Got it right! So I was initially thinking of the location, inner thigh. Okay. Nothing there but muscle and skin, etc. No special organs or blood vessels.
Then I thought about the presentation.
Painful and swollen (inflammation) and then circumscribed and very firm (not just firm, but very, and you know NBME).
M ossificans often occurs after trauma to that area over time. It is a form of metaplasia of mesenchymal tissue.

AV fistula would not become circumscribed and hard (C&H)


Dermatomyositis does not present like this. pg463/521.
Fascitis is an inflammation, it would not become C&H.
Ganglion would not present with a swollen mass. It could be painful, might become firm later idk but didnt "feel" right.
Hemangioma are pretty benign and usually just blood vessel related but would not harden.
Sarcoma didnt feel right either.
A synovial cyst would also not harden.

, ~ ~ p , ,.,,.
Previous Next Lab Values Calculator Review Help Pause
hi Comprehensive Basic Science Self-Assessment - Google Chrome x
i starttest.com/ITDVersions/19.3.0.1/ITDStart.aspx?SVC=54cd3e5f-e4bf-49d0-8371-ba261251f2a1

Exam Section 3: Item 48 of 50 National Board of Medical Examiners Time Remaining:


• Mark Comprehensive Basic Science Self-Assessment 4 hr 53 min 19 sec

48. A 3-week-old female newborn is brought to the physician for a follow-up examination after the results of newborn screening showed an increased serum concentration of
immunoreactive trypsin. Cystic fibrosis is suspected. At 4 months of age, her sweat chloride concentration is greater than 60 mmol/L (N<40). Molecular analysis that includes a panel
of the 70 most common cystic fibrosis transmembrane gene mutations is done. Results show a mutation in one allele. Which of the following best explains the findings in this
patient?

A) The immunoreactive trypsin test result is a false positive


B) The patient has another mutation that was not included in the previous analysis
u C) The patient has a different disorder that is a phenocopy of cystic fibrosis
U D) The patient has a mutation in another gene for a protein that interacts with CFTR
E) The sweat chloride test result is a false positive

B. Page 60 FA2019.
NBME trash alert. My NBME trash alert button is about to break with all the trash in this test.
CF can occur due to God knows how many different problems.
So they did a molecular analysis of the baby using 70 known CF mutations and they only found a mutation in one allele.
CF is autosomal recessive so the mutation shouldve been on both alleles.
B is the "best" answer choice because there must be some other mutation that wasnt accounted for and that did not get checked.
However, it couldve also been taht there is some other issue that is messing up something that interacts with CFTR.

This is actually a fair question because UW also had a very unfair question of this caliber.
Problem is, its ok when UW does it because UW is TEACHING you. Its not ok when a real test does it, so this is NBME trash.

, ~ ~ p , ,.,,.
Previous Next Lab Values Calculator Review Help Pause
hi Comprehensive Basic Science Self-Assessment - Google Chrome x
i starttest.com/ITDVersions/19.3.0.1/ITDStart.aspx?SVC=54cd3e5f-e4bf-49d0-8371-ba261251f2a1

Exam Section 3: Item 49 of 50 National Board of Medical Examiners Time Remaining:


• Mark Comprehensive Basic Science Self-Assessment 4 hr 53 min 14 sec

49. In a study of drug action on neoplastic cells in culture , drug X markedly inhibits cell replication. A microscopic view of a typical cell
incubated with drug Xis shown. Drug Xis most likely to be which of the following?

A) Cyclophosphamide
B) Cyclosporine
C) Doxorubicin
CJ D) 5-Fluorouracil
E) Vincristine

E. Page 433/479.
NBME Trash alert. uff.
I immediately thought of paclitaxel when I saw the given image.
After I didnt see it in the answer choices, the only other explanation was vincristine since they are listed together.
This is one of the big reasons I tell people to revisit FA for each UW question when you do qbanks.

Problem is, vincristine INHIBITS MITOTIC SPINDLE FORMATION. But in this image...it has formed.
They will probably use the same image for another question in some NBME later where paclitaxel will be the answer.

, ~ ~ p , ,.,,.
Previous Next Lab Values Calculator Review Help Pause
hi Comprehensive Basic Science Self-Assessment - Google Chrome x
i starttest.com/ITDVersions/19.3.0.1/ITDStart.aspx?SVC=54cd3e5f-e4bf-49d0-8371-ba261251f2a1

Exam Section 3: Item 50 of 50 National Board of Medical Examiners Time Remaining:


• Mark Comprehensive Basic Science Self-Assessment 4 hr 53 min 7 sec

50. A 27-year-old woman comes to the physician for a follow-up examination. She underwent a cesarean delivery 2 months ago. Physical examination shows a firm, subcutaneous
nodule adjacent to the surgical incision. Microscopic examination of the resected nodule shows fibrous connective tissue, macrophages, multinucleated giant cells, fibroblasts, a few
lymphocytes, and scattered fragments of polarizable foreign material. Which of the following substances that promotes fibroblast migration and proliferation most likely led to the
development of this lesion?

A) C3a
B) lnterleukin-1 (IL-1)
U C) IL-4
U D) Transforming growth factor-13
E) Tumor necrosis factor

D.
As I said the last time these answer choices popped up in an earlier question. Know your interleukins and cytokines!
TGFb does not have one specific area it is discussed in the book, but it is mentioned throughout the book.
Mainly its related to inflammation and fibroblast proliferation.
The other answer choices are really not related to this.

, ~ ~ p , ,.,,.
Previous Next Lab Values Calculator Review Help Pause
hi Comprehensive Basic Science Self-Assessment - Google Chrome x
i starttest.com/ITDVersions/19.3.0.1/ITDStart.aspx?SVC=54cd3e5f-e4bf-49d0-8371-ba261251f2a1

Exam Section 4: Item 1 of 50 National Board of Medical Examiners Time Remaining:


• Mark Comprehensive Basic Science Self-Assessment 4 hr 59 min 29 sec

1. A 15-year-old girl with cystic fibrosis has a mutation in the cystic fibrosis transmembrane regulator (CFTR) gene, which results in deletion of phenylalanine 508. This mutation does
not prevent synthesis of the CFTR protein but does prevent it from folding properly. The improperly folded CFTR protein will accumulate in which of the following cellular
compartments?

A) Cytosol
B) Endoplasmic reticulum
C) Nucleus
D) Peroxisome
E) Secretory granules

B. Page 60.
NBME = Naturally Bad at Making Exams.

In CFTR, the misfolded protein gets retained in the RER and this is in FA.
Cytosol was a trick answer.
UW had some questions regarding this misfolded protein being destroyed by ubiquitination and proteosomes, but that is different.
Most CFTR misfolds are so misfolded they cannot even leave the RER.
Eventually so much accumulates that the RER practically breaks and then it can be sent to proteosomes.

~ ~ p , ,.,,.
Next Lab Values Calculator Review Help Pause
hi Comprehensive Basic Science Self-Assessment - Google Chrome x
i starttest.com/ITDVersions/19.3.0.1/ITDStart.aspx?SVC=54cd3e5f-e4bf-49d0-8371-ba261251f2a1

Exam Section 4: Item 2 of 50 National Board of Medical Examiners Time Remaining:


• Mark Comprehensive Basic Science Self-Assessment 4 hr 59 min 20 sec

2. A 22-year-old woman comes to the physician for a follow-up examination. She has a 5-year history of migraines. She began taking an oral contraceptive 6 months ago, but she
stopped the contraceptive 1 month ago because it resulted in increased frequency and severity of her migraines. The headaches have since improved. She does not want to try other
forms of hormonal birth control, and she asks about the most effective alternative. It is most appropriate for the physician to recommend which of the following contraceptive methods
for this patient?

A) Contraceptive sponge C. Page 642/723.


B) Diaphragm Overall, the only forms of contraception youll ever give someone as a gynecologist are either OCPs or IUDs.
u C) Intrauterine device
The other things dont require a doctor and are found OTC.
Contraceptive sponge, diaphragm, and rhythm method are really unreal answers.
U D) Rhythm method
The first two are devices anyone can buy, and the rhythm method is based off of watching someones period cycles.
E) Spermicidal foam
Spermicidal foam is also something you can purchase OTC.

The catch in this question was to use something nonhormonal since hormonal OCPs caused her migraines to worsen.
A copper IUD is not hormonal and will work great.

, ~ ~ p , ,.,,.
Previous Next Lab Values Calculator Review Help Pause
hi Comprehensive Basic Science Self-Assessment - Google Chrome x
i starttest.com/ITDVersions/19.3.0.1/ITDStart.aspx?SVC=54cd3e5f-e4bf-49d0-8371-ba261251f2a1

Exam Section 4: Item 3 of 50 National Board of Medical Examiners Time Remaining:


• Mark Comprehensive Basic Science Self-Assessment 4 hr 59 min 15 sec

3. An investigator is studying a new virus isolated from a 4-year-old girl with fever and cough. Initial experiments show that the virus forms plaques on cultured human laryngeal cells but
rapidly loses its ability to form plaques when dried or exposed to a pH of 5.0. This infectious agent is most likely similar to which of the following viruses?

CJ A) Coronavirus
B) Coxsackievirus
C) Epstein-Barr virus
U D) Norovirus
E) Rotavirus

A.
How appropriate to ask about coronavirus lol.
This virus causes respiratory symptoms and is enveloped.
You know this virus must be enveloped because it does not hold up to acidity or being dried.
Only coronavirus of all the viruses listed is enveloped, making it the most likely answer even if no question was posed.
Remember the logic game of "which one of these is least like the other?"

, ~ ~ p , ,.,,.
Previous Next Lab Values Calculator Review Help Pause
hi Comprehensive Basic Science Self-Assessment - Google Chrome x
i starttest.com/ITDVersions/19.3.0.1/ITDStart.aspx?SVC=54cd3e5f-e4bf-49d0-8371-ba261251f2a1

Exam Section 4: Item 4 of 50 National Board of Medical Examiners Time Remaining:


• Mark Comprehensive Basic Science Self-Assessment 4 hr 59 min 9 sec

4. An otherwise healthy 35-year-old man sustains a fracture of the left pelvis in a motor vehicle collision. A 3-week period of bed rest for this patient is most likely to cause which of the
following physiologic changes?

CJ A) Decreased blood volume


B) Decreased plasma sodium concentration
C) Increased plasma aldosterone concentration
U D) Increased plasma volume
E) Increased sympathetic nerve activity

A. Not in FA.
Not sure if I wanna hit the "NBME Trash" alert again but it definitely feels like it.
Being in bedrest changes the effects of gravity on your body because you go from an upright plane to a horizontal plane.
Bedrest over 24 hours eventually causes a shift of fluids away from the legs towards the abdomen, thorax and head.
This increases venous return to the heart and elevates intracardial pressure.

This increased venous return stretches the RA, releasing ANP, a diuretic that induces urine output and decreases blood volume.
Normally, the drop in blood volume would be detected by the reduced stretch in the baroreceptors of the aortic and carotid sinus and induce ADH release.
ADH stimulates the kidney to reabsorb water, reducing urine output and increasing blood volume.
In supine individuals, the increased venous return also increases stretch of the aortic arch and carotid sinus baroreceptors, reducing ADH release.
This further increases the volume reduction.

In summary; Supine individuals have increased pressure in the RA and on their baroreceptors. The RA makes more ANP. The baroreceptors reduce ADH.
Mobile individuals would have reduced stretch of their baroreceptors, increasing ADH and balancing the volume loss.

Full in-depth conversation here --


https://www.nursingtimes.net/clinical-archive/cardiovascular-clinical-archive/effects-of-bedrest-1-cardiovascular-respiratory-and-haematological-systems-28-05-2009/

, ~ ~ p , ,.,,.
Previous Next Lab Values Calculator Review Help Pause
hi Comprehensive Basic Science Self-Assessment - Google Chrome x
i starttest.com/ITDVersions/19.3.0.1/ITDStart.aspx?SVC=54cd3e5f-e4bf-49d0-8371-ba261251f2a1

Exam Section 4: Item 5 of 50 National Board of Medical Examiners Time Remaining:


• Mark Comprehensive Basic Science Self-Assessment 4 hr 59 min 3 sec

5. A 23-year-old primigravid woman at 22 weeks' gestation is brought to the emergency department at a small rural hospital by her husband because of intermittent bleeding during the
past week. The patient woke up this morning with a copious amount of vaginal bleeding and severe pelvic pain. She has noticed no fetal movements for the past 24 hours. Her pulse
is 120/min, and blood pressure is 90/60 mm Hg. Her obstetrician is a woman whose practice is 2 hours away. The patient is dressed in a burka and explains that she is a conservative
Muslim. She does not want to be examined by a male physician. However, the only physician available in the emergency department is a man. Which of the following is the most
appropriate action by the physician?

A) Ask the patient if she would allow the examination if her husband is present at all times
B) Have a female nurse examine the patient and report her findings to the physician
C) Obtain an emergency court order to examine the patient
lJ D) Request that the hospital chaplain speak with the patient
U E) Transport the patient to a major medical facility 2 hours away where she can be examined by her female obstetrician

A. Ethics.

NBME TRASH ALERT. This question mocks the practice of Islam and practically deems it a "patriarchal" organization.
Although in some cultures, Islam may be a mockery, it is irrelevant to the exam. Islamic women are conservative for their connection to Allah.
The presence of her husband will not maintain conservatism.

However, this is probably what the idiot question writer was probably thinking:

Very easy question. Always remain calm and think of what you would have to do as the doctor and what you would want as that patient.
You are a female Muslim patient who is very conservative (burka). Some such patients are alright with this kind of problem if their husband is present.
SIMPLY ASKING such a question is PERFECTLY okay. She might say yes, she might say no. She wont be offended nor will the doctor do anything wrong.

This question is so easy, that if 1000 random non-medical non Muslim people were asked this, majority would pick A, probably over 80% of them.

, ~ ~ p , ,.,,.
Previous Next Lab Values Calculator Review Help Pause
hi Comprehensive Basic Science Self-Assessment - Google Chrome x
i starttest.com/ITDVersions/19.3.0.1/ITDStart.aspx?SVC=54cd3e5f-e4bf-49d0-8371-ba261251f2a1

Exam Section 4: Item 6 of 50 National Board of Medical Examiners Time Remaining:


• Mark Comprehensive Basic Science Self-Assessment 4 hr 58 min 58 sec

6. A 37-year-old man who is a farmworker comes to the emergency department because of a 12-hour history of severe pain in his abdomen and legs and painful spasms of his jaw.
Physical examination shows marked spasms of the masseter and abdominal musculature. During the examination, a loud noise in the examination room triggers painful spasms and
respiratory compromise requiring intubation. Administration of antitoxin prevents further symptoms, but the patient continues to require sedation and venti latory support for the next
3 weeks. This patient most likely has a syndrome that involves binding of a toxin to which of the following?

A) Acetylcholinesterase
B) N-Acetylneuraminic acid
u C) Calmodulin
U D) Monamine oxidase
E) Synaptobrevin
0 F) Ubiquitin

E.
The farmer has tetanus. They put choice A as AchASE because they wanted you to think farmer-->organic phosphate poisoning
but this patient does not have cholinergic symptoms. Once you recognized the question is asking about SNARE proteins, you had
to find which of these it could be.

You could cancel out A, C, D, and F since those are irrelevant to tetanus.
That leaves B and E. Acetyl-anything is usually some kind of salicylate and so is this one.
Synaptobrevin (VAMP), SNAP25, and Syntaxin are SNARE proteins involved in tetanus.

, ~ ~ p , ,.,,.
Previous Next Lab Values Calculator Review Help Pause
Comprehensive Basic Science Self-Assessment - Google Chrome x
i starttest.com/ITDVersions/19.3.0.1/ITDStart.aspx?SVC=54cd3e5f-e4bf-49d0-8371-ba261251f2a1

Exam Section 4: Item 6 of 50 National Board of Medical Examiners


• Mark Comprehensive Basic Science Self-Assessment Please Walt

7. A 43-year-old woman comes to the physician because of a 1-week history of abdominal pain, nausea, vomiting, itching, and fatigue. Physical examination shows scleral icterus and
right upper quadrant abdominal tenderness. There is no rash. Abdominal ultrasonography shows a large stone in the common bile duct. Complete blood count with differential shows
no abnormalities. Serum total bilirubin concentration is markedly increased. The serum concentration of which of the following is most likely to be markedly increased?

A) Acid phosphatase
B) ALT
C) Alkaline phosphatase
D) AST
E) Lactate dehydrogenase
F) Unconjugated bilirubin

C. Page 384/427 discusses liver pathology . Page 390/433 discusses cholelithiasis.


Patient has gallstones possibly due to biliary stasis. This has caused damaged to the biliary tract because a stone must be lodged in the common bile duct.
Now bile will back up the common hepatic duct and cause injury to the epithelial lining of the biliary tract.
Its leakage into the bloodstream induces the itching shes having.

Any damage to biliary tract epithelia leads to leakage of alkaline phosphatase, which is significant for cholestasis and its consequences like biliary obstruction.

AST and ALT would be elevated in hepatocyte injury, which is not the case here.
Acid phosphatase is just a phosphatase, so it removes phosphates off other molecules.
LDH converts pyruvate to lactate. Clinically it is significant of hemolysis. LDH is abundant in RBC.
Bilirubin conjugation occurs in hepatocytes and they are functional so you would not have an elevation in unconjugated bilirubin.
There would be an elevation in conjugated bilirubin.
hi Comprehensive Basic Science Self-Assessment - Google Chrome x
i starttest.com/ITDVersions/19.3.0.1/ITDStart.aspx?SVC=54cd3e5f-e4bf-49d0-8371-ba261251f2a1

Exam Section 4: Item 8 of 50 National Board of Medical Examiners Time Remaining:


• Mark Comprehensive Basic Science Self-Assessment 4 hr 58 min 51 sec

8. Serum cholesterol concentrations are measured as part of a community study. The means and standard deviations are given for women by age group.
Age (Years) Cholesterol (mg/dl)
45-49 229 ± 47
50-54 246 ± 50
55-59 255 ± 48
60-62 244 ± 36

Assuming serum cholesterol concentrations follow a normal (gaussian) distribution, which of the following is the probability that a woman between the ages of 50 and 54 years has a
serum cholesterol concentration greater than 296 mg/dl?

A) 1%
B) 2.5%
C) 5%
D) 16%
E) 95%

D. Page 261/273.
The data presented is simple.
We are only concerned with the 50-54 age group and they said you can assume a normal distribution bell curve.
For greater than 296, we are concerned with the people after 1SD because within 1SD would be people up to 296 mg/dL.

1 SD = 34%
2 SD = 34% + 13%
3 SD = 34% + 13% + 2%

Our patient is the 13 + 2 roughly 15 or 16% based on how you round the decimals. So D.

, ~ ~ p , ,.,,.
Previous Next Lab Values Calculator Review Help Pause
hi Comprehensive Basic Science Self-Assessment - Google Chrome x
i starttest.com/ITDVersions/19.3.0.1/ITDStart.aspx?SVC=54cd3e5f-e4bf-49d0-8371-ba261251f2a1

Exam Section 4: Item 9 of 50 National Board of Medical Examiners Time Remaining:


• Mark Comprehensive Basic Science Self-Assessment 4 hr 58 min 46 sec

9. A 20-year-old woman with asthma comes to the physician because of exacerbation of her symptoms during the past 3 months. She says, "I'm able to control my asthma attacks using
my inhaler, but it concerns me that these attacks are happening so often lately." She is a college student who has always carried a heavy course load; she says that it is sometimes
stressful taking so many classes at one time, but she enjoys the challenge. Three months ago, she moved from the campus dormitory into an off-campus apartment with some
friends. She tells the physician, "I'm really happy that I was able to bring my pet poodle to live in the apartment. Sometimes I have disagreements with my roommates because they
never seem to want to clean up after themselves, and one of them kind of annoys me by smoking in the apartment. Also, I think he keeps the apartment too warm. But all in all, I'm
really happy to be there." Physical examination shows no abnormalities. It is most appropriate for the physician to advise that the patient do which of the following?

A) Ask the roommate not to smoke in the apartment


B) Begin treatment with daily prednisone
C) Decrease her course load until the exacerbations decline in frequency
() D) Get an air cleaner for the apartment
u E) Move back to the dormitory
U F) Try to find a new home for the dog
G) Turn down the thermostat to keep the ambient temperature at 65°F

A.
NBME TRASH ALERT. I thought it would be D. And as I checked the answers I was chanting "dont be NBME trash" as I was concerned about A.
It would be great if the roommate wasnt a peice of shit that smoked inside the house with other roommates present, but that is not in the doctors control.
The student herself shouldve already asked the roommate to not smoke, or is fully aware of it and has accepted it. Cant be changed.

Of the remaining choices, getting an air cleaner is the best choice. There are amazing air purifiers out in the market today that can resolve all of this.

The other things like the dog and her school stress are not new variables.

, ~ ~ p , ,.,,.
Previous Next Lab Values Calculator Review Help Pause
hi Comprehensive Basic Science Self-Assessment - Google Chrome x
i starttest.com/ITDVersions/19.3.0.1/ITDStart.aspx?SVC=54cd3e5f-e4bf-49d0-8371-ba261251f2a1

Exam Section 4: Item 10 of 50 National Board of Medical Examiners Time Remaining:


• Mark Comprehensive Basic Science Self-Assessment 4 hr 58 min 42 sec

10. A 22-year-old man is brought to the emergency department 30 minutes after he was involved in a motorcycle coll ision. Physical examination shows dysmetria on the right. Muscle
strength is normal. Which of the following labeled structures in the photograph of the brain is the most likely site of injury in this patient?

v A) B) C) U D) E)

E. Page 487/545.
This patient has dysmetria on the right after a brain injury. Dysmetria is a type of ataxia that has an
overshoot or undershoot due to a lack of movement coordination. Patients fail the finger-to-nose test.
This is immediately supposed to make you think of the cerebellum, which comes down to D and E.
The D might be the flocculonodular lobe or central cerebellum, which would cause nystagmus and
truncal ataxia. This patient has an issue with the lateral cerebellum causing ipsilateral dysmetria.

, ~ ~ p , ,.,,.
Previous Next Lab Values Calculator Review Help Pause
hi Comprehensive Basic Science Self-Assessment - Google Chrome x
i starttest.com/ITDVersions/19.3.0.1/ITDStart.aspx?SVC=54cd3e5f-e4bf-49d0-8371-ba261251f2a1

Exam Section 4: Item 11 of 50 National Board of Medical Examiners Time Remaining:


• Mark Comprehensive Basic Science Self-Assessment 4 hr 58 min 36 sec

11. A 54-year-old man with a myocardial infarction becomes progressively hypotensive. Which of the following physiologic events is most likely to cause swelling of parenchymal cells in
his vital organs within minutes?

CJ A) Decrease in intracellular ATP concentrations


B) Decrease in intracellular pH
C) Increase in ribosomal protein synthesis
U D) Large influx of extracellular Ca 2 +
E) Release of osmotically active glucose from glycogen granules

A. Page 302/327 progression of an MI.


This is a high-yield topic that is easy to understand and master.
After an MI, the hypoxic environment of that damaged heart leads to use of anaerobic respiration and a deficiency of ATP and rise in lactate.
The lack of ATP and rise of lactate is similar to what happens in strenuous exercise. Now, without the ATP, the Na/K ATPase fails so there
is an accumulation of extracellular K, which predisposes patients to ventricular arrhythmia.

At the same time, there is also an intracellular accumulation of Na and Ca, which pulls in water and causes edema.
The intracellular Ca also causes hypercontraction of the myofibrils.

After this there is extensive coagulative necrosis and postinfarction fibrinous pericarditis, granulation tissue that can cause tamponade or MR,
and finally a full scar with a chance of a mural thrombus or HF.

, ~ ~ p , ,.,,.
Previous Next Lab Values Calculator Review Help Pause
hi Comprehensive Basic Science Self-Assessment - Google Chrome x
i starttest.com/ITDVersions/19.3.0.1/ITDStart.aspx?SVC=54cd3e5f-e4bf-49d0-8371-ba261251f2a1

Exam Section 4: Item 12 of 50 National Board of Medical Examiners Time Remaining:


• Mark Comprehensive Basic Science Self-Assessment 4 hr 58 min 33 sec

12. The pedigrees of patients with schizophrenia most closely resemble those of patients with which of the following?

A) Cystic fibrosis
U B) Diabetes mellitus, type 1
U C) Fragile X syndrome
D) Polycystic kidney disease
E) Tay-Sachs disease

B. Page 548/612.
This was a silly question. I approached it using the logic method of "which one of these is different from the other?"
That being said, DM1 is the only one without a clear pattern of inheritance, similar to schizophrenia.
I doubt the pedigrees of DM1 and schizo would look similar, but they would be "most" similar in comparison to the rest.

, ~ ~ p , ,.,,.
Previous Next Lab Values Calculator Review Help Pause
hi Comprehensive Basic Science Self-Assessment - Google Chrome x
i starttest.com/ITDVersions/19.3.0.1/ITDStart.aspx?SVC=54cd3e5f-e4bf-49d0-8371-ba261251f2a1

Exam Section 4: Item 13 of 50 National Board of Medical Examiners Time Remaining:


• Mark Comprehensive Basic Science Self-Assessment 4 hr 58 min 29 sec

13. A 62-year-old man comes to the physician for a follow-up examination. One month ago, he was prescribed a proton pump inhibitor (PPI) for epigastric pain that is relieved when he
eats. Physical examination shows no abnormalities. The patient says that the medication has only partially relieved his symptoms, and he still has discomfort. Serum studies show a
gastrin concentration four times the reference range. The physician discontinues the PPI therapy in the patient. Three weeks later, additional laboratory studies are ordered. It is
most appropriate to measure the concentration of which of the following at this time?

A) Plasma vasoactive intestinal polypeptide


B) Serum cholecystokinin
u C) Serum gastrin
U D) Serum histamine
E) Serum somatostatin

C. Page 392/436.
No reason to measure anything else. You measured gastrin to be high, now discontinue the PPI and see if the gastrin goes down.
If the patient does not have a gastrinoma or something, then once the PPI is taken away, gastrin will cause acid release and the gastrin will decrease.
If the gastrin didnt decrease once it successfully lead to more acid production, youd suspect a gastrinoma of sorts.

, ~ ~ p , ,.,,.
Previous Next Lab Values Calculator Review Help Pause
hi Comprehensive Basic Science Self-Assessment - Google Chrome x
i starttest.com/ITDVersions/19.3.0.1/ITDStart.aspx?SVC=54cd3e5f-e4bf-49d0-8371-ba261251f2a1

Exam Section 4: Item 14 of 50 National Board of Medical Examiners Time Remaining:


• Mark Comprehensive Basic Science Self-Assessment 4 hr 58 min 24 sec

14. A 59-year-old woman has a 10-year history of progressive right-sided hearing loss. An MRI of the head shows a large cerebellopontine angle mass that has compressed the
vestibulocochlear nerve on the right. This mass most likely arose from which of the following cell types?

CJ A) Astrocytes
C.Page 514/575.
B) Glioblasts
Giveaway question.
C) Neural crest cells The patient has an acoustic schwannoma.
U D) Neuroblasts Schwann cells are neural crest cells.
E) Oligodendrocytes Stains S-100 positive.

, ~ ~ p , ,.,,.
Previous Next Lab Values Calculator Review Help Pause
hi Comprehensive Basic Science Self-Assessment - Google Chrome x
i starttest.com/ITDVersions/19.3.0.1/ITDStart.aspx?SVC=54cd3e5f-e4bf-49d0-8371-ba261251f2a1

Exam Section 4: Item 15 of 50 National Board of Medical Examiners Time Remaining:


• Mark Comprehensive Basic Science Self-Assessment 4 hr 58 min 19 sec

15. A 32-year-old woman recently diagnosed with AIDS is at greatest risk for developing which of the following neoplasms?

A) Epstein-Barr virus-induced brain lymphoma


U B) Helicobacter pylori-associated gastric lymphoma A. Page 177/187.
U C) Hepatitis B virus-induced hepatocellular carcinoma The only AIDS defining illness in this list is EBV lymphoma.
D) Hormone-induced endometrial carcinoma The others can happen to anyone.
E) Ultraviolet light-induced skin melanoma

, ~ ~ p , ,.,,.
Previous Next Lab Values Calculator Review Help Pause
hi Comprehensive Basic Science Self-Assessment - Google Chrome x
i starttest.com/ITDVersions/19.3.0.1/ITDStart.aspx?SVC=54cd3e5f-e4bf-49d0-8371-ba261251f2a1

Exam Section 4: Item 16 of 50 National Board of Medical Examiners Time Remaining:


• Mark Comprehensive Basic Science Self-Assessment 4 hr 58 min 15 sec

16. A 28-year-old man has a blood pressure cuff placed around his left arm; the cuff is inflated to totally occlude arterial blood flow for 2 minutes and is then removed. Blood flow in the
left arm increases by 50% during the next 3 minutes and then decreases to control values. Which of the following humeral substances is most likely involved?

CJ A) Acetylcholine
B) Adenosine
C) Aldosterone
U D) Epinephrine
E) Norepinephrine
F) Prostacyclin (PGI 2)
G) Prostaglandin F 2 a
H) Serotonin
u I) Thromboxane A 2

B. Page 319/348.
Honestly, none of the other choices here have anything to do with this topic.
Adenosine is a very short acting substance that has effects in vasculature.

When the cuff is tight, cells and tissue distal to the cuff will run out of ATP now that new blood isnt arriving.
There will be an accumulation of ADP (adenosine), which causes vasodilation in vascular smooth muscle.
Here, adenosine acts via A2 receptors (not alpha 2, but A2), using Gs second messengers to raise cAMP.

In the heart nodes, adenosine acts via A1 receptors to act through Gi second messengers and reduce cAMP to lower chronotropy and dromotropy.

, ~ ~ p , ,.,,.
Previous Next Lab Values Calculator Review Help Pause
hi Comprehensive Basic Science Self-Assessment - Google Chrome x
i starttest.com/ITDVersions/19.3.0.1/ITDStart.aspx?SVC=54cd3e5f-e4bf-49d0-8371-ba261251f2a1

Exam Section 4: Item 17 of 50 National Board of Medical Examiners Time Remaining:


• Mark Comprehensive Basic Science Self-Assessment 4 hr 58 min 11 sec

17. A 37-year-old woman with HIV infection is brought to the emergency department because of a 6-hour history of chest and abdominal pain and shortness of breath with exertion. She
is currently receiving antiretroviral therapy. Her pulse is 100/min, respirations are 20/min, and blood pressure is 104/62 mm Hg. Physical examination shows no abnormalities.
Laboratory studies show:
Serum
HC0 3- 16 mEq/L
AST 255 U/L
ALT 198 U/L
Lactate 90 mg/dL (N=9-18)
Arterial blood gas on room air
pH 7.25
Po 2 98 mm Hg

A drug from which of the following classes is the most likely cause of the findings in this patient?

U A) CCR5 receptor antagonist


B) HIV fusion inhibitor
0 C) HIV protease inhibitor
D) lntegrase inhibitor
E) Nucleoside reverse transcriptase inhibitor

E. Page 203/213.
Second or third HIV treatment question this NBME. Worth mastering. Theyre not so bad just takes time and its not complex.
THe NRTIs have many side effects, but the NNRTIs are what are really known for rash and hepatotoxicity (plus vivid dreams from Efavirenzzz).

The protease inhibitors have metabolic issues for the most part but not really hepatotoxicity.
Integrase and entry inhibitors are practically clear of any major side effects.

, ~ ~ p , ,.,,.
Previous Next Lab Values Calculator Review Help Pause
hi Comprehensive Basic Science Self-Assessment - Google Chrome x
i starttest.com/ITDVersions/19.3.0.1/ITDStart.aspx?SVC=54cd3e5f-e4bf-49d0-8371-ba261251f2a1

Exam Section 4: Item 18 of 50 National Board of Medical Examiners Time Remaining:


• Mark Comprehensive Basic Science Self-Assessment 4 hr 58 min 7 sec

18. A 32-year-old woman comes to the physician because of pain and swelling of her right cheek for 24 hours. Physical examination shows a mildly swollen and tender area overlying
the parotid gland. Sialolithiasis is suspected. The calculus is most likely present in a duct that passes through which of the following muscles to enter the oral cavity?

CJ A) Buccinator
B) Masseter
C) Orbicularis oris
U D) Temporalis
E) Zygomaticus major

A.
Why would you know this?
Who truly cares?
Find out in the next episode of more NBME Trash series...

The parotid gland's secretions follow the Stensons duct and pierce through the buccinator to enter the oral cavity.

, ~ ~ p , ,.,,.
Previous Next Lab Values Calculator Review Help Pause
hi Comprehensive Basic Science Self-Assessment - Google Chrome x
i starttest.com/ITDVersions/19.3.0.1/ITDStart.aspx?SVC=54cd3e5f-e4bf-49d0-8371-ba261251f2a1

Exam Section 4: Item 19 of 50 National Board of Medical Examiners Time Remaining:


• Mark Comprehensive Basic Science Self-Assessment 4 hr 58 min 3 sec

19. A 35-year-old woman with asthma comes to the emergency department because of a 1-week history of abdominal pain and
diarrhea. She also has had progressive cough and wheezing that paradoxically worsened shortly after she began treatment with
oral prednisone for an asthma exacerbation. She recently returned from a trip to Papua New Guinea. Her temperature is 37.8°C
(100°F), pulse is 96/min, respirations are 24/min, and blood pressure is 124/84 mm Hg. Physical examination shows cutaneous
larva currens over the abdomen. A chest x-ray shows bilateral central alveolar infiltrates. Her leukocyte count is 18,000/mm 3 (with
23% eosinophils). Stool examination and analysis of fluid obtained on bronchoalveolar lavage show the presence of the organism
shown in the photomicrograph. Prednisone is discontinued. Which of the following is the most appropriate pharmacotherapy for
this patient?

A) Dexamethasone
u B) Hydroxychloroquine
C) Mefloquine
D) Praziquantel
E) Thiabendazole

E. Page 159/164.
This patient has Strongyloides stercoralis infection.
Her recent travel to PNG, cough, alveolar infiltrates, eosinophelia, and stool sample with a worm are all indicative of intestinal parasite infection.
The physical exam of "cutaneous larva currens" is more specific for S stercoralis infection.
Cutaneous larva migrans is seen more on the feet in patients with Ancylostoma or Necator infections.

Treatment of Strongyloides is with ivermectin or bendazoles.


Praziquantel is appropriate for worm or liver fluke infections.

, ~ ~ p , ,.,,.
Previous Next Lab Values Calculator Review Help Pause
hi Comprehensive Basic Science Self-Assessment - Google Chrome x
i starttest.com/ITDVersions/19.3.0.1/ITDStart.aspx?SVC=54cd3e5f-e4bf-49d0-8371-ba261251f2a1

Exam Section 4: Item 20 of 50 National Board of Medical Examiners Time Remaining:


• Mark Comprehensive Basic Science Self-Assessment 4 hr 57 min 57 sec

20. A 65-year-old man is brought to the emergency department 30 minutes after the sudden onset of shortness of breath and discomfort in his chest. He says he feels weak and
apprehensive. His pulse is 110/min, respirations are 22/min , and blood pressure is 100/80 mm Hg. Physical examination shows diaphoresis. An ECG shows ST-segment elevation in
the anterior leads. Compared with a healthy man of the same age, which of the following sets of cardiopulmonary changes is most likely in this patient?

Systemic Vascular Pulmonary Vascular Pulmonary Capillary


Resistance Resistance Wedge Pressure
U A) i i !
U B) i ! i
u C) i ! !
D) ! i i
E) ! i !
F) ! ! i

B. Page 307/335.
This patient has cardiogenic shock.
He is most likely experiencing an MI.
Since his heart cannot pump blood well, blood will stay in the heart and increase PCWP (that knocks out A, C, and E).
To fix this the body will have a sympathetic response and catecholamines will cause peripheral vasoconstriction (knocks out D and F).
That makes B the answer.

The pulmonary vascular resistance is determined by autoregulation. Since the heart is failing, it may release local vasodilatory metabolites.
That can cause pulmonary vasculature to dilate.

, ~ ~ p , ,.,,.
Previous Next Lab Values Calculator Review Help Pause
hi Comprehensive Basic Science Self-Assessment - Google Chrome x
i starttest.com/ITDVersions/19.3.0.1/ITDStart.aspx?SVC= 701083bd-6fcd-4283-9abd-818a8e3d62c7

Exam Section 4: Item 21 of 50 National Board of Medical Examiners Time Remaining:


• Mark Comprehensive Basic Science Self-Assessment 4 hr 56 min 41 sec

21. A 3-year-old girl has a history of recurrent infections. In vitro, neutrophils isolated from this patient are capable of phagocytosis and can kill Lactobacillus species but not
Staphylococcus aureus. This patient most likely has a defect involving which of the following enzymes?

CJ A) Catalase
B) Elastase
C) Myeloperoxidase
U D) NADPH oxidase
E) Superoxide dismutase

D. Page 117 FA2019. Page 109 discusses respiratory burst.


The patient has chronic granulomatous disease (CGD).
S aureus is catalase positive so it is able to convert its own hydrogen peroxide into oxygen and water.
Otherwise, patients without NADPH oxidase can still use the bacteria's hydrogen peroxide to create ROS and kill the bacteria.

For this reason, patients with CGD are susceptible to catalase positive infections.

, ~ ~ p , ,.,,.
Previous Next Lab Values Calculator Review Help Pause
hi Comprehensive Basic Science Self-Assessment - Google Chrome x
i starttest.com/ITDVersions/19.3.0.1/ITDStart.aspx?SVC= 701083bd-6fcd-4283-9abd-818a8e3d62c7

Exam Section 4: Item 22 of 50 National Board of Medical Examiners Time Remaining:


• Mark Comprehensive Basic Science Self-Assessment 4 hr 56 min 35 sec

22. A 12-year-old boy is brought to the physician by his mother for a well-child examination. His mother says that he has outgrown the clothes that she bought him 6 months ago. He is
at the 50th percentile for height and weight. Based on his medical record , the physician estimates the patient's growth velocity to be 6 cm (2.3 in)/year. Physical examination shows
mild acne over the face. Pubic hair and testes development are Tanner stage 2. Which of the following is the most likely cause of the onset of the physical changes in this patient?

A) Constant secretion of gonadotropin-releasing hormone (GnRH)


B) Decreased GnRH receptors
C) Decreased secretion of inhibin
D) Increased autonomous testosterone secretion
E) Nocturnal luteinizing hormone pulses

E.
Honestly was not 100% sure, but after knowing how the idiots of NBME write their answer choices, I felt E.
LH would increase testosterone production, which is the true cause of his symptoms.
The symptoms are not bad either, it just looks like hes hitting puberty and its not coming slow.

, ~ ~ p , ,.,,.
Previous Next Lab Values Calculator Review Help Pause
hi Comprehensive Basic Science Self-Assessment - Google Chrome x
i starttest.com/ITDVersions/19.3.0.1/ITDStart.aspx?SVC= 701083bd-6fcd-4283-9abd-818a8e3d62c7

Exam Section 4: Item 23 of 50 National Board of Medical Examiners Time Remaining:


• Mark Comprehensive Basic Science Self-Assessment 4 hr 56 min 29 sec

23. A 4-year-old girl has a history of multiple bone fractures and poor wound healing. A photograph of the face is shown. Which of the following components of wound healing is most
likely to be affected as a direct result of her underlying disease?

A) Cell migration
B) Clot formation
C) Granulation tissue deposition
D) Inflammation
E) Scar formation

E. Page 51 FA2019.
Rumor has it this image was taken with a Nokia. That's why the image is still around even though we have 108mp cameras.

This image is supposed to be showing you blue sclerae. I for one can even argue that I see white keratin deposition from vitamin A deficiency.

Scars are type one collagen and this is a kid with Osteogenesis imperfecta. Dont always rely on images when it comes to the NBME.
They might use this same image later to tell you a kid looked at you like this after you told him what to do, and the answer might be oppositional
defiant disorder for a psych question. So use the facts of the question itself.

, ~ ~ p , ,.,,.
Previous Next Lab Values Calculator Review Help Pause
hi Comprehensive Basic Science Self-Assessment - Google Chrome x
i starttest.com/ITDVersions/19.3.0.1/ITDStart.aspx?SVC= 701083bd-6fcd-4283-9abd-818a8e3d62c7

Exam Section 4: Item 24 of 50 National Board of Medical Examiners Time Remaining:


• Mark Comprehensive Basic Science Self-Assessment 4 hr 56 min 22 sec

24. A 17-year-old girl has significant blood loss after being injured in an automobile collision. Blood loss stimulates bone marrow to synthesize which of the following?

A) o-Aminolevulinate
U B) Bilirubin
U C) Erythropoietin
D) Ferritin
E) Uric acid

A. Page 417/461.
The question asks what the bone marrow will synthesize.
Bilirubin is the liver, EPO would be increased in this patient but that is the kidney, ferritin is irrelevant, and uric acid is also irrelevant.
Nor are ferriting and uric acid made by bone marrow.

Blood loss requires hematopoiesis to make up for the loss.


The first step of heme synthesis is the fusion of glycine and succinyl CoA, which can form dALA in the bone marrow.

, ~ ~ p , ,.,,.
Previous Next Lab Values Calculator Review Help Pause
hi Comprehensive Basic Science Self-Assessment - Google Chrome x
i starttest.com/ITDVersions/19.3.0.1/ITDStart.aspx?SVC= 701083bd-6fcd-4283-9abd-818a8e3d62c7

Exam Section 4: Item 25 of 50 National Board of Medical Examiners Time Remaining:


• Mark Comprehensive Basic Science Self-Assessment 4 hr 56 min 17 sec

25. A 1-month-old male newborn is brought to the physician for a routine examination. His parents both have olive-colored skin, dark hair, and dark eyes. Physical examination shows
hypopigmentation of the skin, light blonde hair, and translucent irises. The inherited disorder that causes this phenotypic expression is most likely due to a defect in the metabolism
of which of the following?

A) Epinephrine
B) Phenylalanine E. Page 83.
C) Serotonin The child has albinism, which is due to a lack of tyrosinase activity.
D) Tryptophan
E) Tyrosine

, ~ ~ p , ,.,,.
Previous Next Lab Values Calculator Review Help Pause
hi Comprehensive Basic Science Self-Assessment - Google Chrome x
i starttest.com/ITDVersions/19.3.0.1/ITDStart.aspx?SVC= 701083bd-6fcd-4283-9abd-818a8e3d62c7

Exam Section 4: Item 26 of 50 National Board of Medical Examiners Time Remaining:


• Mark Comprehensive Basic Science Self-Assessment 4 hr 56 min 13 sec

26. A 37-year-old man is admitted to the hospital because of gradually progressive weakness, anorexia, and weight loss over the past 6 months. His blood pressure is 74/40 mm Hg and
his skin is hyperpigmented. Morning serum cortisol concentration is 2 µg/dl. Which of the following is the most likely diagnosis?

CJ A) Adrenocortical carcinoma In summary-


No Aldosterone = Lose Na+, Keep K+ & H+.
B) Amyloidosis
Keeping H+ means losing HCO3-.
C) Autoimmune adrenalitis Losing HCO3- means keeping Cl- via
U D) Basophilic pituitary adenoma basolateral HCO3/Cl exchanger found in PCT.
E) Metastasis to the adrenal gland Metabolic acidosis.
Low Na and HCO3. High K, Cl, and H.
F) Pituitary necrosis (Sheehan syndrome) Think - Lose NaHCO3, Keep, HKCl.
G) Sarcoidosis No Cortisol = Anorexia, hypoglycemia, low BP.
H) Waterhouse-Friderichsen syndrome

C. Page 334/364.
This patient is presenting with symptoms resembling Addison disease.
The patient's adrenal glands must not be responding to the ACTH so he is making more ACTH (skin hyperpigmentation from MSH).
This is not a pituitary adenoma because this is not Cushing, where the patient would be overweight (cushingoid habitus).

This patient has primary adrenal insufficiency and would present with hyponatremic volume contraction and metabolic acidosis.

, ~ ~ p , ,.,,.
Previous Next Lab Values Calculator Review Help Pause
hi Comprehensive Basic Science Self-Assessment - Google Chrome x
i starttest.com/ITDVersions/19.3.0.1/ITDStart.aspx?SVC= 701083bd-6fcd-4283-9abd-818a8e3d62c7

Exam Section 4: Item 27 of 50 National Board of Medical Examiners Time Remaining:


• Mark Comprehensive Basic Science Self-Assessment 4 hr 56 min 9 sec

C
H T
T
L
T,P s
L??

27. A 34-year-old woman is brought to the emergency department 45 minutes after she slipped on an icy sidewalk and landed on her outstretched right hand. Examination of the right
upper extremity shows an exquisitely tender, swollen wrist with a palpable hard mass located immediately proximal to the wrist joint anteriorly. X-rays of the right wrist are shown.
Which of the following bones is most likely dislocated in this patient?

A) Capitate To the left we have an X-ray of the right hand's palmar view.
U B) Hamate A]Capitate B]Trapezoid C]Trapezium D]Scaphoid E]Lunate F]Pisiform G]Hamate
U C) Lunate
D) Pisiform The hook of hamate injury can lead to ulnar nerve entrapment causing a distal ulnar nerve
E) Trapezium injury presenting with paresthesias, numbness, and pain over the medial 2 fingers and claw hand.
F) Trapezoid
C. Page 439/485.
U G) Triquetral
You can tell that in the x-ray given, the middle bone, labelled E in our good xray, is missplaced.
For once, NBME did an amazing job showing a pathological xray clearly missing something. Always sing the song.
So Long To Pink, Here Comes The Thumb. Ive labelled them. The lateral view shows the lunate poking out.
I like to remember that 2 bones lay on the radius. Scaphoid (most common fracture) and lunate (causes acute carpal tunnel).
, ~ ~ p , ,.,,.
Previous Next Lab Values Calculator Review Help Pause
hi Comprehensive Basic Science Self-Assessment - Google Chrome x
i starttest.com/ITDVersions/19.3.0.1/ITDStart.aspx?SVC= 701083bd-6fcd-4283-9abd-818a8e3d62c7

Exam Section 4: Item 28 of 50 National Board of Medical Examiners Time Remaining:


• Mark Comprehensive Basic Science Self-Assessment 4 hr 56 min 4 sec

28. HOX genes play a role in embryogenesis through which of the following processes?

A) Allelic exclusion
U B) Alternate mRNA splicing
C. Page 598/673.
U C) Regulation of transcription
HOX genes organize the embryo in a craniocaudal direction by coding for transcription factors.
D) Regulation of translation
Shh works by signal transduction.
E) Signal transduction

, ~ ~ p , ,.,,.
Previous Next Lab Values Calculator Review Help Pause
hi Comprehensive Basic Science Self-Assessment - Google Chrome x
i starttest.com/ITDVersions/19.3.0.1/ITDStart.aspx?SVC= 701083bd-6fcd-4283-9abd-818a8e3d62c7

Exam Section 4: Item 29 of 50 National Board of Medical Examiners Time Remaining:


• Mark Comprehensive Basic Science Self-Assessment 4 hr 55 min 59 sec

29. A 35-year-old woman with a long-standing history of asthma treated with corticosteroids dies of complications of histoplasmosis. Examination at autopsy shows that the lungs are
1.5 times the normal weight. Extensive focal areas of fibrosis and 2- to 5-mm nodules are seen throughout both lungs. Examination of a biopsy specimen of the nodules is most
likely to show which of the following findings?

A) Diffuse lgG deposition


B) Immune complex deposition D. Page 151/153.
C) Increased concentration of eosinophils
This patient is immunocompromised and has severe histoplasma induced pneumonia.
All systemic mycoses can form granuloma like TB so D makes sense.
D) Infiltration of lymphocytes and monocytes
Mycoses are not fought with neutrophils (for bacteria) or immune complexes (also for bacteria).
E) Neutrophilic infiltrates

, ~ ~ p , ,.,,.
Previous Next Lab Values Calculator Review Help Pause
hi Comprehensive Basic Science Self-Assessment - Google Chrome x
i starttest.com/ITDVersions/19.3.0.1/ITDStart.aspx?SVC= 701083bd-6fcd-4283-9abd-818a8e3d62c7

Exam Section 4: Item 30 of 50 National Board of Medical Examiners Time Remaining:


• Mark Comprehensive Basic Science Self-Assessment 4 hr 55 min 55 sec

30. A 2-year-old boy is brought to the emergency department because of a 12-hour history of headache, loss of appetite, and vomiting. His temperature is 39.9°C (103.8°F), pulse is
120/min, respirations are 40/min, and blood pressure is 90/50 mm Hg. Physical examination shows nuchal rigidity. A lumbar puncture is done. Cerebrospinal flu id analysis shows an
increased protein concentration, decreased glucose concentration, abundant neutrophils, and gram-positive diplococci. Immunization with which of the following would most likely
have prevented this patient's infection?

A) Envelope glycoprotein
B) Killed bacterial vaccine
u C) Polysaccharide protein conjugate vaccine
U D) Recombinant vaccine
E) Toxoid

C. Page 136/137.
This patient is Strep pneumo induced bacterial meningitis,
S pneumo vaccination is extensively discussed in my microbiology video.
Children get the conjugate PCV13 is given to kids under age 5 years.
It is attached to Diptheria protein to elicit memory B cell development as well as T cell and IgG, so it is immunogenic.
The PPSV23 is given mainly to adults and forms IgM because it is not immunogenic.

, ~ ~ p , ,.,,.
Previous Next Lab Values Calculator Review Help Pause
hi Comprehensive Basic Science Self-Assessment - Google Chrome x
i starttest.com/ITDVersions/19.3.0.1/ITDStart.aspx?SVC= 701083bd-6fcd-4283-9abd-818a8e3d62c7

Exam Section 4: Item 31 of 50 National Board of Medical Examiners Time Remaining:


• Mark Comprehensive Basic Science Self-Assessment 4 hr 55 min 51 sec

31. A 62-year-old man comes to the physician because of a 2-month history of intermittent, severe pain of his left leg that began shortly after the leg was amputated below the knee. The
pain is described as throbbing, aching, and shooting, and is localized to the distal portion of the absent extremity. Each episode lasts from several seconds to a few minutes. The
pain is exacerbated by feelings of anxiety and changes in temperature and is not relieved by changing position. His vital signs are within normal limits. Examination of the left lower
extremity shows a well-healed surgical scar over the stump. Neurologic examination, including muscle strength testing , range of motion, deep tendon reflexes, and muscle tone, is
within normal limits. Which of the following is the most likely cause of this patient's symptoms?

A) Chronic infection of the residual extremity


B) Fibromyalgia
C) Neuralgia
lJ D) Phantom limb pain
U E) Radiculitis

D. Not in FA.
Again. Knowing NBME as soon as I saw that answer choice I was like this must be the answer.
On top of that, nothing made me really want to pick B or E.
A was out of the question, he has no infection.
C was a possibility but they said most of his neurologic exam is perfect.

This video discusses phantom limb pain in detail --> https://youtu.be/5BlsoyE1CIw

It is apparently an extremely common sequela of amputation and mainly due to the psychological stress your brain goes through
upon visualizing that the limb is physically missing.

, ~ ~ p , ,.,,.
Previous Next Lab Values Calculator Review Help Pause
hi Comprehensive Basic Science Self-Assessment - Google Chrome x
i starttest.com/ITDVersions/19.3.0.1/ITDStart.aspx?SVC= 701083bd-6fcd-4283-9abd-818a8e3d62c7

Exam Section 4: Item 32 of 50 National Board of Medical Examiners Time Remaining:


• Mark Comprehensive Basic Science Self-Assessment 4 hr 55 min 44 sec

32. A 33-year-old woman comes to the physician because of a 2-month history of easy fatigability and dark urine in the mornings. She has not had fever or weight loss during this
period. She has not traveled internationally. She takes no medications. Her temperature is 36.9°C (98.4 °F), pulse is 107/min, respirations are 20/min , and blood pressure is
121/73 mm Hg. Physical examination shows scleral icterus. Laboratory studies show:
Hemoglobin 9.2 g/dl
Hematocrit 35%
Serum
Bilirubin, total 3.5 mg/dl
Direct 0.2 mg/dl
Indirect 3.3 mg/dl
Urine
Blood 2+
Protein 1+

The result of a direct antiglobulin (Coombs) test is negative, and the result of an acidified serum test is positive. Which of the following best describes the underlying cellular cause of
this patient's condition?

U A) Abnormal cell morphology


B) Decreased glucose 6-phosphate dehydrogenase activity
U C) Defect in a cell membrane anchor protein
D) Destabilization of the cytoskeleton
E) Underproduction of globin proteins

C. Page 107 of FA2019.


This patient has paroxysmal nocturnal hemoglobinuria. Its a nocturnal thing, so she has dark urine "in the mornings."
G6PD deficiency is irrelevant because shes not sick, hasnt had fava beans, and is not in any stress.
Abnormal cell morphology (such as hereditary spherocytosis) would have presented earlier in her life, not acutely like this.
The "acidified serum test" is an obsolete test that could diagnose PNH by putting RBCs in mild acid to see if the RBC are fragile.
It is not sensitive or specific so it sucks and isnt really done (get up to date on things NBME wtf) and can even be positive in aplastic anemia.
Underproduction of globins would also have had presented earlier in her life.

, ~ ~ p , ,.,,.
Previous Next Lab Values Calculator Review Help Pause
hi Comprehensive Basic Science Self-Assessment - Google Chrome x
i starttest.com/ITDVersions/19.3.0.1/ITDStart.aspx?SVC= 701083bd-6fcd-4283-9abd-818a8e3d62c7

Exam Section 4: Item 33 of 50 National Board of Medical Examiners Time Remaining:


• Mark Comprehensive Basic Science Self-Assessment 4 hr 55 min 38 sec

33. A 3-month-old boy is brought to the emergency department because of shortness of breath and listlessness for 3 hours. He has had an upper respiratory tract infection and has
eaten poorly for the past 3 days. His temperature is 37°C (98.6°F), and respirations are 30/min. Physical examination shows lethargy and mild hepatomegaly. Laboratory studies
show hypoglycemia, lactic acidemia, ketonemia , and metabolic acidosis. Following the intravenous administration of glycerol or fructose on different days, his serum glucose
concentrations do not increase. However, they do increase normally after the intravenous administration of galactose. A defect in which of the following liver metabolic pathways is
the most likely cause of these findings?

A) Fatty acid oxidation


B) Gluconeogenesis
C) Glycogen breakdown
lJ D) Glycogen synthesis
U E) Glycolysis

B. Page 74. Highlighted parts in image to right.


The most important fact they told was that administration of fructose or glycerol was not
increasing serum glucose.
The child has hypoglycemia and is making ketones.
When you give someone glycerol, its helpful in making glycerol-3-phosphate.
When you give someone fructose, that can also become glycerol-3-phosphate.

This G3P then becomes pyruvate which should be able to help go become acetyl CoA,
and become HMG CoA for ketogenesis. This is fine in the baby boy.

The same G3P can also go backwards for gluconeogenesis to make glucose.
For this baby it seems this path is not working.

Giving galactose helps because it skips this step regarding g3p.


Galactose becomes glucose via glycogenolysis and forms free glucose.

, ~ ~ p , ,.,,.
Previous Next Lab Values Calculator Review Help Pause
hi Comprehensive Basic Science Self-Assessment - Google Chrome x
i starttest.com/ITDVersions/19.3.0.1/ITDStart.aspx?SVC= 701083bd-6fcd-4283-9abd-818a8e3d62c7

Exam Section 4: Item 34 of 50 National Board of Medical Examiners Time Remaining:


• Mark Comprehensive Basic Science Self-Assessment 4 hr 55 min 32 sec

34. An 83-year-old woman is brought to the physician by her daughter to discuss the results of a complete dementia work-up. The patient has had mild memory impairment for
8 months. She takes no medications. Vital signs are normal. Her Mini-Mental State Examination score is 23/30. A rapid plasma reagin is 1:4, and a microhemagglutination assay for
Treponema pallidum is positive. Which of the following is the best next step for the physician?

A) Discussion of the diagnosis with the daughter privately


B) Discussion of the diagnosis with the patient privately
C) Disregarding the results since the patient is too old for treatment
D) Repeated tests
E) Lumbar puncture

B. Ethics and Treponema. Page 147/149.


This patient definitely has syphilis. The screening was done with RPR and the confirmatory test was the MHA-TP.
That being said, the patient also has dementia but her memory impairment is "mild" and her MMSE is not below 20/30.

Thus, it is best to first discuss HER health with HER first, in privacy.
If she fails to comprehend or if she herself says to discuss with her daughter present, then you proceed from there.

, ~ ~ p , ,.,,.
Previous Next Lab Values Calculator Review Help Pause
hi Comprehensive Basic Science Self-Assessment - Google Chrome x
i starttest.com/ITDVersions/19.3.0.1/ITDStart.aspx?SVC= 701083bd-6fcd-4283-9abd-818a8e3d62c7

Exam Section 4: Item 35 of 50 National Board of Medical Examiners Time Remaining:


• Mark Comprehensive Basic Science Self-Assessment 4 hr 55 min 27 sec

35. A 27-year-old man comes to the physician because he and his wife have not been able to conceive a child. He has poor libido and is unable to maintain an erection. He has been
receiving thyroid hormone and corticosteroid replacement therapy since surgical removal of a pituitary adenoma 2 years ago. His serum testosterone concentration is 0.05 nmol/L
(N=10-35). Semen analysis shows azoospermia. Which of the following is the most appropriate treatment to restore this patient's fertility?

A) Injection of gonadotropin-releasing hormone


B) Injections of gonadotropins
C) Oral clomiphene citrate
D) Oral progesterone
E) Testosterone patches

B. Page 640/722.
This man had his pituitary removed.
Today he has low testosterone, leading to azoospermia.
To raise his testosterone, we need him to make LH.
So inject LH.

Clomiphene or GnRH injections would fail because those rely on the pituitary.

, ~ ~ p , ,.,,.
Previous Next Lab Values Calculator Review Help Pause
hi Comprehensive Basic Science Self-Assessment - Google Chrome x
i starttest.com/ITDVersions/19.3.0.1/ITDStart.aspx?SVC= 701083bd-6fcd-4283-9abd-818a8e3d62c7

Exam Section 4: Item 36 of 50 National Board of Medical Examiners Time Remaining:


• Mark Comprehensive Basic Science Self-Assessment 4 hr 55 min 18 sec

36. A 30-year-old man develops tingling around the lips and mouth after consuming a small portion of fugu (puffer fish) in a Japanese restaurant. The liver of this fish contains
tetrodotoxin, a substance that causes cardiac arrest when consumed in high doses. This patient's symptoms are most likely due to the blocking action of tetrodotoxin on which of the
following ion channels?

A) Calcium
B) Chloride D. Page 246/256.
C) Potassium
This patient had pufferfish with tetrodotoxin.
This toxin binds Na channels in nerve tissue, preventing depolarization.
D) Sodium

, ~ ~ p , ,.,,.
Previous Next Lab Values Calculator Review Help Pause
hi Comprehensive Basic Science Self-Assessment - Google Chrome x
i starttest.com/ITDVersions/19.3.0.1/ITDStart.aspx?SVC= 701083bd-6fcd-4283-9abd-818a8e3d62c7

Exam Section 4: Item 37 of 50 National Board of Medical Examiners Time Remaining:


• Mark Comprehensive Basic Science Self-Assessment 4 hr 55 min 13 sec

37. A group of physicians submits a report to a medical journal that describes three patients with idiopathic pulmonary fibrosis who developed hepatotoxicity following treatment with a
recently approved drug. In the report, the physicians state that they are unaware of any previous description of this adverse effect. Which of the following best describes the study
design used by these clinicians?

A) Case series
B) Case-control study
C) Clinical trial
D) Correlational study
E) Prospective cohort study
A. Page 256/266.
F) Retrospective cohort study
Having done clinical trials and being a clinical research coordinator at montefiore hospital really made things clear first-hand.

This is a case series study. It is a type of medical research that uses demographics like age, gender, and ethnicity in conjunction with information
on their diagnosis, treatment, and response to treatment over a period of time.

Some people thought this is C.

Physicians report new side-effects of medications in phase 4 of clinical trials, which is when the drug is under post-marketing surveillance.
Patients take the drug because it is approved but if any unprecedented side effects show up, physicians must report it to the pharma company
and submit the adverse event report to the IRB so all other places where this medication is being administered can know of this. However, in this
scenario there would also be a control group. Drugs in phase 4 are offered often to clinical trial patients where the patient consents that they
might be put in Arm A (treatment) or Arm B (placebo) and then side effects can pop up.

The NBME 20 question-


An investigator is conducting a study to identify potential risk factors for post-transplant hypertension. The investigator selects post-transplant patients with
hypertension and gathers detailed information regarding their age, gender, preoperative blood pressure readings, and current medications. The results of the
study reveal that some of the patients had been treated with cyclosporine. This study is best described as which of the following? Case Series.
This study, which was compiled by aggregating several similar patient cases (i.e., all patients with post-transplant hypertension), represents a case (or clinical)
series. A case series has no control group and samples individuals based on either exposure (e.g., all patients treated with cyclosporine) or outcome (as seen
here). Without a control group, no measures of association between exposure and outcome (e.g., relative risk, odds ratio) can be calculated.
Therefore, case series do not provide evidence of cause and effect.

, ~ ~ p , ,.,,.
Previous Next Lab Values Calculator Review Help Pause
hi Comprehensive Basic Science Self-Assessment - Google Chrome x
i starttest.com/ITDVersions/19.3.0.1/ITDStart.aspx?SVC= 701083bd-6fcd-4283-9abd-818a8e3d62c7

Exam Section 4: Item 38 of 50 National Board of Medical Examiners Time Remaining:


• Mark Comprehensive Basic Science Self-Assessment 4 hr 55 min 7 sec

38. A man accidentally touches the surface of a hot stove; 20 minutes later a blister develops at the site. Light and electron microscopy of the inflamed tissue at this time is most likely to
show which of the following?

CJ A) Accumulation of macrophages
B) Fragmentation and hyalinization of dermal collagen
C) lnterendothelial gaps in venules
U D) Necrosis of arterioles
E) Perivascular collections of eosinophils

C.
This patient simply has a blister.
This has nothing to do with macrophages or necrosis, so A and D are out of the question.
This patient would not have eosinophelia in the area but the use of "perivascular" is a distracter to make you think of choosing this answer.

This burn is simply going to cause an inflammatory response (think of it as the calor) and that leads to post-venule capillary dilation.
Post venule is related to choice C.

, ~ ~ p , ,.,,.
Previous Next Lab Values Calculator Review Help Pause
hi Comprehensive Basic Science Self-Assessment - Google Chrome x
i starttest.com/ITDVersions/19.3.0.1/ITDStart.aspx?SVC= 701083bd-6fcd-4283-9abd-818a8e3d62c7

Exam Section 4: Item 39 of 50 National Board of Medical Examiners Time Remaining:


• Mark Comprehensive Basic Science Self-Assessment 4 hr 54 min 20 sec

39. A 43-year-old woman comes to the physician because of a nonproductive cough for 3 weeks. She has had a 6.8-kg (15-lb) weight loss
during the past 3 months. A chest x-ray shows three 0.3- to 1-cm nodules in the right lung. Cytologic examination of fi ne-needle aspirate
from the largest nodule strongly suggests a malignant neoplasm. A photograph representative of the findings in this patient's lungs is
shown. Which of the following is the most likely diagnosis?

A) Carcinoid tumor
U B) Malignant mesothelioma
C) Metastatic carcinoma
U D) Pulmonary hamartoma
E) Small cell carcinoma

C. Page 669/769.
They confirmed that this is a malignant neoplasm, so you can cancel out D since that is usually a coincidental, benign finding.
No risk factor or pathological image is given to make us think of malignant mesothelioma (psammoma bodies, pleural thickening, asbestos).

Small cell carcinoma is central and carcinoid tumor is central with an excellent prognosis (no mass effects of flushing, diarrhea, wheezing).

This is clearly a metastatic carcinoma because there are multiple (three) nodules found.
In the lung, metastases are more common than a primary neoplasm. It could be that this lady has breast, colon, or bladder cancer.

, ~ ~ p , ,.,,.
Previous Next Lab Values Calculator Review Help Pause
hi Comprehensive Basic Science Self-Assessment - Google Chrome x
i starttest.com/ITDVersions/19.3.0.1/ITDStart.aspx?SVC= 701083bd-6fcd-4283-9abd-818a8e3d62c7

Exam Section 4: Item 40 of 50 National Board of Medical Examiners Time Remaining:


• Mark Comprehensive Basic Science Self-Assessment 4 hr 54 min 14 sec

40. A cohort study is done to evaluate the association between use of video display terminals (VDTs) by women and the risk for congenital heart disease in their offspring. The relative
risk (risk ratio) of congenital heart disease in newborns born to women who work for 6 or more hours daily using a VDT is 1.1 (95% confidence interval: 0.8-1.4) compared with
women who are not exposed to VDTs. Which of the following is the p-value calculated from a chi square test?

A) 0
B) 0 < p < 0.01
C) 0.01 < p < 0.05
D) 0.05 < p < 1.0
E) p > 1.0

D. Page 274/262.

Great question to test the basic understanding of this biostatistical concept of confidence interval.
When the 95% CI for an odds ratio or relative risk (this question) has 1 in it, the null hypothesis is not rejected and the data is not significant.
Since the data is not significant, the p value must be more than 0.05, which is choice D.

The p value cannot be more than 1 because that would mean there is more than 100% chance the null hypothesis is true, and that is unreal.
When data is significant, there is less than 5% of a chance that the data is by chance and the null hypothesis is true.

, ~ ~ p , ,.,,.
Previous Next Lab Values Calculator Review Help Pause
hi Comprehensive Basic Science Self-Assessment - Google Chrome x
i starttest.com/ITDVersions/19.3.0.1/ITDStart.aspx?SVC= 701083bd-6fcd-4283-9abd-818a8e3d62c7

Exam Section 4: Item 41 of 50 National Board of Medical Examiners Time Remaining:


• Mark Comprehensive Basic Science Self-Assessment 4 hr 54 min 9 sec

41. Drug Xis given to a 25-year-old normal subject. This drug causes resting heart rate to increase from 62/min to 74/min. Prior to administration of the drug, the subject's heart rate
increased to 150/min with exercise; after administration of Drug X, his heart rate increased to 98/min at the same level of exercise. The mechanism of action of Drug X most likely
involves which of the following?

A) Antagonist at dopaminergic receptors


B) Full agonist at angiotensin II receptors
C) Full agonist at ADH (vasopressin) receptors
D) Partial agonist at 13-adrenergic receptors
E) Partial agonist at serotoninergic receptors

D.

Great question to test the understanding of partial vs full agonists. This concept plays a bigger role when it comes to opioid receptor agonists.

Partial agonists have weak agonist activity on their own (thus in this case it causes HR to increase, b-adrenergic effect) but when an actual
agonist is present (aka when you are exercising, you are producing NE and E that have full b-agonist effects), partial agonist actually have
a mild antagonist effect (thus the heart rate decreases).

, ~ ~ p , ,.,,.
Previous Next Lab Values Calculator Review Help Pause
hi Comprehensive Basic Science Self-Assessment - Google Chrome x
i starttest.com/ITDVersions/19.3.0.1/ITDStart.aspx?SVC= 701083bd-6fcd-4283-9abd-818a8e3d62c7

Exam Section 4: Item 42 of 50 National Board of Medical Examiners Time Remaining:


• Mark Comprehensive Basic Science Self-Assessment 4 hr 54 min 5 sec

42. Which of the following terms best describes the fibrous proteins that form the two-dimensional network on the inner surface of the nuclear membrane?

A) Actin filaments
U B) Fibronectin
U C) Granum
D) Lamins
E) Tubulin

D. Not in FA.
Lamins (not to be confused with laminins, which work with fibronectin and integrin to connect cells to the basement membrane)
are a type 5 intermediate filament that interacts with the inner nuclear membrane protines to form nuclear lamina on the inferior
nuclear envelope.

Really unnecessary, stupid question.

, ~ ~ p , ,.,,.
Previous Next Lab Values Calculator Review Help Pause
hi Comprehensive Basic Science Self-Assessment - Google Chrome x
i starttest.com/ITDVersions/19.3.0.1/ITDStart.aspx?SVC= 701083bd-6fcd-4283-9abd-818a8e3d62c7

Exam Section 4: Item 43 of 50 National Board of Medical Examiners Time Remaining:


• Mark Comprehensive Basic Science Self-Assessment 4 hr 54 min O sec

43. A 7-year-old girl is brought to the emergency department (ED) because of a 1-hour history of excessive sleepiness and slurring of speech. The mother reports that the girl seemed to
be feeling well earlier in the day and had eaten lunch 2 hours before this episode. Fingerstick blood glucose concentration ordered on arrival in the ED is 42 mg/dl. Intravenous
dextrose is administered, and 5 minutes later her symptoms resolve. The patient has a history of three previous ED visits for similar symptoms during the past year. Her pulse is
94/min, respirations are 24/min , and blood pressure is 102/64 mm Hg. Physical examination shows no abnormalities. Results of serum studies obtained prior to administration of
dextrose are now available and are shown:
C-peptide 0.5 ng/ml (N=0.8-3.1)
Insulin 32 µU/ml (fasting N=5-20)
Beta-hydroxybutyrate 1.2 mg/dl (N=0.2-3.0)

Which of the following is the most likely diagnosis?

A) Factitious disorder imposed on another


u B) Glucose 6-phosphatase deficiency
C) Nesidioblastosis
U D) Pyruvate carboxylase deficiency
lJ E) Type 1 diabetes mellitus

A. Page 554/619.
This is a clear and good easy question. Manchausen syndrome by proxy.
The physician should report this parent because this is child abuse.
The high insulin with low C-peptide levels indicates that the girl was given exogenous insulin.
This patient has ketosis and normal response to dextrose. Her metabolic pathways are working fine.

, ~ ~ p , ,.,,.
Previous Next Lab Values Calculator Review Help Pause
hi Comprehensive Basic Science Self-Assessment - Google Chrome x
i starttest.com/ITDVersions/19.3.0.1/ITDStart.aspx?SVC= 701083bd-6fcd-4283-9abd-818a8e3d62c7

Exam Section 4: Item 44 of 50 National Board of Medical Examiners Time Remaining:


• Mark Comprehensive Basic Science Self-Assessment 4 hr 53 min 56 sec

44. A 55-year-old man comes to the physician because of a 2-week history of palpitations and anxiety. He has primary hypothyroidism treated with levothyroxine. He says that he has
been taking twice the prescribed dosage of his medication for the past 2 months. His pulse is 104/min, and blood pressure is 146/88 mm Hg. Physical examination shows a fine
resting tremor. Which of the following sets of laboratory findings is most likely in this patient?

Serum Serum
Free Thyroxine (FT J Free Triiodothyronine (FT :J Thyroidal Iodine Uptake
U A) Increased increased decreased
U B) Increased decreased increased
u C) Increased decreased decreased
D) Decreased increased increased
E) Decreased increased decreased
F) Decreased decreased increased

A. Page 349/383.
Great question about levothyroxine (T4) vs liothyronine (T3).
Excess levothyroxine intake will increase T4, which will be converted into T3 (this conversion is not regulated).
Since there is so much active thyroid hormone, TSH will be extremely low and the thyroid gland will not take up iodine because more
thyroid hormone production will not be necessary.

Some people may have felt that the answer would be B because ingesting T4 would prevent the activation into T3 by peripheral 5'-deiodinase.
However, that enzyme just converts any T4 in circulation into T3, even if there is abundant T3.
This is why taking excess levothyroxine can be a form of abusing the medication for weight loss.
If 5'-deiodinase worked on a feedback mechanism, that would be amazing and overdosing levothyroxine would have no major effect.

Liothyronine is T3, so when you ingest a lot of T3 you stop making more T4 thus you also stop taking up iodine (choice E).

, ~ ~ p , ,.,,.
Previous Next Lab Values Calculator Review Help Pause
hi Comprehensive Basic Science Self-Assessment - Google Chrome x
i starttest.com/ITDVersions/19.3.0.1/ITDStart.aspx?SVC= 701083bd-6fcd-4283-9abd-818a8e3d62c7

Exam Section 4: Item 45 of 50 National Board of Medical Examiners Time Remaining:


• Mark Comprehensive Basic Science Self-Assessment 4 hr 53 min 51 sec

45. A 55-year-old man with alcoholism is brought to the emergency department 30 minutes after consuming a bottle of methanol. In order to decrease methanol toxicity in this patient,
the physician recommends that the patient be treated with ethanol. To increase the Kmof hepatic alcohol dehydrogenase for methanol in this patient the ethanol must act as which of
the following?

A) Competitive inhibitor
B) Complete agonist
C) Feedback regulator
D) Noncompetitive antagonist
E) Positive allosteric effector

A. Page 72. Page 232/242.


The question is on quite a hilarious method, but the concept is clear. If you want to increase Km, what do you have to do?
Well, Km is inversely related to the affinity of the enzyme for its substrate. Raising Km means that the enzyme is not attaching to the substrate.
This occurs when another substrate is more attractive, so the enzyme now attaches to that.

Remember the mnemonics from my book and videos and remember the lessons.

Km Increase = Kompetitive Inhibitors.

Think of Km as Kim kardashian and relationship-drama.


To increase the kim kardashian level of the scenario, introduce new competition of hot women and the enzyme (the man, the one running everything)
will now be more attracted to the new hot woman. Etc.

Very easy good question :)


The hospital mustve ran out of fomepizole lol.

, ~ ~ p , ,.,,.
Previous Next Lab Values Calculator Review Help Pause
hi Comprehensive Basic Science Self-Assessment - Google Chrome x
i starttest.com/ITDVersions/19.3.0.1/ITDStart.aspx?SVC= 701083bd-6fcd-4283-9abd-818a8e3d62c7

Exam Section 4: Item 46 of 50 National Board of Medical Examiners Time Remaining:


• Mark Comprehensive Basic Science Self-Assessment 4 hr 53 min 44 sec

46. A 21-year-old woman with asthma comes to the physician because her current medication regimen is not relieving her symptoms. The physician tel ls the patient about a clinical trial
of a new drug for asthma. This trial is a large, randomized , prospective, double-blind study on volunteers with asthma. This trial is most likely occurring at which of the following
phases of drug development?

A) Phase 0
B) Phase 1
C) Phase 2
D) Phase 3
E) Phase 4

D. Page 256/266.
Never get these wrong.
Phase 3 revolves around 10^3+ people (so 2000 or 5000, etc) who have the disease and now you compare the new drug to the current standard.
This is why at this point it is double-blinded and randomized, because this patient MIGHT get the current gold standard.
Whatever drug she gets, her data will be used to compare with other patients taking the other drug and phase 3 checks for improvement.

Phase 0 trials are also known as human micro-dosing studies and are designed to speed up the development of promising drugs or imaging agents
by establishing very early on whether the drug or agent behaves in human subjects as was expected from pre-clinical studies.

, ~ ~ p , ,.,,.
Previous Next Lab Values Calculator Review Help Pause
hi Comprehensive Basic Science Self-Assessment - Google Chrome x
i starttest.com/ITDVersions/19.3.0.1/ITDStart.aspx?SVC= 701083bd-6fcd-4283-9abd-818a8e3d62c7

Exam Section 4: Item 47 of 50 National Board of Medical Examiners Time Remaining:


• Mark Comprehensive Basic Science Self-Assessment 4 hr 53 min 32 sec

47. A 27-year-old man who works from home as a software engineer comes to the physician with his girlfriend because she is worried that he may be depressed. His girlfriend tells the
physician, "I practically have to drag him out of the house to see my family or friends, and then when we get there he refuses to talk to them or interact with them. He just stands
around looking uncomfortable, and then he asks to go home half an hour after we arrive." She says that he seems fine when he is alone with her or with his own family and friends.
After she leaves the room , he tells the physician, "I wish I could have relationships with her family and friends, but I can't. I just know they'll hate me once they get to know me." He
has been this way for as long as he can remember. This patient most likely has which of the following types of personality disorders?

A) Antisocial
B) Avoidant
C) Paranoid
lJ D) Schizoid
U E) Schizotypal

B. Page 553/618 of FA2019.


This patient is exhibiting the exact actions Sheldon Cooper of Big Bang Theory exhibited in the show after he got married.
This is an avoidant personality where the person WANTS to have relationships with people but is hypersensitive to criticism and
socially inhibited. This is in comparison to schizoid, where the person is like a robot and does not care for relationships and is
perfectly fine on their own.

Antisocial is a trick answer that someone who hasnt mastered the personality disorders would pick.
In normal, every day language, this patient could be described to be antisocial.
However, medically, this patient is not antisocial (a cluster B personality resonant of Cardi B).
Antisocial people are very mean and have a disregard for the rights of others and can be manipulative and hostile.

, ~ ~ p , ,.,,.
Previous Next Lab Values Calculator Review Help Pause
hi Comprehensive Basic Science Self-Assessment - Google Chrome x
i starttest.com/ITDVersions/19.3.0.1/ITDStart.aspx?SVC= 701083bd-6fcd-4283-9abd-818a8e3d62c7

Exam Section 4: Item 48 of 50 National Board of Medical Examiners Time Remaining:


• Mark Comprehensive Basic Science Self-Assessment 4 hr 53 min 28 sec

48. A 38-year-old woman comes to the physician because of blood-tinged discharge from her right breast for 3 months. Menses have occurred at regular 28-day intervals. She takes no
medications. There is no family history of breast cancer. Physical examination shows no breast masses and no palpable axillary adenopathy. Bloody discharge can be expressed
from the upper outer corner of the right nipple. Mammography shows no abnormalities. Which of the following is the most likely cause of the discharge?

A) Fibrocystic changes of the breast


B) Fibrosarcoma
C) lntraductal papilloma
D) Paget disease of the breast
E) Prolactinoma

C. Page 635/716.
This is the most viable answer choice to pick even without complete knowledge of all the various breast cancers.
The patient has no mass on exam, no history of breast cancer, and doesnt take meds.
Bleeding from her nipple must mean intraductal problems and C is the best for that.
This is the most common cause of painless blood discharge from the nipple in a woman of reproductive age.

A= Most common in premenopausal women. Would find painful lump possibly bilaterally. Sclerosing adenosis has risk of cancer.
Epithelial hyperplasia has risk of cancer if you see atypical cells.
B= No mention in FA of a fibrosarcoma. But a fibroadenoma would be a well defined mobile tender mass that grows in size with estrogen.
A fibrosarcoma is extremely rare and would also have a palpable mass.
D= Paget disease would have the eczematous patches over the areola.
E= Prolactinoma would simply cause milk discharge possibly but not blood discharge.

, ~ ~ p , ,.,,.
Previous Next Lab Values Calculator Review Help Pause
hi Comprehensive Basic Science Self-Assessment - Google Chrome x
i starttest.com/ITDVersions/19.3.0.1/ITDStart.aspx?SVC= 701083bd-6fcd-4283-9abd-818a8e3d62c7

Exam Section 4: Item 49 of 50 National Board of Medical Examiners Time Remaining:


• Mark Comprehensive Basic Science Self-Assessment 4 hr 53 min 23 sec

49. An 18-year-old woman is brought to the physician because of sharp chest pain and shortness of breath 1 hour after
receiving a nonpenetrating injury during a rugby game. Her respirations are 22/min. Physical examination shows
decreased breath sounds and increased tympany to percussion on the right. A chest x-ray is shown. This patient is at
greatest risk for developing which of the following complications?

A) Amyloidosis
U B) Carcinoid tumor
C) Diffuse alveolar damage
U D) Empyema
E) Pulmonary edema
F) Pulmonary embolism
G) Respiratory acidosis

G. Page 667/765.
This patient has a pneumothorax. The right lung is just gone and its all black due to air.
WHAT THE $#@! KIND OF CHEST XRAY IS THIS?!! WHY??
You cannot visualize the trachea at all but this is probably a tension pneumothorax with tracheal deviation towards the left.
Honestly, none of the choices make sense once you realize this patient has a pneumothorax, leaving only G to be even remotely feasible.
Arterial blood gas studies may show respiratory alkalosis caused by a decrease in CO2 as a result of tachypnea
but later hypoxemia, hypercapnia, and acidosis would be seen.

, ~ ~ p , ,.,,.
Previous Next Lab Values Calculator Review Help Pause
hi Comprehensive Basic Science Self-Assessment - Google Chrome x
i starttest.com/ITDVersions/19.3.0.1/ITDStart.aspx?SVC= 701083bd-6fcd-4283-9abd-818a8e3d62c7

Exam Section 4: Item 50 of 50 National Board of Medical Examiners Time Remaining:


• Mark Comprehensive Basic Science Self-Assessment 4 hr 53 min 18 sec

50. A 15-year-old girl is brought to the physician because of a 3-day history of fever, sore throat, and malaise. Her temperature is 39.2°C (102.6°F). Physical examination shows diffuse
pharyngeal erythema, moderately enlarged tonsils, and tender anterior and posterior cervical lymphadenopathy. A complete blood count shows:
Leukocyte count 19,500/mm 3 (N=3500-10,500)
Segmented neutrophils 30%
Bands 7%
Eosinophils 2%
Lymphocytes 25%
Lymphocytes, atypical 30%
Monocytes 6%

Incubation of this patient's serum with sheep erythrocytes results in agglutination. The atypical lymphocytes in this patient are most likely which of the following cell types?

A) B lymphocytes
B) CD4+ T lymphocytes
U C) CD8+ T lymphocytes
D) FOXP3-expressing regulatory T lymphocytes
U E) Natural killer cells

C. Page 165/170 of FA2019.


I LOVE THIS QUESTION. Ive spoken about this topic numerous times and this is a great question.
This patient has infectious mononucleosis due to HHV4 Epstein Barr Virus (remember that Barr is 4 letters so HHV4).
The sheep agglutination is the heterophile antibody test.
These patients have reactive T cells presenting as atypical lymphocytes, while the infected B cells look totally normal.
For viral infections, cytotoxic CD8+ T cells would be the ones reacting.

In comparison, AIDS patients with PCNSL have atypical lymphocytes that are actually the infected B cells.

, ~ ~ p , ,.,,.
Previous Next Lab Values Calculator Review Help Pause

You might also like